PICU Boards Flashcards

1
Q

A 3-year-old male child is found playing with an open bottle of his grandfather’s pain medications (opioids). He is sleepy, so his mother rushes him to the ED, which is not far from her house. Upon arrival to the ED, he has perioral cyanosis and the arterial saturations are reading 88% on room air. The rest of his vital signs are within normal range except for a respiratory rate of 12 breaths per minute. His lung examination is normal and his neurological examination is significant for lethargy. Which one of the following options best describes his alveolar arterial oxygenation (A-a) gradient and arterial partial pressure of CO2 (PaCO2) levels?

A - Normal A-a gradient and increased PaCO2

B - Increased A-a gradient and increased PaCO2

C - Normal A-a gradient and decreased PaCO2

D - Increased A-a gradient and normal PaCO2

E - Normal A-a gradient and normal PaCO2

A

A - Normal A-a gradient and increased PaCO2

The patient in this scenario has ingested pain medication which has depressed his respiratory drive. Opioids are known to depress respiratory drive, leading to hypoventilation. Hypoventilation leads to an increase in PaCO2 (making options B, C, D, and E all incorrect). His SaO2 is low because of arterial hypoxemia. Arterial hypoxemia indicates limitation of pulmonary gas exchange. There are 4 reasons for limitations of pulmonary gas exchange: (1) hypoventilation; (2) shunts; (3) diffusion abnormalities; (4) ventilation/perfusion mismatch. The A-a gradient is increased in all conditions except for hypoventilation (making options B and D incorrect). Hence, this child will have a normal A-a gradient and increased PaCO2 (making option A the correct response option).

How well did you know this?
1
Not at all
2
3
4
5
Perfectly
1
Q

The internal diameter of the trachea of an infant is 8 mm. As a result of airway edema, consequent to viral infection (croup), the diameter is now reduced to 4 mm. Which one of the following options will be its effect on the resistance to airflow in the airways?

A - Remain the same

B - Double

C - Increase 4 times

D - Increase by 8 times

E - Increase by 16 times

A

E - Increase by 16 times

The relation between flow of air through a tube and radius of the tube was described by French physician Poiseuille. In straight circular tubes, the volume flow rate is given by the following equation: V = Pπr4 / 8 ηL. Where P is driving pressure, r = radius, η = viscosity, and L = length. Because flow resistance is driving pressure divided by flow, the equation can be rearranged as the following: R= 8 ηL / πr4. Tube radius is of critical importance, if the radius is halved, the resistance increases 16 fold. However, doubling the length only doubles the resistance

How well did you know this?
1
Not at all
2
3
4
5
Perfectly
1
Q

A 9-year-old girl presents with acute decompensated heart failure. She has a history of adequately treated end-stage heart failure and is currently hemodynamically stable. Which one of the following treatments should be administered following admittance to the cardiac intensive care unit?

A - Dopamine

B - Epinephrine

C - Positive pressure ventilation

D - Milrinone

E - Sedation

A

D - Milrinone

This question describes the most appropriate treatment for a child in end-stage heart failure with acute decompensated heart failure who is currently hemodynamically stable. Options A and B are incorrect because dopamine, dobutamine, and epinephrine are typically administered to children with acute decompensated heart failure who are not hemodynamically stable. Option C is incorrect because the patient does not require positive pressure ventilation at this time because she is hemodynamically stable. Option D is correct as inotropic treatment with either milrinone or dobutamine should be initiated in patients with acute decompensated heart failure that are hemodynamically stable. Option E is incorrect because sedation should be administered in cases of left ventricular failure and pulmonary hypertension.

How well did you know this?
1
Not at all
2
3
4
5
Perfectly
1
Q

A 15-year-old male (weight 40 Kg) admitted with hyponatremic dehydration with serum sodium of 120 mEq/liter. If you want to correct his sodium deficit using a 3% saline infusion, what would be the maximum rate of infusion of the 3% solution (assuming, total body water 60% of body weight, target serum sodium 135 mEq/lt and target maximum increase in serum sodium 0.5 mEq/lt/hour)?

a. 20 ml/hour
b. 23 ml/hour
c. 26 ml/hour
d. 29 ml/hour
e. 32 ml/hour

A

b. 23 ml/hour

When corrective therapy requires the inclusion of isotonic saline or hypertonic saline, the replacement therapy can be guided by the calculated sodium deficit. This is determined as follows: Sodium deficit (mEq) = normal TBW × (135 - current PNa). Normal TBW (in lt) is 60% of the lean body weight (in kg) in males and 50% of the lean body weight in women. Thus, for a 40 kg male with plasma sodium of 120, the sodium deficit would be 0.6×40 × (135-120)= 360 mEq. Due to the fact that 3% sodium chloride contains 513 mEq of sodium/lt, the volume of hypertonic saline needed to correct sodium deficit of 360 mEq will be 0.700 lt. Using a maximum rate of rise of 0.5 mEq/lt/hr for plasma sodium, the sodium concentration deficit of 15 mEq/lt should be corrected over at least 30 hours. Thus, the maximum rate of hypertonic fluid administration will be 700/30 = 23 cc/hr.

How well did you know this?
1
Not at all
2
3
4
5
Perfectly
1
Q

Which one of the following options best explains pulsus paradoxus in severe asthma?

A - Inhaled ß2 agonist causes diastolic hypotension leading to myocardial ischemia, as cardiac blood supply occurs during diastole

B - Children with asthma are dehydrated on presentation—this leads to pulsus paradoxus

C - Children with severe asthma have pericardial effusion due to inflammation that leads to pulsus paradoxus

D - Frequent asthma exacerbations lead to chronic obstructive pulmonary disease (COPD)—a COPD exacerbation causes pulsus paradoxus

E - Hyperinflation compromises right ventricular function and left ventricular afterload is increased leading to drop in systolic blood pressure

A

E - Hyperinflation compromises right ventricular function and left ventricular afterload is increased leading to drop in systolic blood pressure

Hyperinflation in severe asthma has significant cardiac consequences. Pulsus paradoxus is exaggerated drop in blood pressure during inspiration. Normally, there is a drop of 10 mmHg in blood pressure during inspiration. Hyperinflation stretches the pulmonary vasculature, causing an increase in pulmonary vascular resistance and decrease right ventricular function. Hyperinflation also increases the left ventricular afterload especially during inspiration. This leads to exaggeration of drop of blood pressure during inspiration in severe asthma. Hence, option E is the correct. Inhaled ß2 agonists are known to cause diastolic hypotension; however, that does not explain the variation of blood pressures during different phases of the respiratory cycle. Thus, option A is incorrect. Option B is incorrect because, though children with asthma are usually dehydrated, pulsus paradoxus cannot be explained. In severe dehydration leading to shock, hypotension will be evident. Option C is incorrect as there is no evidence of pericardial inflammation in asthmatics. In pericardial effusion, there is limited space of the pericardial sac. During inspiration, there is an increase in the venous return and the right ventricle expands; however, bulges in the left ventricle, causing a drop in blood pressure, limits the venous return during inspiration, therefore causing greater than drop in blood pressure. Option D is incorrect as there is no evidence that asthma exacerbations lead to COPD. In adults, COPD exacerbation is an important cause of pulsus paradoxus.

How well did you know this?
1
Not at all
2
3
4
5
Perfectly
1
Q

A new urine marker has been identified for early detection of acute kidney injury (AKI). Your ICU is interested in participating in a multisite clinical study comparing patients with serum markers consistent with AKI and results on the new urine test. Assume that the serum markers are 100% accurate in identifying AKI. The following results are obtained:

  • 80 patients with AKI confirmed by serum marker and with the urine marker
  • 20 patients with AKI confirmed by serum marker and without the urine marker • 40 patients without AKI by serum marker test and with the urine marker
  • 60 patients without AKI by serum marker test and without the urine marker

What is the negative predictive value of this test?

A. 40%

B. 60%

C. 67%

D. 75%

E. 80%

A

Answer: D - 75%

Negative Predictive Value = True Negative/(True Negative + False Negative) = 60/(60+20) = 0.75 = 75%

How well did you know this?
1
Not at all
2
3
4
5
Perfectly
2
Q

A 3-year-old is admitted to the PICU with ingestion of an unknown substance. She is awake and at her baseline level of mentation. Her vitals are as follows: HR 68; BP 65/24; RR 20; SPO2 99% on RA. On ECG, she has prolongation of the PR interval but a normal QT interval. She has capillary refill of about 3 seconds peripherally. A small dose of subcutaneous epinephrine results in improvement in her BP to 95/55 and increase in her HR to 110. Which one of the following substances is most like to be at fault?

A - Beta-blocker

B - Nondihydropyridine calcium-channel blocker

C - Dihydropyridine calcium-channel blocker

D - Tricyclic antidepressant

E - Clonidine

A

B - Nondihydropyridine calcium-channel blocker

The toxidrome described is that of a nondihydropyridine calcium-channel blocker (bradycardia, hypotension, sluggish capillary refill, normal mentation until hypotension results in altered mental status, PR prolongation with normal QT interval). If beta-blockers were at fault, epinephrine would not reverse symptoms. If dihydropyridine CCB were at fault, symptoms would more likely be hypotension with reflexive tachycardia. Tricyclic antidepressant would cause hypotension but likely prolonged QT interval as well as sedation and likely metabolic acidosis resulting in increased respiratory drive as a compensatory mechanism. Clonidine would result in similar symptoms to those seen but the patient would be expected to have sedation as an additional side effect.

How well did you know this?
1
Not at all
2
3
4
5
Perfectly
2
Q

A 10-year-old child is on high frequency oscillatory ventilator (HFOV) for 2 days because of pneumonia and hypoxemic respiratory failure. The ventilator settings are mean airway pressure (MAP) 34 cm of H20, delta P 60 Hertz 6, inspiratory time 33%. His FiO2 is 0.60. Arterial blood gas shows a pH of 7.2; PaO2 78 mmHg; Pa CO2 80 mmHg and base deficit of 1. CXR does not show a pneumothorax and there is 9-rib expansion. Which one of the following options will help bring the pH to 7.3?

A - Deflating the endotracheal tube cuff

B - Increasing the MAP to 36 cm of H20

C - Increasing FiO2 to 0.80

D - Increasing Hz to 8

E - Decreasing MAP to 32 cm of H20

A

A - Deflating the endotracheal tube cuff

Creating endotracheal tube cuff leak is an alternative way to enhance CO2 clearance. Deflating the endotracheal tube creates a path for the CO2 to escape outside of the endotracheal tube. Thus, option A is correct. Option B is incorrect, as increasing the MAP affects alveolar recruitment and oxygenation. At high levels, MAP may worsen hypercarbia because of hyperinflation. Option C is wrong, because increasing FiO2 will not have any effect on CO2 clearance. Option D is incorrect, because increasing Hz will actually worsen hypercarbia. Decreasing MAP will cause atelectasis and cause hypoxemia.

How well did you know this?
1
Not at all
2
3
4
5
Perfectly
2
Q

A 3-year-old girl who weighs 20 kg is admitted to PICU with diagnosis of status asthmaticus. She receives 5 mg/kg of aminophylline loading dose over 20 minutes. Blood level of aminophylline immediately after loading dose is 12 mcg/ml. Assuming no elimination or clearance has occurred, which one of the following values represents the correct volume of distribution of aminophylline in this patient?

A - 8.33 L

B - 0.42 L

C - 0.83 L

D - 60 L

E - Not able to determine from the given information

A

A - 8.33 L

The formula to calculate volume of distribution (Vd) is as follows: Vd = total amount of drug in the body (drug) / concentration of drug in the blood or plasma (Cp). In this patient total drug given is 5 mg x 20 (weight of the patient) is 100 mg. Substituting the values in the formula Vd = 100 / 12 = 8.33 L. Option A is the correct response because this is the value obtained from the formula. Option B is not correct. If it were to be volume of distribution/kg then Vd would have been 0.42 L/kg. Options C, D, and E are all incorrect.

How well did you know this?
1
Not at all
2
3
4
5
Perfectly
2
Q

Which one of the following factors will not alter a patient’s alveolar PO2?

A - Right-left cardiac shunt

B - High altitude

C - High carbohydrate diet

D - FiO2

E - Hypopnea

A

A - Right-left cardiac shunt

This question’s answer is described by the alveolar gas equation. The equation is as follows (where PAO2 = alveolar PO2, Atm = atmospheric pressure, PH2O = partial pressure of water vapor, PaCO2 = partial pressure of arterial CO2, R = respiratory quotient which is a constant determined by an organism’s CO2 eliminated per O2 consumed. R is increased toward 1 with high carbohydrate diet from a normal value of 0.8): PAO2 = FiO2 x (Atm - PH2O) - PaCO2/R. A right to left cardiac shunt will not affect alveolar PO2 but it will affect the A-a gradient.

How well did you know this?
1
Not at all
2
3
4
5
Perfectly
2
Q

In patients with heart failure, which of the following mediators that compose the neurohumoral response functions as a vasodilator?

A. Endothelin

B. Angiotensin II

C. Arginine vasopressin

D. Norepinephrine

E. B-type natriuretic peptide

A

Answer: E - B-type natriuretic peptide

In patients with heart failure, norepinephrine, angiotensin II, arginine vasopressin, and endothelin are vasoconstrictors that result in cardiac stimulation and fluid retention. B-type natriuretic peptide (BNP) is both a vasodilator and a natriuretic. The natriuretic peptides function as both hormones and neurotransmitters. There are 3 types: A-type, B-type, and C-type. BNP is secreted by the ventricular myocardium in response to elevated end-diastolic pressure and volume. It has diuretic, natriuretic, and venous and arterial vasodilation properties. The latter occur via cyclic guanosine monophosphate generation. BNP has been used as a diagnostic and prognostic marker in patients with congestive heart failure.

How well did you know this?
1
Not at all
2
3
4
5
Perfectly
4
Q

Which one of the following statements regarding respiratory quotient (RQ) is accurate?

a. It is the ratio at any given time of the volume of CO2 produced to the volume of O2 consumed
b. RQ of fat is 1.0 and carbohydrate is 0.7
c. RQ for protein is 0.6
d. RQ of brain is 0.97-0.99
e. RQ is affected by multiple factors including exercise/metabolic acidosis

A

d. RQ of brain is 0.97-0.99

The respiratory quotient is the ratio in the steady state of the volume of CO2 produced to the volume of oxygen consumed per unit of time. The RQ of carbohydrate is 1, and that of fat is about 0.7. Determining the RQ of proteins in the body is a complex process, but an average value of 0.82 has been calculated. RQ should be distinguished from the respiratory exchange ratio (R), which is the ratio of CO2 produced to O2 consumed at any given time whether or not equilibrium has been reached. R is affected by factors other than metabolism. The oxygen consumption and CO2 production of an organ can be calculated at equilibrium, by multiplying its blood flow per unit of time by the arteriovenous difference for O2 and CO2 across the organs, RQ of the brain is regularly 0.97–0.99, indicating that its principle but not its only fuel is carbohydrate.

How well did you know this?
1
Not at all
2
3
4
5
Perfectly
4
Q

In a pediatric patient who is intubated and on mechanical ventilation with SIMV volume control mode of ventilation, which one of the following options is the dependent control variable for the flow controller?

A - Pressure

B - Flow

C - Volume

D - Resistance of the ventilator circuit

E - Compliance of the lung

A

A - Pressure

During SIMV volume control, the mode is controlled by a flow controller. The dependent variable is pressure and the independent variable is flow. Volume is the limiting variable. Flow is determined by pressure divided by flow but does not affect the dependent variable. Compliance of the lung has no effect on the control variable but may affect the limit variable.

How well did you know this?
1
Not at all
2
3
4
5
Perfectly
6
Q

With regard to hemoglobin oxygen dissociation curve, which one of the following statements is correct?

A - The curve shifts to the left with an increase in temperature

B - Oxygen affinity of hemoglobin increases with acidosis

C - The curve shifts to the right with an increase in thyroid hormones

D - There is a decrease in p50 with exercise

E - The curve shifts to the left with increase in HbF

A

C - The curve shifts to the right with an increase in thyroid hormones

The oxygen dissociation curve relates to the percent saturation of oxygen carrying power of Hb. It has a characteristic sigmoid shape. Depending on the affinity of Hb for oxygen, the curve shifts to the right or to the left. Affinity of Hb for oxygen is affected by multiple physiologic variables. A shift to right is associated with decrease in affinity. A convenient index of such shifts is p50, the pO2 at which Hb is half-saturated with oxygen. The higher the p50, the lower the affinity. Oxygen dissociation curve is shifted to the right (decrease in affinity, increase in p50) with fever, acidosis, and an increase in BPG. Other factors shifting the curve to the right include the following: thyroid/growth hormone/androgens (by increasing concentration of BPG); exercise (increase in p50 as temperature rises in active tissues and CO2 accumulates, lowering pH); high altitude (increase in BPG); decrease in HbF (HbF has high affinity, which facilitates the movement of oxygen from mother to fetus); anemia (increase in BPG); abnormal Hb (high oxygen affinity causes tissue hypoxia, increased PRBC production, polycythemia). BPG decreases in stored blood, so PRBC transfusion lead to increased oxygen affinity and less tissue oxygen delivery.

How well did you know this?
1
Not at all
2
3
4
5
Perfectly
6
Q

Which one of the following pathogens is a common cause of necrotizing fasciitis in children?

a. Group-A Streptococcus (GAS)
b. Streptococcus pyogenes
c. Staphylococcus aureus
d. E. coli
e. Pseudomonas

A

a. Group-A Streptococcus (GAS)

Necrotizing fasciitis is a deep seated infection of the subcutaneous tissue that rapidly invades and destroys underlying fascia and fat despite often sparing the overlying skin and muscle. It is commonly caused by GAS; other inciting pathogen includes clostridium perfringens and clostridium septicum.

How well did you know this?
1
Not at all
2
3
4
5
Perfectly
6
Q

Which one of the following statements is not accurate regarding case-control studies?

A - Decreasing the number of cases strengthens the study

B - Cases are identified before the controls

C - Helpful to study rare diseases

D - Increasing the number of cases strengthens the study

E - Case and controls are identified simultaneously

A

A - Decreasing the number of cases strengthens the study

Case-control studies are ideal for rare diseases, as conducting a randomized controlled trial in such a disease would not be feasible (making option C incorrect). In case-control studies, cases are selected before the controls—hence, options B and E are incorrect. Increasing the number of cases increases the power of the study; however, this may the limiting step in case-control studies. Hence, option D is incorrect while option A is correct.

How well did you know this?
1
Not at all
2
3
4
5
Perfectly
6
Q

A 6-month-old boy with a single ventricle that was previously palliated with an aortopulmonary shunt returns from surgery after undergoing a bidirectional cavopulmonary anastomosis. His oxygen saturation is 65%. What is the most likely cause for the systemic desaturation?

A. Pulmonary arteriovenous malformations

B. Decompressing venous collaterals

C. Atrioventricular valve regurgitation

D. Increased pulmonary vascular resistance

A

Answer: D - Increased PVR

Possible causes of hypoxemia following a bidirectional cavopulmonary anastomosis can be grouped into 3 categories: pulmonary venous desaturation, systemic venous desaturation, and decreased pulmonary blood flow. Decompressing venous collaterals are one cause of decreased pulmonary blood flow, along with undiagnosed contralateral superior vena cava (SVC). In bidirectional cavopulmonary circulation, any increase in pulmonary vascular resistance can open decompressing veins, resulting in a shunt of venous blood into the heart. Factors related to development of decompressing venous collaterals include bilateral SVC, elevated SVC pressure, and high early postoperative transpulmonary gradient. Vessels may be closed on the pre-Glenn cardiac catheterization, but a left SVC-to-coronary sinus collateral may develop postoperatively.

Pulmonary arteriovenous malformations typically cause hypoxemia months to years after the surgery. Atrioventricular valve regurgitation is less likely to affect systemic oxygenation. Increased pulmonary vascular resistance causes systemic desaturation.

How well did you know this?
1
Not at all
2
3
4
5
Perfectly
7
Q

A newborn underwent full correction of interrupted aortic arch (type 1B) and right aortic arch at 1 week of age and recovered uneventfully. Postoperative evaluation by echocardiography showed normal function, trivial residual ventricular septal defect, and no arch gradient. The infant is now 3 weeks old and has had difficulty with feeding due to poor coordination of suck and swallow. The infant is receiving nasogastric tube feeds. Current medications include furosemide, 2 mg/kg every 8 hours, and aspirin, 41 mg daily, both via nasogastric tube. You are called urgently to evaluate him because the ward team observed a 2-minute generalized seizure. He received a dose of midazolam, 0.1 mg/kg IV, and bedside glucose measurement indicated no hypoglycemia (90 mg/dL). The infant is in no distress as HR is 140/min, RR is 30/min, BP is 65/35 mm Hg, and SaO2 is 100%. The most appropriate immediate step in investigation for an etiology for the seizure includes:

A. Arranging electroencephalography

B. Doing an emergency head ultrasonography

C. Ordering chest radiography to rule out aspiration

D. Sending a formal blood glucose sample to the lab

E. Sending blood for serum electrolyte and ionized calcium values

A

Answer: E - Send blood for serum electrolytes and ionized calcium

This patient has DiGeorge syndrome, which is commonly associated with hypoparathyroidism and hypocalcemia. The seizure may or may not be related to hypoglycemia; however, in this scenario, low blood sugar has been ruled out and a correctable cause of seizures would be to investigate the possibility of hypocalcemia. Head ultrasonography and electroencephalography, although important to delineate the underlying etiology, will not direct acute interventions. Chest radiography to rule out aspiration is not indicated since the patient has no respiratory distress.

How well did you know this?
1
Not at all
2
3
4
5
Perfectly
7
Q

Which of the following conditions is represented on the monitor?

A. Hypovolemia

B. Ebstein anomaly

C. Junctional ectopic tachycardia

D. Tricuspid regurgitation

A

Answer: C - Junctional ectopic tachycardia

An atrial tracing has 3 positive waves, a, c, and v, and 2 negative descents, x and y, as shown in Figure B below. The a wave is caused by atrial contraction or systole, the c wave is caused by ventricular contraction or tricuspid valve closure, and the v wave by atrial filling against a closed tricuspid valve. Atrial pressure is commonly monitored following cardiac surgery; analysis of the waveforms can provide important information about cardiac function.

Large or cannon a waves occur when the atrium contracts against a closed or obstructed atrioventricular valve or when there is resistance to ventricular filling. Structural lesions that can produce cannon a waves include tricuspid valve stenosis, pulmonary stenosis, pulmonary atresia, pulmonary hypertension, or a noncompliant right ventricle (due to hypertrophy or diastolic dysfunction). Arrhythmias associated with atrioventricular dyssynchrony can also produce cannon a waves; these include complete heart block and junctional ectopic tachycardia.

Enlarged v waves result when ventricular pressure is transmitted to the atria as seen with tricuspid regurgitation, Ebstein anomaly, or a left ventricle–to–right atrial shunt. Correlation of the observed waveforms on the ECG tracing can help distinguish a waves from v waves.

How well did you know this?
1
Not at all
2
3
4
5
Perfectly
7
Q

A 6-month-old boy is admitted to the pediatric ICU 1 hour after presenting to the clinic with a 2- week history of poor feeding and pallor. A session of acupuncture was done the day before admission and the parents both believe that it helped him to feel better. He has episodic tachypnea (RR of 30-65/min), a temperature of 37.8°C (100°F), HR of 175/min, and BP of 75/46 mm Hg. The lung fields are clear to auscultation, with normal heart sounds. The abdomen is full with adequate bowel sounds and mild erythema at the umbilicus. The extremities are warm and the capillary refill is less than 2 seconds. Past medical history is significant for chronic omphalitis and a bone marrow examination showing a paucity of neutrophils. Laboratory findings reveal a hemoglobin level of 6.4 g/dL, hematocrit of 19%, WBC count of 13,500/μL (0% neutrophils, 45% lymphocytes), sodium level of 134 mEq/L, potassium level of 4.6 mEq/L, and total carbon dioxide level of 23 mEq/L. Which of the following next steps is most appropriate?

A. Administer recombinant human granulocyte colony-stimulating factor (G-CSF), which may improve the patients’ quality of life

B. Consider a 10-mL/kg bolus of fluid with packed red blood cell transfusion and simultaneous infusion of IV antibiotics that includes ceftriaxone

C. Initiate resuscitation with high-flow nasal cannula oxygen followed by a fluid bolus of 30 mL/kg within 15 minutes according to the Surviving Sepsis guidelines.

D. Obtain an interleukin-8 assay to evaluate the immune response of the patient relative to the capacity of the infiltrative cells

E. Order a bone marrow aspiration to confirm a diagnosis of Kostmann syndrome (severe congenital agranulocytosis).

A

Answer: B

This case highlights the dilemmas that clinicians may encounter when faced with a patient with immunosuppression and severe anemia in the era of implementation of sepsis protocols. In addition, the knowledge about severe congenital agranulocytosis as a cause of immunodeficiency and weighing the value of therapeutic interventions is highlighted.

Neutrophils are a major contributor to the infiltrative capacity of the innate immune response and also produce interleukin-8. However, an assay is not essential in the initial management of this patient who is presenting with signs of sepsis.

The implementation of high-flow nasal cannula oxygen is highly recommended for the initial resuscitation of a patient who presents with severe sepsis. In addition, fluid resuscitation is recommended up to 20 mL/kg over 5-10 min, targeting improved perfusion, resolution of urine output, and amelioration of mental status. In this case, there are other factors contributing to the hemodynamic changes that should be considered: the severe anemia and the elevated temperature. Since the capillary refill is less than 2 seconds, aggressive fluid resuscitation could tip the balance and drown the patient into fluid overload and pulmonary congestion in the face of possible high-output failure from the chronic anemia.

The early administration of granulocyte colony-stimulating factor (G-CSF) has been known to improve the immunologic condition of these patients and help them to lead a normal life. However, this is not a critical step in the acute management of the patient at this time.

The absence of neutrophils in the peripheral smear and a bone marrow sample devoid of neutrophil precursors is diagnostic for Kostmann syndrome

Resuscitation with a blood transfusion in cases of severe anemia is considered to be superior to crystalloid or albumin and should be considered as a critical initial step in the management of this patient who is presenting with systemic inflammatory response syndrome and sepsis that is likely to progress to septic shock.

How well did you know this?
1
Not at all
2
3
4
5
Perfectly
8
Q

A new urine marker has been identified for early detection of acute kidney injury (AKI). Your ICU is interested in participating in a multisite clinical study comparing patients with serum markers consistent with AKI and results on the new urine test. Assume that the serum markers are 100% accurate in identifying AKI. The following results are obtained:

  • 80 patients with AKI confirmed by serum marker and with the urine marker
  • 20 patients with AKI confirmed by serum marker and without the urine marker • 40 patients without AKI by serum marker test and with the urine marker
  • 60 patients without AKI by serum marker test and without the urine marker

What is the sensitivity of the test in this study?

A. 40%

B. 60%

C. 67%

D. 75%

E. 80%

A

Answer - E - 80%

Sensitivity is the fraction of people with acute kidney injury(AKI) (serum marker positive) who test positive with the urine test. Specificity is the fraction of people without AKI (serum marker negative) who test negative with the urine test. Positive predictive value is the fraction of the people who test positive with the urine test who actually have AKI (serum marker positive). Negative predictive value is the fraction of people who test negative for the urine test who do not have AKI (serum marker negative).

How well did you know this?
1
Not at all
2
3
4
5
Perfectly
9
Q

Which one of the following choices indicates the need for permanent pacing in acquired atrioventricular (AV) block in a pediatric population?

A - Advanced second- or third-degree AV block with low cardiac output, congestive heart failure (CHF), or symptomatic bradycardia

B - Asymptomatic sinus node dysfunction during age-inappropriate bradycardia

C - Congenital third-degree AV block without ventricular dysfunction and with a normal QRS

D - Congenital third-degree AV block in child ≥1 year of age with an average heart rate of 65 bpm

E - Long-term treatment with digitalis for bradycardia-tachycardia syndrome

A

A - Advanced second- or third-degree AV block with low cardiac output, congestive heart failure (CHF), or symptomatic bradycardia

This question describes the most accurate indication for permanent pacing in AV block in a pediatric population. Option A is correct because according to the American Heart Association (AHA) guidelines, pediatric patients with second- or third-degree AV block with low cardiac output, CHF, or symptomatic bradycardia are ideal candidates for permanent pacing. Option B is incorrect because sinus node dysfunction during age-inappropriate bradycardia that is correlated with symptoms is an indication for permanent pacing. Option C is incorrect because congenital third-degree AV block is an indication for permanent pacing when there is ventricular dysfunction or a wide QRS escape rhythm. Option D is incorrect because permanent pacing is indicated in congenital third-degree AV block in children 1 year of age or older who have an average heart rate less than 50 bpm. Option E is incorrect because long-term treatment with digitalis for bradycardia-tachycardia syndrome is not an indication for permanent pacing; if long-term treatment is required with another agent, then permanent pacing may be indicated.

How well did you know this?
1
Not at all
2
3
4
5
Perfectly
9
Q

Which one of the following tests is most appropriate to confirm the initial diagnosis of Lyme disease?

A - Serological testing for antibodies

B - Biopsy of the rash

C - Appearance of erythema migrans is sufficient

D - Lumbar puncture

E - Blood studies to determine WBC

A

E - Blood studies to determine WBC

C - Appearance of erythema migrans is sufficient

This question describes the most appropriate diagnostic test to confirm infection with B. burgdorferi. Because the child is in the early stage of Lyme disease and erythema migrans is present, option A is incorrect. Serological testing for antibodies is often negative during early stages of Lyme disease and would not provide a differential diagnosis. Option B is incorrect as biopsy of the rash would not provide a differential diagnosis. Option C is correct because the presence of erythema migrans is highly specific to infection with B. burgdorferi and Lyme disease. Option D is incorrect because lumbar puncture is only indicated in some patients with late stage Lyme disease with cranioneuropathy. Option E is incorrect because the WBC may or may not be elevated in Lyme disease patients and therefore adds no diagnostic value.

How well did you know this?
1
Not at all
2
3
4
5
Perfectly
10
Q

A 4-year-old, 22-kg boy with influenza pneumonia develops worsening respiratory failure and requires intubation and mechanical ventilation for severe hypoxemia. It’s noted that he is being ventilated in volume-control SIMV mode with a rate of 20 (which he is overbreathing to 22), a tidal volume of 220 ml (with resultant plateau pressures of 32-35), an end expiratory pressure of 10 cm H2O, and an FiO2 of 1. His recent ABG indicates a pH of 7.41, PaCO2 of 32, and PaO2 of 58. Which one of the following changes in management would most likely reduce his mortality on the basis of existing animal and adult clinical data on ARDS?

A - Transition to high-frequency oscillating ventilation

B - Increase in positive end-expiratory pressure to 16 to 20 cm H2O to optimize recruitment

C - Sedation and paralysis (if needed) to reduce his overbreathing and create mild hypercapnia

D - Extracorporeal membrane oxygenation

E - Reduction of tidal volumes to reduce plateau pressures

A

E - Reduction of tidal volumes to reduce plateau pressures

In a large randomized trial of adult ARDS patients, low tidal volume ventilation (6 ml/kg) has been proven to reduce mortality compared to higher tidal volumes (12 ml/kg). Additional animal and clinical studies have correlated ventilator induced lung injury with excessive tidal volumes and plateau pressures. No one mode of ventilation is known to be superior for management to another (e.g., HFOV is not better than conventional). Although a recent adult study did reveal a mortality benefit from 24-hour neuromuscular blockade, this was in conjunction with low tidal volume ventilation and not due to the effect of respiratory rate or CO2. Transfer to an ECMO center in the UK provided superior outcomes in a recent adult ARDS trial in the UK, but this result was present in both patients who received ECMO and those who did not; thus, bringing into question whether it was in fact additional management strategies, such as low tidal volume ventilation, which underlay the benefit.

How well did you know this?
1
Not at all
2
3
4
5
Perfectly
10
Q

Concentration of which one of the following substances is higher in CSF, as compared to plasma?

a. Protein
b. Glucose
c. Potassium
d. Chloride
e. Cholesterol

A

D - Chloride

The following table compares the concentration of various substances in human CSF and plasma:

How well did you know this?
1
Not at all
2
3
4
5
Perfectly
10
Q

Identify endothelial cells, fixed macrophages, capillaries, and interalveolar pores (of Kohn) in a histological slide of an alveolus

A

Endothelial cells (E) – pulmonary capillaries

Fixed macrophages (M)

Alveolar macrophages – motile, in alveolar lumen

RBC and WBC’s in the capillaries (C)

Interalveolar pores (of Kohn)

  • Through the common alveolar septum;
  • Equilibrates the air pressure in adjacent alveolar spaces
How well did you know this?
1
Not at all
2
3
4
5
Perfectly
11
Q

A 2-year-old child has a history of recurrent attacks of acute onset edema of tongue/throat with stridor, without urticaria. This edema happens without any precipitating factor; swelling gradually worsens over 12-36 hours and then subsides over a period of days. He has a family history of similar episodes in his father and paternal uncle. Which one of the following options is a good screening test to diagnose his condition?

A - Serum C3 level

B - Serum C4 level

C - Serum C1 level

D - Absolute eosinophil count

E - Histamine challenge test

A

B - Serum C4 level

Hereditary angioedema (HAE) is a rare, autosomal dominant disorder of C1 inhibitor (CIINH) deficiency. CIINH deficiency presents in adolescent or early childhood with recurrent attacks of submucosal or subcutaneous edema. The edema and swelling gradually worsens for 12-36 hours and then subsides over a period of days. Edema of the upper respiratory tract, which can variably affect some or all of the tongue, pharynx and larynx, can be life threatening. Submucosal edema of the G.I tract can present as abdominal pain, nausea and vomiting. Urticaria is not a presenting feature of CIINH deficiency. Serum C4 level has been recommended as a screening test for CIINH deficiency, as serum C4 is invariably low in untreated HAE (C4

How well did you know this?
1
Not at all
2
3
4
5
Perfectly
11
Q

Which one of the following muscles is the most important for forced expiration?

a. Diaphragm
b. Sternocleidomastoid
c. External Intercostal
d. Internal Intercostal
e. Pectoralis major

A

d. Internal Intercostal

Respiratory muscles and movement of the diaphragm accounts for 75% of the change in intrathoracic volume during quiet inspiration. The other important inspiratory muscles are the external intercoastal muscles. The scalene and sternocleidomastoid muscles in the neck are accessory inspiratory muscles that help to elevate the thoracic cage during deep, labored respiration. The main expiratory muscles are the internal intercoastal. The internal intercostals have this action because they pass obliquely downwards and posteriorly from rib to rib and therefore pull the ribcage downwards when they contract. Contractions of the muscles of the anterior abdominal wall also aid expiration by pulling the ribcage downwards and inwards and by increasing the intraabdominal pressure, which pushes the diaphragm upward.

How well did you know this?
1
Not at all
2
3
4
5
Perfectly
11
Q

Which one of the following factors is reduced in neonates which affects drug absorption?

A - Intestinal motility

B - Proportion of water in body weight

C - Plasma half-lives for many drugs

D - Circulating free fatty acids

E - Circulating bilirubin

A

A - Intestinal motility

This question describes one factor that is reduced in neonates which affects drug absorption. Option A is correct because intestinal motility is one factor that is reduced in neonates which affects drug absorption and should be taken into consideration when determining proper dosage for a drug. Option B is incorrect because neonates have the greatest proportion of water to body weight as compared to other ages. Option C is incorrect because plasma half-lives for many drugs are increased in neonates. Options D and E are incorrect because there are greater levels of circulating free fatty acids and bilirubin in neonates as compared to adults.

How well did you know this?
1
Not at all
2
3
4
5
Perfectly
12
Q

Which of the following would result in an acute decrease in SVO2 in cardiogenic shock?

A. Decreased stroke volume

B. Inotropic therapy

C. Transfusion of PRBCs

D. Vasodilator therapy

A

Answer: A - Decreased stroke volume

SVO2 serves as a surrogate marker for cardiac output. Factors that decrease cardiac output will result in increased arteriovenous DO2 difference and decreased SVO2. These would be factors such as decreased stroke volume, anemia, and increased afterload. Vasodilator therapy and therefore decreased afterload would lead to improved cardiac output and a corresponding increase in SVO2.

SvO2 is assumed to reflect the balance between arterial oxygen delivery (DO2) and oxygen consumption (VO2) provided arterial blood oxygen saturation (SaO2) is normal. The modified

Fick equation states:
SvO2 = SaO2 – [VO2/(Cardiac Output × Hemoglobin × 1.34)]

Care must be taken when interpreting SvO2 in shock states. In any shock state, oxygen demand exceeds VO2 by definition such that SvO2 cannot reflect the balance between DO2 and oxygen demand. Second, as VO2/DO2 dependency is a characteristic pattern of shock states, any increase in DO2 during resuscitation will be associated with a simultaneous increase in VO2 and hence with no or only a small increase in SvO2 until a critical DO2 is reached. SvO2 should not change much in response to an increase in cardiac output in cases of hyperdynamic shock states.

Interest in SvO2 monitoring, SvO2 surrogate, and central venous oxygen saturation, in septic shock is recommended. SvO2 has been recommended as a major hemodynamic target of early resuscitation in septic shock.

How well did you know this?
1
Not at all
2
3
4
5
Perfectly
13
Q

A 6-year-old child with renal failure and DIC is on continuous venovenous hemofiltration (CVVH). Due to risk of bleeding with heparin, a commercial citrate infusion is being used. Which one of the following is an SE of citrate anticoagulation?

A - Hypercalcemia

B - Metabolic acidosis

C - Metabolic alkalosis

D - Hypokalemia

E - Hyponatremia

A

C - Metabolic alkalosis

Citrate anticoagulation is based on the concept of chelation of the calcium in order to avoid clotting of blood in the chronic renal replacement therapy (CRRT) circuit. Complications of citrate anticoagulation include hypocalcemia, metabolic alkalosis, and citrate toxicity. Metabolic alkalosis develops due to hepatic conversion of 1 mol of citrate to 3 mol of bicarbonate.

How well did you know this?
1
Not at all
2
3
4
5
Perfectly
13
Q

A 5-year-old boy presents to the emergency department with altered mental status after a motor vehicle collision. He is on a backboard and a cervical collar is in place. HR is 62/min, and he is being bag-ventilated at a rate of 15/min with an oxygen saturation of 98% on 100% oxygen. He has good chest rise with bagging. His BP is 152/89 mm Hg and he has unequal pupils. The most appropriate next step in his management is to:

A. Have ketamine, atropine, and rocuronium drawn up for intubation

B. Initiate intentional hyperventilation at 25/min with bag-valve-mask while preparing for intubation

C. Perform a tertiary survey

D. Place a roll under the shoulders and hyperextend the neck for better airway alignment and visualization

A

Answer: B

The child is showing signs of increased intracranial pressure and should have immediate hyperventilation while preparing for intubation. Ketamine is relatively contraindicated in patients with elevated intracranial pressure. Additionally, while one may remove the cervical collar to facilitate intubation, cervical spine immobilization should be maintained at all times. Tertiary survey is not performed until a patient has been stabilized.

How well did you know this?
1
Not at all
2
3
4
5
Perfectly
14
Q

A 7-year-old boy with pneumonia is intubated and is on mechanical ventilator. His current settings are: mode: pressure control, PIP above PEEP 15, PEEP5, FiO2 50%, rate 25. On these settings, his exhaled tidal volume is 200 ml (6.5 ml/kg). His arterial blood gas on these settings are pH 7.13/PaCO2 70, PaO2 80. Patient is sedated and paralyzed. In order to decrease the PaCO2 to 50 mmHg, PIP needs to be increased to which one of the following levels?

A - 18

B - 21

C - 24

D - 27

E - 30

A

B - 21

Desired Vt = known PaCO2 × known Vt / desired PaCO2. Desired Vt = 70 × 200 / 50 = 280 ml. A pressure change (ΔP) of 15 results in a Vt of 200 ml, A ΔP than will generate tidal volume of 280 ml, can be estimated by first calculating static compliance. Static compliance Cs = Vt / ΔP. Cs = 200/15 = 13 ml/cm H2O. If compliance remains the same: Desired ΔP = desired Vt / Cs. Desired ΔP = 280/13. Desired ΔP = 21.

How well did you know this?
1
Not at all
2
3
4
5
Perfectly
14
Q

Which one of the following options is a characteristic of intraparenchymal catheters as compared to intraventricular systems?

A - Less invasive

B - More accurate

C - The property of “zero drift”

D - Less expensive

E - Higher risk of infections

A

C - The property of “zero drift”

There are 2 types of ICP monitors currently in use: intraparenchymal catheters and the intraventricular systems. A ventricular catheter connected to an external monitoring devise, is the most accurate and low-cost method for ICP monitoring. This method has proven to be reliable and permits periodic re-zeroing; it also allows the benefit of therapeutic CSF drainage. However, currently the most common location for ICP monitoring is the cerebral parenchyma. Contemporary intraparenchymal transducers may be classified as solid-state based on silicon chip with pressure sensitive resistors, or of fiberoptic design. Although both systems are very accurate at the time of placement, they have been reported to zero-drift over time, which can result in an error after 4-5 days.

How well did you know this?
1
Not at all
2
3
4
5
Perfectly
14
Q

A 2-week-old male child, with history of tachypnea and poor feeding, was admitted to PICU with a diagnosis of bronchiolitis. Patient is on mechanical ventilation for respiratory distress with positive end- expiratory pressure (PEEP) of 8; FiO2 of 0.7; and pulse oximeter oxygen saturation is 88%-95%. On examination, his heart rate is 160/minute, blood pressure is 65/35 mm kg, and capillary refill is 2-3 seconds. He has fixed split second sound; no cardiac murmur. His liver is enlarged 4 cm below the costal margin. His urine output is 2 ml/kg/hr. His arterial blood gas shows pH of 7.34, PaCO2 of 40, PaO2 of 56, base excess of negative 2. Rapid test for respiratory syncytial virus is negative. His chest X-ray is shown below. Which one of the following courses of action is the most appropriate for this child?

a. Fluid bolus followed by prostaglandin (PGE1) infusion to open ductus arteriosus
b. Start iNO at 20 parts per million and increase PEEP to 10
c. Echocardiography followed by surgery as soon as possible
d. Start continuous milrinone infusion, followed by epinephrine infusion
e. Place child on venoarterial extracorporeal membrane oxygenation (ECMO)

A

c. Echocardiography followed by surgery as soon as possible

This is a child with supracardiac total anomalous venous return (TAPVR). His chest X-ray has classic snowman appearance. This patient has pulmonary venous congestion with right heart failure. His hemodynamic status seems acceptable with no significant acidosis. Option A is incorrect because prostaglandin infusion will increase pulmonary blood flow and should be avoided. Option B is also incorrect because iNO can increase pulmonary blood flow and will worsen the situation. This patient may need iNO after surgery but not indicated during preoperative period. Option C is correct response because echocardiography is usually enough to diagnose total anomalous venous return TAPVR in this patient. TAPVR patients require surgical correction as soon as possible. Option D is incorrect because milrinone can increase pulmonary blood flow and epinephrine is not needed due to acceptable hemodynamic status. Option E is incorrect because this is a relatively stable patient and there is no need to place this patient on ECMO before surgery. Some of the TAPVR patients may require ECMO after surgery, more so if they have associated hypoplastic left ventricle.

How well did you know this?
1
Not at all
2
3
4
5
Perfectly
15
Q

A 6-year-old girl who weighs 15 kg (33 lbs) and is 100 cm (39 in) in length is admitted to the pediatric ICU with pneumococcal sepsis. Her serum creatinine level on admission is 0.4 mg/dL, and it rises to 1.5 mg/dL 2 days later. Which of the following best characterizes estimated glomerular filtration rate at this time?

A. >50 mL/min/1.73 m2; unchanged from normal baseline

B. Between 10 and 50 mL/min/1.73 m2; 75% of baseline

C. Between 5 and 10 mL/min/1.73 m2; 50% of baseline

D.

A

Answer:D -

Serum creatinine and creatinine-based equations can only be used to assess glomerular filtration rate (GFR) when a steady state exists. In the setting of acute kidney injury, an abrupt decline in GFR is associated with a gradual rise in serum creatinine until a new steady state is achieved. Use of the serum creatinine level to estimate the GFR in this period will result in an overestimation of the renal function. A doubling of serum creatinine corresponds to an approximate 50% decrease in remaining renal function. Therefore, her nearly 4-fold increase in serum creatinine suggests that her GFR is less than 30% of normal. Serum creatinine levels should be monitored once or twice a day to better estimate her GFR.

How well did you know this?
1
Not at all
2
3
4
5
Perfectly
16
Q

What is cardiac output?

A

HR x SV

How well did you know this?
1
Not at all
2
3
4
5
Perfectly
16
Q

Osmotherapy is most effective in which one of the following forms of cerebral edema?

a. Cytotoxic
b. Vasogenic
c. Interstitial
d. Equally not effective in all forms
e. Equally effective in all forms

A

a. Cytotoxic

Cerebral edema is conventionally classified into vasogenic, cytotoxic, and interstitial edema based on its underlying pathogenesis. Vasogenic cerebral edema occurs due to disruption of blood brain barrier. Increased capillary permeability due to blood brain barriers disruption allows intravascular fluid and solutes to enter brain interstitial fluid, causing its expansion. Cytotoxic cerebral edema occurs due to accumulation of higher amount of intracellular salute compared to extracellular fluid, which leads to movement of water into the cells to maintain osmotic equilibrium. Diseases which impair cellular metabolism (hypoglycemia, rye syndrome) or sustained hyperosmolar state (hypernatremia, hyperglycemia) are associated with cytotoxic edema. Interstitial cerebral edema occurs when there is increased hydrostatic pressure gradient between the ventricle system and the brain interstitium, resulting in transependymal movement of CSF. Interstitial endemia is seen in obstructive and nonobstructive hydrocephalus. Consideration of pathophysiologic mechanism in cerebral edema has important therapeutic significance. Osmotic therapy is most effective in cytotoxic cerebral edema, whereas relief of CSS obstruction is necessary to treat interstitial cerebral edema. It is, however, important to realize that all 3 forms of cerebral edema can occur concurrently in the same patient.

How well did you know this?
1
Not at all
2
3
4
5
Perfectly
16
Q

Which of the following letters in the figure refers to the closing capacity?

A. A

B. B

C. C

D. D

E. E

A

Answer: C

The closing capacity is defined as the closing volume plus the residual volume. In the figure, the residual volume is represented by the letter B. The closing volume is not depicted in the figure, but represents some portion of the expiratory reserve volume. The closing capacity exceeds functional residual capacity (FRC) (letter E) when some lung segments are closed during tidal breathing. This leads to hypoxia and hypercapnia. If the closing capacity exceeds both the FRC and tidal volume (letter A), lung segments will remain closed during both inspiration and expiration. This is important clinically and represents areas of atelectasis. Opportunities to raise FRC above closing capacity by administering positive pressure in the form of positive end- expiratory pressure or continuous positive airway pressure are useful. Children younger than 6 years have a closing capacity greater than FRC when supine.

How well did you know this?
1
Not at all
2
3
4
5
Perfectly
17
Q

A 5-year-old who presents in status asthmaticus required intubation approximately 30 minutes earlier due to obtundation and severe hypercarbia (PaCO2 = 78). Rocuronium and ketamine were used at the time of intubation, and the child is now on fentanyl 1 mcg/kg/hour infusion for sedation. When called to evaluate him for hypotension, it’s noted that he’s quite agitated, with HR of 142, RR of 54 (vent set rate 18), and BP of 62/45. His oxygen saturations are 98% with FiO2 of 0.4, and he has diffuse wheezing bilaterally. Which one of the following next steps will best address the underlying cause of his hypotension?

A - Order a stat chest radiograph

B - Needle decompression of the chest

C - Disconnect the patient from the mechanical ventilator followed by sedation and/or paralysis

D - Administer a fluid bolus and begin an epinephrine infusion

E - Administer nebulized albuterol

A

C - Disconnect the patient from the mechanical ventilator followed by sedation and/or paralysis

This is a classic situation where auto-PEEP, due to tachypnea in the setting of airflow obstruction (bronchospasm), results in shock by impairing venous return. Given the severity of the shock, there is a need for rapid action to permit exhalation, which in this case would come from disconnection from the vent circuit followed by sedation. In addition, paralysis (if needed) would reduce the respiratory rate and permit adequate exhalation. While a tension pneumothorax may result in hypotension, it is unlikely to be present in this scenario due to the presence of bilateral breath sounds. A fluid bolus could help the hypotension but does not address the underlying etiology. Beta agonist therapy is necessary to combat the bronchospasm, but will not act sufficiently quickly to reverse the patient’s shock in this case.

How well did you know this?
1
Not at all
2
3
4
5
Perfectly
17
Q

Regarding emergency airway management in children, which one of the following statements is the most accurate?

a. Succinylcholine can be safely used in children with spinal cord injuries
b. Laryngeal mask airway can be safely used in patient with full stomach
c. Blind nasotracheal intubation is recommended for conscious children with multiple traumas
d. Orotracheal intubation with sedation, analgesia, and nondepolarizing paralytics is preferred method
e. Primary tracheotomy without any attempts of intubation is safe in children with cervical spine injuries

A

d. Orotracheal intubation with sedation, analgesia, and nondepolarizing paralytics is preferred method

Safe management of airway is of paramount importance in acute emergencies. Option A is incorrect. Patients with spinal cord injuries are at risk for extreme hyperkalemia following administration of succinylcholine. Option B is incorrect. Laryngeal mask airway is not the ideal technique to use in a patient with full stomach because its design does not prevent aspiration of gastric contents. Option C is incorrect because current advanced trauma support guidelines no longer recommend blind nasotracheal intubation. Option D is the correct response. Orotracheal intubation with sedation, analgesia, and nondepolarizing neuromuscular blocking agents is safer in children. In suspected cervical spine injury, immobilization of head and neck in neutral position by an assistant is necessary. Option E is incorrect. Data does not support either the necessity or safety of routinely using primary tracheotomy before attempting intubation.

How well did you know this?
1
Not at all
2
3
4
5
Perfectly
17
Q

A 3-year-old child with a single left ventricle underwent a fenestrated Fontan procedure. He returns from the cardiac catheterization lab with pulmonary artery pressure of 20 mm Hg, left atrial pressure of 5 mm Hg, and systemic oxygen saturation of 99%. There is no pressure gradient between a simultaneous pulmonary artery wedge pressure and an end-diastolic pressure of the ventricle of 4 mm Hg. Which of the following assessments of the patient’s physiology is most accurate?

A. The fenestration is patent

B. The transpulmonary gradient is normal

C. The pulmonary vascular resistance is normal

D. There is no evidence of atrioventricular valve or pulmonary venous stenosis

A

Answer: D - There is no evidence of AVV or pulmonary venous stenosis

The catheterization data show an elevated transpulmonary gradient (pulmonary artery pressure minus left atrial pressure) of 15 mm Hg. Therefore, the patient is likely to have elevated pulmonary vascular resistance. The fenestration probably is not patent because the oxygen saturation is 99%, despite a pulmonary artery pressure of 20 mm Hg. With a pulmonary artery pressure of 20 mm Hg, some shunting of venous blood should be expected through a patent fenestration. Hence, the systemic oxygen saturation would be less than 99%. Simultaneous pulmonary artery wedge and end-diastolic pressure shows no gradient, evidence that there is no stenosis of the atrioventricular valve or pulmonary venous anatomy.

How well did you know this?
1
Not at all
2
3
4
5
Perfectly
18
Q

According to pediatric guidelines for severe traumatic brain injury (TBI), the target cerebral perfusion pressure (CPP) for a 2-year-old, 15-kg (33-lb) child should be maintained above which of the following minimum levels?

A. 30 mm Hg

B. 40 mm Hg

C. 50 mm Hg

D. 60 mm Hg

A

Answer: B - 40 mmHg

Cerebral perfusion pressure (CPP) is the difference between mean arterial pressure and mean intracranial pressure (ICP). In children, a CPP of less than 40 mm Hg is associated with increased mortality. Although the optimal values are unknown, the pediatric guideline for treating severe traumatic brain injury is maintaining a CPP of more than 40 mm Hg in an adolescent. A modification to this recommendation is to titrate the CPP threshold according to age, with age-specific thresholds for children less than a year of age and adolescents.

How well did you know this?
1
Not at all
2
3
4
5
Perfectly
19
Q

A 3-year-old child involved in a motor vehicle collision (MVC) sustains an injury at T7. He is initially in no respiratory distress. But on hospital day 2 develops respiratory distress. Which one of the following muscle groups is important for active expiration in this child?

A - Deep intercostals

B - Outer intercostals

C - Diaphragm

D - Trapezius

E - Sternocleidomastoid

A

A - Deep intercostals

Internal or deep intercostals are the primary muscle of expiration. They are located between ribs deep and oriented at a right angle to external intercostals, continuous with internal obliques. The internal intercostals function to depress the ribs during expiration. The accessory muscles of expiration are the rectus abdominis, external obliques, internal obliques, transversus thoracis, serratus posterior inferior, quadratus lumborum. The diaphragm and outer or external intercostals are the major muscles in inspiration. The trapezius and sternocleidomastoid muscles are accessory muscles of inspiration.

How well did you know this?
1
Not at all
2
3
4
5
Perfectly
21
Q

A 1-day-old, full-term neonate has cyanosis, tachypnea, and an oxygen saturation of 82% in room air by pulse oximetry. Chest radiography shows streaky bilateral interstitial opacities. Cyanotic congenital heart disease is suspected. A hyperoxia test is performed with PaO2 levels obtained on 100% oxygen. Which of the following results is suggestive of cyanotic congenital cardiac disease?

A. PaO2 30 mm Hg before, 250 mm Hg after

B. PaO2 30 mm Hg before, 80 mm Hg after

C. PaO2 150 mm Hg before, 150 mm Hg after

D. PaO2 150 mm Hg before, 250 mm Hg after

A

Answer: B - PaO2 30mmHg before and 80mmHg after

An arterial PaO2 less than 100 mm Hg on 100% oxygen in the absence of obvious lung disease (“failed” hyperoxia test) is most likely due to intracardiac right-to-left shunting and is virtually diagnostic of cyanotic congenital heart disease. The increased PaO2 in the remaining answers is most likely related to variable responses to parenchymal lung disease.

How well did you know this?
1
Not at all
2
3
4
5
Perfectly
22
Q

A 3-month-old boy is being evaluated for the onset of hypotonia and feeding difficulties. His mother also reports he has a history of constipation. On examination, he is afebrile with normal growth parameters. Neurologic examination is significant for decreased muscle tone and a decreased gag reflex. His pupils are 4 mm bilaterally and sluggish to light. Deep tendon reflexes are 2+ bilaterally in upper and lower extremities. The remainder of his physical examination is unremarkable. A lumbar puncture reveals normal cell counts and chemistry results. Which of the following tests would be most helpful in establishing the diagnosis?

A. Brain MRI

B. Electroencephalography (EEG)

C. Stool toxin assay

D. Sweat chloride level

E. Thyroid-stimulating hormone test

A

Answer: C - Stool toxin assay

This patient has infantile botulism. Botulinum toxin is one of the most potent poisons known. Clostridium botulinum is a spore-forming, anaerobic, gram-positive bacillus with 8 known toxins. Botulinum toxin acts to prevent acetylcholine release, which decreases cholinergic activity in the peripheral nervous system. The central nervous system remains unaffected. Infantile botulism was first described in 1976. Typically, this disease affects infants 1-3 months of age. A common presentation involves constipation, feeding problems, and floppiness. Patients can also present with ophthalmoplegia and respiratory failure. Fever is usually absent. The main clinical dilemma is differentiating infantile botulism from Guillain-Barré syndrome (GBS). Deep tendon reflexes and normal cerebrospinal fluid (CSF) findings are found in infantile botulism patients, whereas elevated CSF protein levels and absent deep tendon reflexes are common in GBS. Management is primarily supportive. Monitoring, typically in an ICU setting, is required for the first week of symptoms when the risk of respiratory failure is highest. Use of antitoxin is not recommended for infantile botulism because of the belief that there are low circulating levels of toxin in this disease. Education about washing foods and objects that the infant places in his or her mouth is an important factor in prevention.

How well did you know this?
1
Not at all
2
3
4
5
Perfectly
23
Q

A 2-year-old child has a history of recurrent attacks of acute onset edema of tongue/throat with stridor, without urticaria. This edema happens without any precipitating factor; swelling gradually worsens over 12-36 hours and then subsides over a period of days. He has a family history of similar episodes in his father and paternal uncle. Which one of the following options is a good screening test to diagnose his condition?

a. Serum C3 level
b. Serum C4 level
c. Serum C1 level
d. Absolute eosinophil count
e. Histamine challenge test

A

b. Serum C4 level

Hereditary angioedema (HAE) is a rare, autosomal dominant disorder of C1 inhibitor (CIINH) deficiency. CIINH deficiency presents in adolescent or early childhood with recurrent attacks of submucosal or subcutaneous edema. The edema and swelling gradually worsens for 12-36 hours and then subsides over a period of days. Edema of the upper respiratory tract, which can variably affect some or all of the tongue, pharynx and larynx, can be life threatening. Submucosal edema of the G.I tract can present as abdominal pain, nausea and vomiting. Urticaria is not a presenting feature of CIINH deficiency. Serum C4 level has been recommended as a screening test for CIINH deficiency, as serum C4 is invariably low in untreated HAE (C4

How well did you know this?
1
Not at all
2
3
4
5
Perfectly
23
Q

A 3-week-old neonate with transposed great arteries and intact ventricular septal defect returns to the ICU following an arterial switch operation. HR is 185/min, BP is decreased to 45/30 mm Hg, and the left atrial pressure is elevated at 17 mm Hg. Right atrial pressure and the ECG tracing are normal. Which of the following is the most likely cause of these findings?

A. Left ventricular pump failure

B. Mitral valve insufficiency

C. Left ventricular outflow tract obstruction

D. Pulmonary hypertension

E. Pericardial tamponade

A

Answer: A - Left ventricular pump failure

Left ventricular pump failure is the most likely cause of low cardiac output syndrome with increased left atrial pressure in transposition of the great arteries with intact ventricular septal defect (TGA-IVS). This is because of inappropriate adaptation of the left ventricle to cope with higher resistance of systemic circulation compared to pulmonary circulation after the arterial switch operation. Mitral valve insufficiency and left ventricular outflow tract obstruction also can occur but not specifically in the case of postoperative TGA-IVS. Pulmonary hypertension rarely happens in TGA-IVS. Pericardial tamponade can occur and is common postoperative to all types of cardiac surgery.

How well did you know this?
1
Not at all
2
3
4
5
Perfectly
24
Q

The dicrotic notch on the aortic pressure curve is caused by which one of the following occurrences?

A - Closure of the mitral valve

B - Closure of the tricuspid valve

C - Closure of the aortic valve

D - Closure of the pulmonary valve

E - Rapid filling of the LV

A

C - Closure of the aortic valve

The blood forced into the aorta during systole not only move the blood in the vessels forward but also sets up a pressure wave that travels along the arteries. The pressure waves expand the arterial wall as it travels, and the expansion is palpable as the pulse. The dicrotic notch—a small oscillation on the falling phase of the pulse wave—is caused by vibrations set up when the aortic valve is snapped shut.

How well did you know this?
1
Not at all
2
3
4
5
Perfectly
24
Q

Ventricular afterload is best approximated by which of the following values?

A. Ventricular end-diastolic pressure

B. Ventricular end-diastolic volume

C. Systolic blood pressure

D. Mean arterial pressure

E. Ventricular wall stress

A

Answer E - Ventricular wall stress

The ventricular afterload is best approximated by ventricular wall stress, or the degree of stretching of the ventricular muscle. It is approximated by the formula:

Wall Stress = (P × r)/2t,

pressure (P) times the radius (r) divided by twice the wall thickness (t).

How well did you know this?
1
Not at all
2
3
4
5
Perfectly
24
Q

A 2-day-old neonate presents with lethargy, vomiting, and seizures. An inborn error of metabolism is suspected. The blood glucose level is normal, there is no acidosis, and the ammonia level is 950 μmol/L; urea cycle defect is diagnosed. Which of the following enzyme abnormalities is most likely involved in this patient?

A. Argininosuccinic acid lyase (ASL) deficiency

B. Argininosuccinic acid synthetase (ASS) deficiency

C. Branched-chain alpha-ketoacid dehydrogenase complex

D. Carbamyl phosphate synthetase 1 (CPS1) deficiency

E. Ornithine transcarbamylase (OTC) deficiency

A

Answer: E - OTC deficiency

Ammonia is normally produced from catabolism of amino acids and at high levels is a neurotoxin. In humans, ammonia is detoxified to urea in the liver by the urea cycle. Any increase in blood ammonia levels reflects an impairment of the urea cycle or extensive hepatic damage. Signs and symptoms of hyperammonemia include anorexia, irritability, lethargy, vomiting, somnolence, disorientation, and subsequently cerebral edema, coma, and death. The urea cycle defects are listed below. ASL, ASS, CPS1, and OTC are all urea cycle defects. However, OTC deficiency is by far the most commonly reported defect in the neonatal period and is often evident in the first few days of life.

An infant with OTC deficiency may be lacking in energy (lethargic) or unwilling to eat, and have poorly controlled breathing rate or body temperature. Some babies with this disorder may experience seizures or unusual body movements, or go into a coma. Complications from OTC deficiency may include developmental delay and intellectual disability. Progressive liver damage, skin lesions, and brittle hair may also be seen in any of the urea cycle defects. Children may have complete or partial expression of the gene and present later in life. Deficiency of the branched-chain alpha-ketoacid dehydrogenase complex causes maple syrup urine disease, one of the organic acidemias.

How well did you know this?
1
Not at all
2
3
4
5
Perfectly
25
Q

Which one of the following options describes why a higher dose of water-soluble agents are required in younger children as opposed to older children?

A - Greater rates of plasma protein binding

B - Lower rates of plasma protein binding

C - Erratic drug absorption

D - Greater proportion of body weight is water

E - Lower proportion of body weight is water

A

D - Greater proportion of body weight is water

This question describes why a higher dose of water-soluble agents are required in younger children as opposed to older children. Although plasma protein binding is important in drug distribution, options A and B are incorrect because they do not result in the need for a greater dose of water-soluble drugs in younger children versus older children. Option C is incorrect because water-soluble drugs are not administered in higher doses to younger children due to erratic absorption. Option D is correct while option E is incorrect because as a child ages, the percentage of their body that is water decreases, resulting in differences in drug volume distribution.

How well did you know this?
1
Not at all
2
3
4
5
Perfectly
26
Q

Which one of the following options is the most important action of epinephrine in terms of usage in cardiac arrest?

a. β1 causing increase in contractility
b. β2 causing vasodilation
c. α1 causing systemic vasoconstriction
d. All actions are equally important
e. PDE III inhibition inhibiting release of cAMP

A

c. α1 causing systemic vasoconstriction

The alpha-adrenergic mediated vasoconstriction due to epinephrine, increases aortic diastolic pressure, and thus coronary perfusion pressure—a critical determinant of successful resuscitation from cardiac arrest. At low doses, the beta-adrenergic effects may predominate, leading to decreased systemic vascular resistance; in the doses used during cardiac arrest, the vasoconstrictive alpha effects predominate.

How well did you know this?
1
Not at all
2
3
4
5
Perfectly
27
Q

A 5-year-old child is status-post a cardiac transplant for dilated cardiomyopathy. Which one of the following statements is true regarding his heart rate and use of inotropic medications in postoperative period?

A - Transplanted hearts have a lower basal rate due to denervation

B - Dopamine has an accentuated inotropic effect on the transplanted heart

C - Atropine is ineffective for the treatment of bradycardia in immediate post-cardiac transplant

D - Stress and exercise induced heart rate increase remains preserved

E - Permanent pacing wires are very frequently required in transplanted hearts

A

C - Atropine is ineffective for the treatment of bradycardia in immediate post-cardiac transplant

Loss of cardiac innervation in heart transplant patients results in significant changes in patient’s ability to respond to inotropic medications. The donor sinoatrial (SA) node—which controls the rate of the transplanted heart—does not receive vagal stimulation and thus fires at a higher basal rate. The stress and exercise induced increase in heart rate are blunted relative to normal. There is decreased cardiac response to dopamine; however, epinephrine and NE tend to have an accentuated inotropic effect on a denervated heart. The vagolytic effects of atropine are ineffective for the treatment of bradycardia. Temporary pacing wires are frequently used in the early postoperative period to maintain an adequate heart rate and cardiac output. Permanent pacing is rarely required.

How well did you know this?
1
Not at all
2
3
4
5
Perfectly
27
Q

You are consulting on a previously healthy, 6-month-old girl in the emergency department (ED). She has fever to 39.3 C (102.8 F), irritability, a bulging fontanelle, and what her parents describe as seizure activity at home. The ED staff have obtained a complete blood count, blood and urine culture, and a coagulation profile with the following results: WBCs, 20,500/μL; hemoglobin, 10 g/dL; hematocrit, 30%; platelets, 7,000/μL; prothrombin time, 20 seconds; international normalized ratio, 2.3; partial thromboplastin time, 60 seconds; and fibrinogen, 300 mg/dL. She has not yet received any antibiotics. You suspect meningitis and would like to perform a lumbar puncture to confirm the diagnosis. Hemodynamics are appropriate and you feel she would tolerate the procedure at this time. What would be the safest and most helpful sequence of events?

A. Administer vancomycin and cefotaxime, transfuse cryoprecipitate and platelets, perform the lumbar puncture once coagulopathy is improved, and adjust antibiotics if needed once cerebrospinal fluid (CSF) results are available.

B. Administer vancomycin and cefotaxime, transfuse fresh frozen plasma and platelets, perform the lumbar puncture once coagulopathy is improved, and adjust antibiotics if needed once CSF results are available.

C. Perform lumbar puncture while platelets are infusing, administer vancomycin and cefotaxime once lumbar puncture is complete, and adjust antibiotics if needed once CSF results are available.

D. Perform the lumbar puncture while factor VII is infusing, and administer antibiotics based upon the results of the Gram stain.

E. Transfuse fresh frozen plasma and platelets, perform the lumbar puncture, and administer antibiotics based upon the results of the Gram stain.

A

Answer: B

In a patient with suspected meningitis and especially sepsis, antibiotics should not be delayed while waiting to perform lumbar puncture. In this particular patient, it will be at least an hour’s delay for blood products to become available and probably at least another hour to infuse them. This time delay in receiving antibiotics is unacceptable. While giving antibiotics prior to obtaining lumbar puncture may diminish your chances of obtaining antibiotic sensitivities to the causative organism, the cerebrospinal fluid should still show biochemical changes of meningitis and can still have bacterial antigen studies performed, and the blood culture may reveal positive results.

Patients with coagulopathy are at risk for epidermal hematomas with lumbar puncture. This patient’s coagulopathy is most likely caused from disseminated intravascular coagulopathy as evidenced by a low platelet count, prolonged prothrombin time, and partial thromboplastin time. This is most quickly corrected by transfusion of platelets and fresh frozen plasma. Cryoprecipitate would not be as helpful, as the patient’s fibrinogen level is more than adequate. Cryoprecipitate is typically used to replace fibrinogen and factor VIII, while fresh frozen plasma contains multiple clotting factors and is more helpful in correcting a decrease in multiple clotting factors as is seen in disseminated intravascular coagulation. Factor VII could be used to correct the coagulopathy. However, due to the expense and high risk of complications from thromboses, it would not be the best choice. It is typically reserved for patients with life- threatening bleeding not controlled with standard blood product replacement.

How well did you know this?
1
Not at all
2
3
4
5
Perfectly
29
Q
  1. After motor vehicle accident, a 5-year-old boy is admitted to PICU for observation. On examination, he has bruises on right side of chest and diminished air entry on the right side. He had hypotension on admission which responded to fluid boluses. His oxygen saturation on room air is 94%. Chest X-ray shows good-sized pneumothorax on right side. Which one of the following treatment plans is the safest for this patient?
    a. Needle decompression followed by chest tube insertion on the right side
    b. Needle decompression and observation without chest tube
    c. Place child on 100% oxygen and do no other intervention
    d. Continue to monitor without any intervention
    e. Fibro-optic bronchoscopy at bed side
A

a. Needle decompression followed by chest tube insertion on the right side

This is a patient with tension pneumothorax. Recommended treatment for child with tension pneumothorax is needle decompression followed by chest tube insertion. Option A is correct because this is the recommended treatment. Needle decompression will convert tension pneumothorax to simple pneumothorax and chest tube is needed regardless of the response to needle decompression. Option B is incorrect. Needle decompression alone is not sufficient because pneumothorax can recur and can lead to further complications. Options C and D are incorrect because this approach can place child at risk for further deterioration. Option E is incorrect because bronchoscopy is not indicated.

How well did you know this?
1
Not at all
2
3
4
5
Perfectly
30
Q

Pulmonary hypertensive crisis after surgery in the neonatal period mostly occurs in patients with which of the following cardiac lesions?

A. Truncus arteriosus

B. Moderate ventricular septal defect

C. Patent ductus arteriosus

D. Transposition of the great arteries with intact ventricular septal defect

A

Answer: A - Truncus arteriosus

Truncus arteriosus is the type of congenital cardiac lesion most likely to have a severe course of pulmonary hypertension after surgery because most of the blood unrestrictedly flows to the lung. Intermediate ventricular septal defect and patent ductus arteriosus(PDA) that also have high flow to the lung due to left-to-right shunt are unlikely to have pulmonary hypertensive crisis after surgery, except for some large PDAs and complete atrioventricular septal defects that have undergone surgery in older age. Transposition of the great arteries with intact ventricular septal defect is hardly seen complicated with pulmonary hypertension.

Patients with left-to-right shunt repaired at an older age are at increased risk for the complication of postoperative pulmonary hypertension, presumably due to having more time to develop significant vascular remodeling before repair.

How well did you know this?
1
Not at all
2
3
4
5
Perfectly
31
Q

The left ventricle pressure-volume curve at times A and B is shown in the figure. The increase in stroke volume shown at time B is due to which of the following factors?

A. Increased contractility

B. Increased preload

C. Decreased afterload

D. Improved lusitropy

A

Answer: D - Improved lusitropy

The left ventricle pressure-volume curves show an increase in stroke volume at time B compared with time A. The curves labeled A and B are representative of the end-diastolic pressure-volume relationship. Curve B shows improved diastolic function with improved end- diastolic volume for a similar end-diastolic pressure compared to Curve A, which results in an increase in stroke volume.

How well did you know this?
1
Not at all
2
3
4
5
Perfectly
32
Q

The cause of the syndrome known as thrombotic thrombocytopenic purpura is best described by which one of the following definitions?

A - Factor H deficiency, leading to constitutive activation of the alternate pathway of complement

B - Production of antibodies against platelet GP IIb/IIIa receptors, leading to antibody/platelet complexes

C - Low circulating C3 levels

D - Elevated von Willebrand cleaving protease enzyme activity level

E - Decreased von Willebrand cleaving protease enzyme activity level

A

E - Decreased von Willebrand cleaving protease enzyme activity level

Factor H deficiency is noted to be the cause of non-diarrhea-associated hemolytic uremic syndrome. Antibodies against GP IIb/IIIa receptors are identified as the etiology of immune thrombocytopenic purpura. Low-circulating C3 levels are seen in non-diarrhea-associated HUS, but this is not the underlying etiology for the disorder. Elevated levels of von Willebrand cleaving protease (ADAMTS-13) enzyme activity levels are seen in diarrhea-associated HUS. Decreased ADAMTS-13 levels are seen in TTP.

How well did you know this?
1
Not at all
2
3
4
5
Perfectly
33
Q

Aortic stenosis

A. Curve A

B. Curve B

C. Curve C

D. Curve D

E. Curve E

A

Answer: A

The pressure-volume loop is a useful concept for analyzing the relationship between contractility and loading conditions of the heart. In Figure B, Point D is the beginning of diastole. Point A is the end of diastole prior to ventricular contraction. From point D to point A there is diastolic filling. Point B is the end of isovolumetric contraction. At this point, the aortic valve opens. From point B to point C, there is isotonic contraction, and blood is ejected from the left ventricle. At point C, the aortic valve closes. From point C to point D, there is isovolumic relaxation.

In aortic stenosis, there is a marked increase in systolic pressure and end-diastolic pressure and marked decrease in stroke volume (curve A). The increased afterload results in elevated wall tension and decreased myocardial perfusion. A large ventricular septal defect results in shift of the pressure volume curve to the right with increased end-diastolic volume (curve C). Unlike the normal heart, increased preload will not result in increased stroke volume due to the maximal dilation of the heart.

How well did you know this?
1
Not at all
2
3
4
5
Perfectly
34
Q

Which of the following best characterizes the mechanisms of action of hypertonic saline solution for patients with elevated intracranial pressure?

A. Hypertonic saline solution freely crosses the blood-brain barrier and thereby reduces cerebral edema.

B. Hypertonic saline reduces the blood viscosity and free radical production in the cerebral circulation.

C. Infusion of hypertonic saline solution is used to maintain high intravascular osmolality as long as serum osmolality is maintained below 360 mOsm/L.

D. Treatment of traumatic brain injury with hypertonic saline solution causes central pontine myelinolysis.

A

Answer: C

The blood-brain barrier is nearly impermeable to both mannitol and hypertonic saline solution. The latter has been associated with fewer interventions to maintain an ICP of more than 15 mm Hg.

Hypertonic saline solution may be preferred over mannitol during the initial phase of management of traumatic brain injury because mannitol causes an osmotic diuresis, and low volume status can increase the risk of hypoperfusion. Maintaining intravascular volume status associated with hypertonic saline solution provides a reason why few adverse events occur when compared with mannitol and with sodium levels as high as 160 mEq/L. Mannitol is associated with transient blood viscosity. Hypertonic saline solution is administered as a bolus (10 mL/kg) and the drip is usually administered at a rate of 0.1 to 1 mL/kg/h.

Infusions can be continued if serum osmolality is below 360 mOsm/L. Use in patients with traumatic brain injury has not been associated with either central pontine myelinolysis or extrapontine myelinolysis.

How well did you know this?
1
Not at all
2
3
4
5
Perfectly
35
Q

Which of the following is characteristic of blood pressure after surgical repair of coarctation of the aorta?

A. Late hypertension is due to persistent hyperreninemia.

B. The incidence of postoperative hypertension is highest in children with repair early in life

C. The incidence of late hypertension is not affected by early repair

D. Early postoperative hypertension is biphasic, with the early phase primarily systolic and the late phase primarily diastolic.

A

Answer: D - Early postoperative hypertension is biphasic

Early postoperative hypertension is common following coarctation repair. Population-based studies indicate that the incidence of postoperative hypertension is higher in children with repair later in life. Control is important to minimize the risk of postoperative bleeding at the anastomosis site, minimize the risk of aneurysm formation in the poststenotic segment, and alleviate postcoarctectomy syndrome. The etiology is multifactorial; the pattern is typically biphasic. Early hypertension is thought to be explained by surgical stimulation of the sympathetic nerves located between the media and adventitia of the aortic isthmus. Norepinephrine released by this stimulation leads to immediate postoperative hypertension. The hypertension leads to renin release, which causes secondary hypertension. During this early phase, hypertension is primarily systolic, whereas diastolic hypertension is typically seen 2 to 3 days postoperatively. Renin levels return to normal by 7 days postoperatively; thus, hyperreninemia is not the cause of late hypertension. Balloon angioplasty does not stimulate norepinephrine or renin release, so hypertension is not seen.

How well did you know this?
1
Not at all
2
3
4
5
Perfectly
35
Q

A 50-kg (110-lb) child admitted to the pediatric ICU has a urine output of 75 mL/h. His serum and urine electrolyte levels are as follows: sodium, 139 mEq/L; chloride, 100 mEq/L; blood urea nitrogen, 28 g/dL; potassium, 3.5 mEq/L; bicarbonate, 24 mEq/L; creatinine, 2 mg/dL; urine sodium, 30 mEq/L; urine creatinine, 10 mg/dL. The patient’s fractional excretion of sodium is:

A. 2.3%

B. 4.3%

C. 9.3%

D. 25.3%

A

Answer: B - 4.3%

Fractional Excretion of Sodium (FeNA)=[(Urine Sodium × Plasma Creatinine)/(Plasma Sodium × Urine Creatinine)]100.

Urine chemistries, such as fractional excretion of sodium (FeNa) and fractional excretion of urea, are useful for differentiating prerenal acute kidney injury (AKI) from acute tubular necrosis in select patients. Evaluation of patients with AKI has become more standardized through the use of such definitions as the Risk-Injury-Failure-Loss-End Stage (RIFLE) and Acute Kidney Injury Network (AKIN) criteria to diagnose and classify this entity. These criteria, however, do not permit differentiation of the various types of AKI, including prerenal and acute tubular necrosis, which require nonhomogeneous management. FeNa is based on the premise that intact tubules will reabsorb sodium in the prerenal setting, whereas injured tubules occurring with acute tubular necrosis will not. FeNa less than 1% suggests a prerenal cause of AKI; FeNa greater than 3% suggests a renal cause of AKI.

How well did you know this?
1
Not at all
2
3
4
5
Perfectly
36
Q

Which one of the following options causes about 36% of pediatric patients to experience little or no analgesic effect from codeine?

A - Drug-drug interactions resulting in poor absorption

B - Polymorphism in melanocortin-1 receptor resulting in ultrarapid metabolism

C - Polymorphism in melanocortin-1 receptor resulting in poor metabolism

D - Polymorphism in CYP2D6 resulting in ultrarapid metabolism

E - Polymorphism in CYP2D6 resulting in poor metabolism

A

E - Polymorphism in CYP2D6 resulting in poor metabolism

This question describes one of the causes for up to 36% of pediatric patients to experience little or no analgesic effect from codeine. Option A is incorrect because significant drug-drug interactions that may cause limited effect of codeine in up to 36% of children is unlikely. Options B and C are incorrect because a polymorphism in melanocortin-1 receptor is associated with decreased sensitivity to the anesthetic desflurane, and not codeine. Option D is incorrect while option E is correct because a polymorphism in CYP2D6 results in poor metabolism of codeine into its active metabolite morphine, resulting in up to 36% of children experiencing little or no analgesic effect.

How well did you know this?
1
Not at all
2
3
4
5
Perfectly
37
Q

Maximum reduction in incidence of central line blood stream infections (CLABSI) in pediatric ICUs can be best accomplished by which one of the following measures?

a. Routine use antimicrobial coated central venous catheters
b. Strict implementation of catheter care bundles
c. Regular use of chlorhexidine gluconate (CHG) or silver alginate impregnated dressings
d. Regular change of catheters once every 2 weeks, even with no signs of inflammation
e. Use of chlorhexidine gluconate or povidone iodine before the insertion of central line in all children

A

b. Strict implementation of catheter care bundles

Option A is incorrect. Even though antimicrobial- coated (rifampin or minocycline) catheters can reduce colonization rate in adults, there is no data in children which supports the use antimicrobial-coated central line infections to prevent CLABSI. Option B is correct. To date, the most effective strategy to minimize CLABSI is to use of evidence-based central line care bundles. Option C is incorrect. CHG or silver alginate impregnated dressings alone are not proven to prevent CLABSI. Option D is incorrect. Regular change or rotation of asymptomatic catheters has not shown to be beneficial. Option E is incorrect. Chlorhexidine is better than povidone iodine in preventing CLABSI in children older than 2 months of age.

How well did you know this?
1
Not at all
2
3
4
5
Perfectly
38
Q

A 4-year-old is admitted to the pediatric ICU with severe traumatic brain injury. The intracranial pressure is 25 mmHg. Which one of the following values of intracranial pressure indicates the need for treatment to commence?

A - 5 mmHg

B - 10 mmHg

C - 20 mmHg

D - 30 mmHg

E - 40 mmHg

A

C - 20 mmHg

Poor outcomes have been associated with an intracranial pressure of higher than 20 mmHg. The normal intracranial pressure is usually less than 10 mmHg in adults, and varies between 3-7 mmHg in younger children; however, normal pressure is less than 6 mmHg in infants. The guidelines for severe, pediatric traumatic brain injury state that treatment should begin withan ICP of greater than or equal to 20 mmHg.

How well did you know this?
1
Not at all
2
3
4
5
Perfectly
38
Q

A 2-year-old boy is admitted to PICU immediately after accidental ingestion of his grandmother’s digoxin. Child is awake, alert, and has no symptoms. Electrolytes show sodium of 136 mEq/L, potassium of 6.9 mEq/L, chloride of 105, CO2 of 17 mEq/L, BUN of 10 mg/dL, and creatinine of 0.4 mg/dL. Which one of the following drugs is most appropriate for immediate management of this patient?

a. Intravenous furosemide 2 mg/kg
b. Intravenous glucose and insulin
c. Intravenous sodium bicarbonate 2 mEq/kg IV
d. Intravenous digoxin-specific Fab antibody fragments (DigiFab)
e. Intravenous lidocaine

A

d. Intravenous digoxin-specific Fab antibody fragments (DigiFab)

This is a child with digoxin toxicity. Digoxin blocks the Na+, K+-ATPase pump, leading to intracellular loss of K+ and gain of Na+ and Ca2+. Digoxin has a very narrow therapeutic index. Therapeutic plasma digoxin concentrations are 0.5-2.0 ng/mL; a level of >2 ng/mL is considered toxic and a level of >6 ng/mL is considered potentially fatal. Indications for digoxin –specific Fab antibody. fragments include life-threatening dysrhythmias, K+ value of >5-5.5 mEq/L in the setting of acute overdose, serum digoxin level of >15 ng/mL at any time, or >10 ng/mL 6 hours after ingestion, and ingestion of >4 mg in children or >10 mg in adults. Options A, B, and C are incorrect. This patient has digoxin toxicity and has potential to have fatal levels of digoxin. Treatment should focus on specific treatment rather than symptomatic treatment in this patient without any clinical symptoms other than hyperkalemia. Option D is correct response. Serum potassium levels of >5.5 is an indication for DigiFab. Option E is incorrect. Lidocaine can be used if patient has ventricular arrhythmias but this patient is asymptomatic.

How well did you know this?
1
Not at all
2
3
4
5
Perfectly
40
Q

In a head-injured patient, which of the following best explains the movement from Point 2 to Point 3 on the intracranial pressure curve depicted in the figure?

A. Hypothermia

B. Vasoconstriction

C. Cerebrospinal fluid drainage

D. Hypocarbia

E. Seizure

A

Answer: E - Seizure

The diagram is a reflection of the changes in intracranial compliance during a variety of disease states. The Monro-Kellie hypothesis states that the cranial vault contains a fixed volume consisting of 3 basic components—brain (80%), blood (10%), and cerebrospinal fluid (CSF) (10%)—which are encased by thick, inelastic dura mater and the semi-rigid cranium. These components exist in a state of volume-pressure equilibrium, and expansion of one requires a reduction in one or both of the other components to maintain a normal intracranial pressure. As the figure demonstrates, while movement from Point 1 to Point 2 (ie, by tumor, edema, or hemorrhage) is accompanied by an increase in brain volume, the intracranial pressure remains normal. This occurs by the initial displacement of CSF down the craniospinal axis followed by displacement of cerebral venous volume as well. Point 2 represents the point where even a slight increase in volume (ie, tumor edema, obstructive hydrocephalus, hemorrhage, etc) will lead to exponential rises in intracranial pressure that can be life- threatening as there is no further ability to compensate with decreased volume of the other 2 components. Point 3 represents a decompensated state and a neurosurgical emergency. A seizure will lead to hypoxia, hypercarbia, increased cerebral blood flow, increased metabolic demand, and possible ischemia, all contributing to the movement from Point 2 to Point 3.

Hypothermia, vasoconstriction, and hypocarbia would all reduce cerebral blood flow and hence improve, rather than worsen cerebral compliance.

How well did you know this?
1
Not at all
2
3
4
5
Perfectly
41
Q

A 19-year-old man with stem cell transplantation after recurrent treatment and relapse for acute lymphoblastic leukemia is admitted to the pediatric ICU with acute respiratory failure and altered mental status. He is now mechanically ventilated and arouses on exam to stimulation but does not communicate even with short, nonverbal response options. The patient has no written advance directive and has been estranged from his parents and has not spoken to them in over a year. He has been dating his fiancée for the past 4 years and has been living with her for the past year. Which of the following people will make decisions for the patient?

A. His fiancée, using best interest

B. His fiancée, using substituted judgment

C. His parents, using directive judgment

D. His parents, using substituted judgment

A

Answer: B - His fiancee, using substituted judgment

This patient currently has lack of capacity with altered mental status and inability to express his wishes for his care. Since he has no prewritten advance directive naming a surrogate decision maker or guidance for decision makers, a surrogate must be identified. The parents of a minor would be the de facto surrogates in most cases and would use best-interest criteria for decision making. In the case of a young adult (of legal consenting age), parents might use substituted judgment, but this would require a relationship in which the patient would express the wishes that the surrogate would use to make decisions. In this specific case the patient is estranged from his parents (implying an adversarial relationship) and the long-standing relationship with the woman who is now his fiancée would be likely to produce the appropriate venue for surrogate with the appropriate knowledge to apply substituted judgment. The important concepts here are the differences between substituted judgment and best interest. Directive judgment is not an ethical principle.

How well did you know this?
1
Not at all
2
3
4
5
Perfectly
42
Q

When using a Mapleson D system of bag and mask ventilation during sedation of an infant, which one of the following factors is the main determinant of rebreathing?

A - Length of the connecting tubing

B - Size of the breathing bag

C - Compliance of the tube

D - Size of the mask

E - Rate of fresh gas flow

A

E - Rate of fresh gas flow

A Mapleson D system is a system in which a tube carries fresh gas, traveling inside an outer reservoir tube, to the endotracheal tube connector. The source of fresh gas is delivered close to the mask. The control of rebreathing on such a circuit is mainly determined by the gas flow rate.

How well did you know this?
1
Not at all
2
3
4
5
Perfectly
42
Q

The oxygenation index (OI), one indicator of the severity of lung injury, can be calculated by the following formula:

A. (FIO2 × 100 × PaO2)/Mean Airway Pressure

B. (Mean Airway Pressure × FIO2 × 100)/PaO2

C. (Mean Airway Pressure × PaO2)/FIO2 × 100

D. PaO2 – FIO2 × 100/Mean Airway Pressure

E. PaO2/FIO2 × 100

A

Answer: B

That is the equation.

The oxygenation index is a calculation used in intensive care medicine to measure the fraction of inspired oxygen (FiO2) and its usage within the body.

A lower oxygenation index is better - this can be inferred by the equation itself. As the oxygenation of a person improves, they will be able to achieve a higher PaO2 at a lower FiO2. This would be reflected on the formula as a decrease in the numerator or an increase in the denominator - thus lowering the OI. Typically an OI threshold is set for when a neonate should be placed on ECMO, for example >40.

How well did you know this?
1
Not at all
2
3
4
5
Perfectly
42
Q

You are assessing a 16-year-old male who was intubated and mechanically ventilated at an outside hospital for respiratory failure secondary to a worsening bilateral pneumonia with a normal echocardiogram. On examination, he has poor lung compliance and is on an FIO2 of 0.35 with a PaO2 of 72 cm H2O. Which of the following best describes this patient’s acute respiratory distress syndrome stage by Berlin criteria?

A. Severe, PaO2/FIO2 (P/F) ratio less than 100

B. Moderate, P/F ratio of 100-200

C. Mild, P/F ratio of 200-300

D. Patient does not meet criteria for ARDS

A

Answer: C (Mild)

A PaO2/FIO2 (P/F) between 200 and 300 meets the definition of mild acute respiratory distress syndrome (ARDS) according to the Berlin consensus definition. In this case, the P/F ratio is 72/0.35 = 205. Prior to the Berlin Definition, this patient would have qualified as having acute lung injury (ALI) and not acute respiratory distress syndrome. The Berlin Definition has standardized the terms.

How well did you know this?
1
Not at all
2
3
4
5
Perfectly
42
Q

A 6-month-old boy with a single ventricle that was previously palliated with an aortopulmonary shunt returns from surgery after undergoing a bidirectional cavopulmonary anastomosis. His oxygen saturation is 65%. What is the most likely cause for the systemic desaturation?

A. Pulmonary arteriovenous malformations

B. Decompressing venous collaterals

C. Atrioventricular valve regurgitation

D. Increased pulmonary vascular resistance

A

Answer: D (Increased PVR)

Possible causes of hypoxemia following a bidirectional cavopulmonary anastomosis can be grouped into 3 categories: pulmonary venous desaturation, systemic venous desaturation, and decreased pulmonary blood flow. Decompressing venous collaterals are one cause of decreased pulmonary blood flow, along with undiagnosed contralateral superior vena cava (SVC). In bidirectional cavopulmonary circulation, any increase in pulmonary vascular resistance can open decompressing veins, resulting in a shunt of venous blood into the heart. Factors related to development of decompressing venous collaterals include bilateral SVC, elevated SVC pressure, and high early postoperative transpulmonary gradient. Vessels may be closed on the pre-Glenn cardiac catheterization, but a left SVC-to-coronary sinus collateral may develop postoperatively.

Pulmonary arteriovenous malformations typically cause hypoxemia months to years after the surgery. Atrioventricular valve regurgitation is less likely to affect systemic oxygenation. Increased pulmonary vascular resistance causes systemic desaturation.

How well did you know this?
1
Not at all
2
3
4
5
Perfectly
43
Q

Ventricular septal defect

A. Curve A

B. Curve B

C. Curve C

D. Curve D

E. Curve E

A

Answer: C

The pressure-volume loop is a useful concept for analyzing the relationship between contractility and loading conditions of the heart. In Figure B, Point D is the beginning of diastole. Point A is the end of diastole prior to ventricular contraction. From point D to point A there is diastolic filling. Point B is the end of isovolumetric contraction. At this point, the aortic valve opens. From point B to point C, there is isotonic contraction, and blood is ejected from the left ventricle. At point C, the aortic valve closes. From point C to point D, there is isovolumic relaxation.

In aortic stenosis, there is a marked increase in systolic pressure and end-diastolic pressure and marked decrease in stroke volume (curve A). The increased afterload results in elevated wall tension and decreased myocardial perfusion. A large ventricular septal defect results in shift of the pressure volume curve to the right with increased end-diastolic volume (curve C). Unlike the normal heart, increased preload will not result in increased stroke volume due to the maximal dilation of the heart.

How well did you know this?
1
Not at all
2
3
4
5
Perfectly
44
Q

A 12-year-old patient with Marfan’s syndrome and scoliosis is postoperative day 2 from spinal fusion. Today, his epidural was removed and he was started on a morphine PCA with continuous infusion ordered at 0.05 mg/kg/h and bolus dosing at 0.05 mg/kg/dose every 10 minutes. You are called to the floor to evaluate the patient due to poor responsiveness and find him to be extremely somnolent with HR of 72, BP of 108/68, RR of 8, and oxygen saturation of 94% on room air. He has 1-mm pupils, withdraws to noxious stimuli, and mumbles incoherently to sternal rub. Which one of the following series of actions is best in this case?

A - Disconnect PCA and administer 0.1 mg/kg (max 2 mg) naloxone and observe response, repeating every 3 minutes until breathing and alertness improve ( can administer up to max dose 10 mg). If he remains awake and appropriate for longer than 10 minutes, leave him on the floor; otherwise, PICU transfer is needed

B - Transfer to the PICU, discontinue the bolus PCA dosing, and reduce the continuous infusion to half

C - Transfer to the PICU, administer naloxone 0.1 mg/kg (max 2 mg) repeating as needed, discontinue the bolus PCA dosing, and reduce the continuous infusion to half

D - Disconnect PCA and administer 0.1 mg/kg (max 2 mg) naloxone repeating as needed, transfer to the PICU for observation

E - Disconnect PCA, transfer to the PICU for observation but avoid naloxone due to the possibility of inducing seizures

A

D - Disconnect PCA and administer 0.1 mg/kg (max 2 mg) naloxone repeating as needed, transfer to the PICU for observation

Most PCA overdoses result from medication errors (e.g., incorrect dilution) or mis-programming of the pump. It is therefore crucial to discontinue the PCA until the root cause can be determined. Naloxone (dosed as described) is indicated for reversal of respiratory depression. Since the effects of naloxone last only 45-60 minutes, and may be outlasted by the narcotic, PICU observation is warranted once an overdose has occurred. Naloxone administration may induce nonlethal withdrawal in chronic users, but is unlikely in this situation where narcotics have been administered for acute pain.

How well did you know this?
1
Not at all
2
3
4
5
Perfectly
45
Q

A 6-year-old child with spastic quadriplegia has been brought to the ED by his parents for vomiting/diarrhea for 3 days and decreased PO intake. His weight is 16 kg, while his weight in his pediatrician’s office 2 weeks ago was 18 kg. His admission serum chemistries reveal sodium of 160 mEq/lt, potassium of 3.8 mEq/lt, chloride of 124 mEq/lt, bicarbonate of 20 mEq/lt, BUN of 60 mg/dl, creatinine of 1.2 mg/dl, glucose of 108 mg/dl, and calcium of 9.0 mg/dl. Using normal values of Na of 145 mEQ/lt, K of 4.0 mEq/lt, Cl of 100 mEq/lt, bicarbonate of 24 mEq/lt, BUN of 15 mg/dl, creatinine of 0.5 mg/dl, glucose of 80 mg/dl, calcium of 10.0 mg/dl, and body water fraction of 0.6, which one of the following factors most closely approximates his free water deficit?

A - 1000 ML

B - 1500 ML

C - 2000 ML

D - 2500 ML

E - 3000 ML

A

A - 1000 ML

Free water deficit is calculated by the following equation: H2O deficit (in lts)= (Estimated body water fraction)× ( Current weight in kgs)× (plasma sodium/normal plasma sodium-1). H2O deficit= 0.6×16×160/145-1. H2O deficit= 9.6 × (1.103-1). Finally, H2O deficit is .993.

How well did you know this?
1
Not at all
2
3
4
5
Perfectly
46
Q

In patients with pericardial effusion and tamponade, pulsus paradoxus is seen in which of the following tamponade phases?

A. Phase 1 only

B. Phases 1 and 2 only

C. Phases 2 and 3 only

D. Phases 1, 2, and 3

A

Answer: B - Phases 1 and 2 only

In phase 1 tamponade, accumulation of pericardial fluid produces a rise in pericardial pressure which in turn increases right ventricular (RV) and left ventricular (LV) diastolic pressures. Cardiac output is not usually compromised. The inspiratory fall of arterial pressure may be more obvious than under normal conditions but does not usually reach the diagnostic criterion of pulsus paradoxus (a fall greater than 10 mm Hg). In phase 2 the elevated pericardial pressure becomes equal to RV diastolic pressure but remains lower than LV diastolic pressure. The transmural RV diastolic pressure approaches 0 mm Hg and accounts for a significant fall in cardiac output. The inspiratory fall of arterial pressure is exaggerated and pulsus paradoxus is present in most, but not all patients. In phase 3 tamponade, the pericardial and RV diastolic pressures equilibrate with LV diastolic pressure and these 3 pressures further increase together. Both the transmural RV and LV diastolic pressures become almost nil, consistent with minimal end-diastolic volumes. Cardiac output is severely compromised, and pulsus paradoxus is present in almost all cases.

Increased pericardial pressure obstructs both venous return and cardiac output. The classic presentation of acute cardiac tamponade is Beck’s triad (pulsus paradoxus, quiet precordium, and distended neck veins). Other common signs include tachypnea, tachycardia, narrow pulse pressure, and venous congestion (jugular venous distention, hepatomegaly, and/or crackles).

How well did you know this?
1
Not at all
2
3
4
5
Perfectly
47
Q

In a healthy 17-year-old volunteer, p-amino hippuric acid (PAH) was injected intravenously and its concentration in urine/blood and urine flow rate were calculated, which are as follows: (1) conc of PAH in urine (UPAH)= 12 mg/ml; (2) urine flow= 1 ml/min; and (3) conc of PAH in plasma (PPAH)= 0.02 mg/ml. If the hematocrit is 40%, which one of the following factors represents the renal blood flow?

a. 300 mL/min
b. 600 mL/min
c. 1100 mL/min
d. 1700 mL/min
e. 2000 mL/min

A

c. 1100 mL/min

Renal blood flow ( RBF) can be measured by applying the Fick principle (i.e., by measuring the amount of a given substance taken up per unit of time and dividing this value by the A-V difference for the substance across the kidney). Any excreted substance can be used if its concentration in the art and venous plasma can be measured and if it is not metabolized, stored, or produced by the kidney and does not itself affect blood flow. PAH is a substance which exhibits all such properties. Its extraction ratio is >90%, so for purpose of RBF measurements, the arterial concentration only is substituted for arteria-venous difference (since venous concentration is negligible). In fact, peripheral venous plasma can be used, as its PAH concentration is essentially identical to that in arterial plasma reaching the kidneys. In the current question: ERPF= UPAH ×V/ PPAH = 1× 12/0.02 = 600 ml/min. Average PAH extraction ratio 0.9. So actual RPF= 600/0.9= 660 mL/min. From the RPF, the renal blood flow can be calculated by dividing by 1 –Hct. Renal blood flow= RPF ×1/1-Hct. Finally 600×1/0.60 = 1100 mL.

How well did you know this?
1
Not at all
2
3
4
5
Perfectly
48
Q

A 4-year-old child is admitted to the PICU for dehydration and renal failure. His admission chemistry is as follows: (1) serum sodium of 140 meq/lt; (2) potassium of 4.0 meq/lt; (3) chloride of 105 meq/lt; (4) bicarbonate of 16 meq/lt; (5) serum creatinine of 1.5 mg/dl. Meanwhile, his urine sodium is 30 meq/lt and urine creatinine is 10.7 mg/dl. Which one of the following options is his fractional excretion of sodium?

a. 0.25%
b. 0.5%
c. 1.0%
d. 2.0%
e. 3.0%

A

e. 3.0%

Measuring urinary electrolyte may be worthwhile in determining the etiology of ARF. The fractional excretion of sodium (FeNa) is a measure of the kidneys ability to retain sodium. FENa is calculated by the following equation: FeNa = UNa ×PCr /UCr × PNa × 100. In this current question FeNa = 30 ×1.5/ 11 × 140 ×100= 4500/1540 which results in: 2.9. An FeNa of less than 1% signifies volume depletion or a “pre-renal” condition, and an FeNa of greater than 3% indicates intrinsic renal disease. In general, urinary sodium of 5-10 mEq/lt suggest substantial sodium retention in response to intravascular depletion and > 30 mEq/lt implies less avid sodium reabsorption, which occurs with intrinsic renal disease.

How well did you know this?
1
Not at all
2
3
4
5
Perfectly
48
Q

Which one of the following statements regarding tacrolimus use in post-transplant patients is most accurate?

a. Tacrolimus induced chronic nephropathy can be managed by dose reduction
b. Fluconazole increases tacrolimus metabolism requiring an increase in tacrolimus dose
c. Oral/nasogastric tacrolimus has less incidence of acute renal failure compared to IV tacrolimus
d. If renal dysfunction is complicating post-transplant course tacrolimus should be stopped
e. Calcium-channel antagonists should not be used to treat tacrolimus induced renal toxicity

A

c. Oral/nasogastric tacrolimus has less incidence of acute renal failure compared to IV tacrolimus

Calcineurin inhibitors are commonly used to treat acute rejection after organ transplantation. It is essential for intensivists to be familiar with this commonly used medication. Option A is incorrect because acute adverse effects of calcineurin inhibitors are related to dosage and blood concentration. Chronic nephropathy is largely irreversible and independent of dosage and blood concentration of tacrolimus. Option B is incorrect. Fluconazole is a known CYP3A enzyme inhibitor and thus can increase tacrolimus levels. Option C is the correct response. One of the strategies to prevent tacrolimus-induced renal dysfunction is to use oral/nasogastric protocol. Option D is incorrect because post-transplant renal dysfunction is generally managed by lowering the dose and target level with use of other immunosuppressive medications. Option E is incorrect because calcium-channel blockers may be beneficial in preventing nephrotoxicity induced by calcineurin inhibitors.

How well did you know this?
1
Not at all
2
3
4
5
Perfectly
50
Q

A 2-week-old male child, with history of tachypnea and poor feeding, was admitted to PICU with a diagnosis of bronchiolitis. The patient is on mechanical ventilation for respiratory distress with positive end-expiratory pressure (PEEP) of 8, FiO2 of 0.7, and pulse oximeter oxygen saturation is 88% to 95%. On examination, his heart rate is 160/minute, blood pressure is 65/35 mm kg, and capillary refill is 2 to 3 seconds. He has fixed split second sound, and no cardiac murmur. His liver is enlarged 4 cm below the costal margin. His urine output is 2 ml/kg/hr. His arterial blood gas shows pH of 7.34, PaCO2 of 40, PaO2 of 56, and base excess of negative 2. Rapid test for respiratory syncytial virus is negative. His chest X-ray is shown below. Which one of the following courses of action is the most appropriate for this child?

A - Fluid bolus followed by prostaglandin (PGE1) infusion to open ductus arteriosus

B - Start iNO at 20 parts per million and increase PEEP to 10

C - Echocardiography followed by surgery as soon as possible

D - Start continuous milrinone infusion, followed by epinephrine infusion

E - Place child on venoarterial extracorporeal membrane oxygenation (ECMO)

A

C - Echocardiography followed by surgery as soon as possible

This is a child with supracardiac total anomalous venous return (TAPVR). His chest X-ray has classic snowman appearance. This patient has pulmonary venous congestion with right heart failure. His hemodynamic status seems acceptable with no significant acidosis. Option A is incorrect because prostaglandin infusion will increase pulmonary blood flow and should be avoided. Option B is also incorrect because iNO can increase pulmonary blood flow and will worsen the situation. This patient may need iNO after surgery, but not indicated during preoperative period. Option C is the correct response because echocardiography is usually enough to diagnose total anomalous venous return TAPVR in this patient. TAPVR patients require surgical correction as soon as possible. Option D is incorrect because milrinone can increase pulmonary blood flow and epinephrine is not needed due to acceptable hemodynamic status. Option E is incorrect because this is a relatively stable patient, and there is no need to place this patient on ECMO before surgery. Some of the TAPVR patients may require ECMO after surgery, more so if they have associated hypoplastic left ventricle.

How well did you know this?
1
Not at all
2
3
4
5
Perfectly
51
Q

In a diver exposed to high barometric pressure, which one of the following is associated with the oxygen toxicity?

a. Convulsions
b. Euphoria
c. Somnolence
d. Rapture of the deep
e. High pressure nervous syndrome

A

a. Convulsions

The ambient pressure increases by one atmosphere for every 10 m of depth in seawater. Therefore at depth of 31 m in the ocean, a diver is exposed to a pressure of 4 atm. A diver must breathe air or other gases at increased pressure to equalize the increase in pressure on the chest wall and abdomen. However, at increased pressure, 100% oxygen causes central nervous system symptoms of oxygen toxicity. The main problems associated with oxygen toxicity include lung damage and convulsions. Since the harmful effects of breathing oxygen are proportionate to the PO2, they can be prevented by decreasing the concentration of oxygen in the gas mixture to 20% or less. If a diver breathes compressed air, the increased partial pressure of nitrogen can cause nitrogen narcosis—a condition also called as the “rapture of the deep.” At pressure of 4-5 atm, 80% nitrogen produces definite euphoria. At greater pressures, the symptoms resemble alcohol intoxication. The problem of nitrogen narcosis can be avoided by breathing mixtures of oxygen and helium, and deeper dives can be made. However, the high-pressure nervous syndrome (HPNS) develops during the dives with such mixtures. The condition is characterized by tremors, drowsiness, and a depression of the alpha activity in the EEG. Unlike nitrogen narcosis, intellectual functions are not severely affected; however, manual dexterity is impaired. The cost of HPNS is not settled, but it is worth noting that a variety of gases that are physiologically inert at atmospheric pressures are anesthetics at increased pressure. This is true of nitrogen and also of xenon, krypton, argon, neon, and helium. That anesthetic activity parallels their lipid solubility.

How well did you know this?
1
Not at all
2
3
4
5
Perfectly
51
Q

Which one of the following options is the osmotic activity in the isotonic saline?

a. 154 mOsm/lt
b. 77 mOsm/lt
c. 308 mOsm/lt
d. 145 mOsm/lt
e. 285 mOsm/lt

A

c. 308 mOsm/lt

The activity (concentration) of solute particles in a solution is inversely proportional to the activity (concentration) of water molecules in the solution. The solute activity in a solution is also called the osmotic activity and is expressed as osmoles (osm). The total osmotic activity in a solution is the sum of the individual osmotic activities of all the solute particles in the solution. For monovalent ions, the osmotic activity in milliosmoles (mOsm) per unit volume is equal to the concentration of the ions in milliequivalents (eEq) per unit volume. Thus, the osmotic activity in isotonic saline is 0.9% NaCl= 154 mEq Na+/lt + 154 mEq Cl-/lt = 154 mOsm Na+/lt + 154 mOsm Cl-/lt which result in 308 mOsm/lt.

How well did you know this?
1
Not at all
2
3
4
5
Perfectly
52
Q

A 4-year-old child is admitted to the PICU for dehydration and renal failure. His admission chemistry is as follows: (1) serum sodium of 140 meq/lt; (2) potassium of 4.0 meq/lt; (3) chloride of 105 meq/lt; (4) bicarbonate of 16 meq/lt; (5) serum creatinine of 1.5 mg/dl. Meanwhile, his urine sodium is 30 meq/lt and urine creatinine is 10.7 mg/dl. Which one of the following options is his fractional excretion of sodium?

A - 0.25%

B - 0.5%

C - 1.0%

D - 2.0%

E - 3.0%

A

E - 3.0%

Measuring urinary electrolyte may be worthwhile in determining the etiology of ARF. The fractional excretion of sodium (FeNa) is a measure of the kidneys ability to retain sodium. FENa is calculated by the following equation: FeNa = UNa ×PCr /UCr × PNa × 100. In this current question FeNa = 30 ×1.5/ 11 × 140 ×100= 4500/1540 which results in: 2.9. An FeNa of less than 1% signifies volume depletion or a “pre-renal” condition, and an FeNa of greater than 3% indicates intrinsic renal disease. In general, urinary sodium of 5-10 mEq/lt suggest substantial sodium retention in response to intravascular depletion and > 30 mEq/lt implies less avid sodium reabsorption, which occurs with intrinsic renal disease.

How well did you know this?
1
Not at all
2
3
4
5
Perfectly
53
Q

A 3.5-kg infant is intubated and sedated for viral pneumonia with a 3.5-mm endotracheal tube. She has had no significant changes in her ventilator support over the past 4 hours. Over the past 10 minutes, the end-tidal carbon dioxide pressure has risen from 44 to 58 mm Hg, and respiratory rate has increased from 34 to 58/min. There is no noticeable change in SpO2, heart rate, or blood pressure. Which of the following is the most likely explanation for the increase in end-tidal carbon dioxide and respiratory rate based on analysis of the airway graphics shown in the figure?

A. Flow dyssynchrony

B. Increased expiratory resistance

C. Mucus plugging

D. Tracheobronchomalacia

E. Water in the ventilatory circuit

A

Answer: C (Mucus plugging)

The flow-versus-time scalar demonstrates irregularities in both inhalation and exhalation. This represents an obstruction to gas flow. As the graphics denote a variable inspiratory flow pattern, the ventilator is able to vary flow as needed to accomplish the prescribed goals of volume or pressure delivery over the set inspiratory time. Of importance, the obstruction is somewhat variable from breath to breath and thus represents a dynamic process. The correct answer is mucus plugging, in which gas flow is being obstructed by secretions that move slightly between breaths. Water in the ventilator circuit would be seen as a fine-tooth variation in flow and generally appears as a superimposed “oscillatory” pattern. Tracheobronchomalacia would appear very similar to the graphics above, except each breath would be nearly identical.

How well did you know this?
1
Not at all
2
3
4
5
Perfectly
55
Q

A 6-month-old with hypoplastic left heart syndrome returns from OR after bidirectional cavopulmonary anastomosis procedure. Hemodynamics and oxygen saturations will be most optimized by which one of the following pulmonary/ventilator tactics?

A - Maintaining a PEEP as low as possible while still maintaining tidal volume of 9-10 cc/kg, with mild hyperventilation

B - Maintaining a PEEP of 4 cmH2O while still maintaining a tidal volume of 9-10 cc/kg, with mild hypoventilation

C - Maintaining PEEP at level needed to produce end expiratory lung volumes as close as possible to FRC, with mild hyperventilation

D - Maintaining PEEP at level needed to produce end expiratory lung volumes as close as possible to FRC, with mild hypoventilation

E - Maintaining PEEP and peak pressures as low as possible while still maintaining tidal volumes of 9-10 cc/kg

A

D - Maintaining PEEP at level needed to produce end expiratory lung volumes as close as possible to FRC, with mild hypoventilation

In referring to bidirectional cavopulmonary anastomosis: “To minimize superior vena cava pressure, it is desirable to minimize the use of positive pressure, including PEEP, following surgery. However, setting the ventilator to maintain PEEP at zero may result in atelectasis and an increase in pulmonary vascular resistance. As in the neonate with single ventricle physiology, favorable hemodynamics are most likely maintained by using ventilator settings that allow the end expiratory lung volume to approximate FRC, because pulmonary vascular resistance is lowest at FRC. In the patient with healthy lungs, minimal mean airway pressure and early tracheal extubation are often beneficial… A unique aspect of the physiology of the BCPA is that pulmonary blood flow is largely dependent on the resistance of 2 highly but differentially regulated vascular beds… Because Qp is dependent on venous return via the superior vena cava (largely made up of cerebral blood flow), maneuvers that limit cerebral blood flow may decrease pulmonary blood flow and exacerbate hypoxemia.” Thus, mild hypoventilation will improve O2 saturations via increasing cerebral blood flow.

How well did you know this?
1
Not at all
2
3
4
5
Perfectly
56
Q

The reabsorption of sodium and chloride play a major role in electrolyte and water balance regulated by the kidneys. Which one of the following parts of a nephron does not involve active transport of sodium?

a. Proximal tubules
b. Thin portion of loop of Henle
c. Thick ascending limb of loop of Henle
d. Distal convoluted tubules
e. Collecting ducts

A

b. Thin portion of loop of Henle

Normally about 60% of the filtered Na+ is reabsorbed in the proximal tubule, primarily by the Na+-H+ exchange. Another 30% is absorbed by the Na+-2Cl—K+ cotransporter in the thick ascending limb of loop of Henle (LOH), and about 7% is absorbed by the Na+ – Cl- co-transport in the distal collecting duct. The remaining of the filtered Na—about 3%—is absorbed via the ENaC channels in the collecting duct, and this is the portion that is regulated by aldosterone in the production of homeostatic adjustments in the Na+ balance. The thin portion of the LOH does not contribute to the active transport of Na+.

How well did you know this?
1
Not at all
2
3
4
5
Perfectly
57
Q

A 6-month-old patient with Fallot’s tetralogy is hospitalized beginning in mid-October for congestive heart failure. Three weeks later, he remains in the pediatric intensive care unit due to a requirement for continuous inotrope infusion and is listed for heart transplant. Which of the following next steps is most appropriate?

A - Palivizumab 15 mg/kg dosed monthly for up to 5 doses

B - Palivizumab 15 mg/kg dosed monthly for up to 3 doses

C - Intravenous immunoglobulin 10 ml/kg daily for 3 days

D - Ribavirin 15 mg/kg daily until RSV season ends

E - Ganciclovir 5 mg/kg daily for 5 months

A

A - Palivizumab 15 mg/kg dosed monthly for up to 5 doses

Palivizumab is the only effective form of prophylaxis for respiratory syncytial virus (RSV) and should be administered to children with congenital heart disease, chronic lung disease, or those born under 32 wk EGA. Essentially, up to 5 doses is appropriate for children under 2 years of age; meanwhile, up to 3 doses can be administered in patients born between 32-35 weeks prematurely. Prophylaxis is indicated in children during RSV season (generally November through March). Ribavirin and ganciclovir are not effective against RSV and intravenous immunoglobulins are not recommended, as they are far less specific than the monoclonal antibody in palivizumab.

How well did you know this?
1
Not at all
2
3
4
5
Perfectly
59
Q

Which one of the following statements is accurate regarding Pneumocystis Carini pneumonia in an HIV patient?

a. Prognosis is better in young children compared to older child and adults
b. Classified as protozoa on basis of DNA sequence analysis
c. CO2 retention is a problem from early on in the disease
d. Sensitivity of increased LDH in PCP pneumonia is reported to be between 80%-100%
e. Steroids are C/I in treatment of PCP pneumonia

A

d. Sensitivity of increased LDH in PCP pneumonia is reported to be between 80%-100%

PCP pneumonia is the most frequent reason for PICU admission in children with HIV. The incidence of PCP varies with the age of the child—the younger the child, the higher the incidence of PCP, the earlier the development of PCP and the poorer the prognosis. The etiologic agent is pneumocystis jiroveci (previously known as pneumocystis carinii). It is classified as a fungus on the basis of DNA sequence analysis. Pneumocystis jiroveci pneumonia is distinguished from other pulmonary processes by the severity of the hypoxemia, and marked increase in LDH levels. Early in the course, minute ventilation is maintained or increased, so severe hypoxemia in context of tachypnea and low PaO2 is frequently observed. An elevated LDH is the most consistent lab abnormality present in patients with PCP, sensitivity of an elevated LDH for PCP has been reported to be between 83%-100%.

How well did you know this?
1
Not at all
2
3
4
5
Perfectly
59
Q

A full-term infant born at a local hospital was found to be hypoxic. The pulse oximetry reading was 82% on room air. Administration of 100% oxygen by Oxy-Hood improved the saturations to 84%. Which one of the following tests would best and most cost-effective way to determine whether or not the etiology of this patient’s hypoxia is a result of cyanotic congenital heart disease?

A - Electrocardiogram

B - Echo

C - Hyperoxia test

D - Cardiac CT

E - Cardiac MRI

A

C - Hyperoxia test

The hyperoxia test is a test which is usually performed on an infant to determine whether the patient’s cyanosis is due to lung disease or a problem with cyanotic congenital heart disease. It is performed by measuring the arterial blood gas of the patient while he/she breathes room air, then remeasuring the blood gases after the patient has breathed 100% oxygen for 10 minutes. If the cause of cyanosis is due to poor oxygen saturation by the lungs, allowing the patient to breathe 100% oxygen will augment the lungs’ ability to saturate the blood with oxygen; thus, the partial pressure of oxygen in the arterial blood will rise above 150 mmHg. If the partial pressure of oxygen is less than 100 mmHg, the cyanosis is most likely due to cyanotic congenital heart disease. An electrocardiogram can be helpful in deciphering a heart disease, but is not used as an initial test to diagnose cyanotic congenital heart disease. Echocardiography would be the initial noninvasive test to diagnose the structural heart disease once the patient fails a hyperoxia test. Cardiac catheterization, cardiac CT, and cardiac MRI are all tests which are done either to confirm or to get additional information if echocardiography fails to answer all of the questions.

How well did you know this?
1
Not at all
2
3
4
5
Perfectly
61
Q

An 11-month-old near-drowning patient received bystander CPR and had a witnessed aspiration event. The child is now intubated and sedated due to her Glasgow Coma Scale score of 7 at the scene. Since admission she has an increasing oxygen requirement. Her core temperature is 30°C (86°F). Soon after arrival to the pediatric ICU she develops ventricular tachycardia. Which of the following steps should be taken first?

A. CPR with earliest defibrillation possible and usual antiarrhythmic medications

B. CPR only, with passive warming until core temperature above 33°C (91.4°F)

C. CPR only, with rapid rewarming of 4°C per hour

D. Extracorporeal rewarming

A

Answer: A - CPR with earliest defibrillation possible and usual antiarrhythmic medications

In moderate hypothermia (30°C-33.9°C [86°F-93°F]), it is recommended to perform CPR and defibrillate as soon as possible, with appropriate treatment of the possible causes of ventricular tachycardia, such as hypoxia, acidosis, pulmonary hypertension, hypothermia, and profound hypotension. If the core temperature were less than 30°C, defibrillation and medications would be less effective and should be limited; rewarming with cardiopulmonary support provided by extracorporeal membrane oxygenation may be a viable option. Rapid rewarming is not recommended for core temperatures over 30°C. Rewarming should occur at a rate of 1°C-2°C per hour. CPR with passive warming will take hours and delay a perfusing rhythm.

How well did you know this?
1
Not at all
2
3
4
5
Perfectly
62
Q

Compare the pressures, flows, and resistances in the pulmonary circulation with those in the systemic circulation.

A

Pulmonary blood flow is EQUAL to systemic blood flow (pulmonary and systemic circuits are in series).

However, pulmonary RESISTANCE and PRESSURE are much lower than systemic circulation

How well did you know this?
1
Not at all
2
3
4
5
Perfectly
63
Q

Osmotherapy is most effective in which one of the following forms of cerebral edema?

a. Cytotoxic
b. Vasogenic
c. Interstitial
d. Equally not effective in all forms
e. Equally effective in all forms

A

A - Cytotoxic

Cerebral edema is conventionally classified into vasogenic, cytotoxic, and interstitial edema based on its underlying pathogenesis. Vasogenic cerebral edema occurs due to disruption of blood brain barrier. Increased capillary permeability due to blood brain barriers disruption allows intravascular fluid and solutes to enter brain interstitial fluid, causing its expansion. Cytotoxic cerebral edema occurs due to accumulation of higher amount of intracellular salute compared to extracellular fluid, which leads to movement of water into the cells to maintain osmotic equilibrium. Diseases which impair cellular metabolism (hypoglycemia, rye syndrome) or sustained hyperosmolar state (hypernatremia, hyperglycemia) are associated with cytotoxic edema. Interstitial cerebral edema occurs when there is increased hydrostatic pressure gradient between the ventricle system and the brain interstitium, resulting in transependymal movement of CSF. Interstitial endemia is seen in obstructive and nonobstructive hydrocephalus. Consideration of pathophysiologic mechanism in cerebral edema has important therapeutic significance. Osmotic therapy is most effective in cytotoxic cerebral edema, whereas relief of CSS obstruction is necessary to treat interstitial cerebral edema. It is, however, important to realize that all 3 forms of cerebral edema can occur concurrently in the same patient.

How well did you know this?
1
Not at all
2
3
4
5
Perfectly
63
Q

How are FVC and FEV1 used to diagnose lung disease?

A

FVC and FEV1 are useful indices of lung disease

Normally, FEV1/FVC ~0.8 –> 80% of vital capacity can be expired in the first second of forced expiration. Also, in normal spirometry, you’ll see a change with a bronchodilator– greater than or equal to 12% in FEV1 or FVC and 200 ccs.

Obstructive lung disease such as Asthma

  • FEV1 definitely decreases; FVC could decrease slightly, or not at all, or even increase slightly!
  • Thus, FEV1/FVC DECREASES
  • Cannot expire as much due to blocked airway → lower FEV1. FVC decreases too bc you can’t expire as much but it also “expands” a little because you’re leaving some air in the lung each time you expire.

Restrictive lung disease such as fibrosis
FEV1 decreases, but FVC decreases a lot more
–> Increased/normal FEV1/FVC

How well did you know this?
1
Not at all
2
3
4
5
Perfectly
64
Q

Which of the following tests is most useful for predicting neurologic outcome after cardiopulmonary resuscitation (CPR) in children?

A. Biomarkers, specifically neuron-specific enolase and S100b protein

B. Burst-suppression pattern on electroencephalography

C. CT of the head

D. MRI of the head

A

Answer: B - Burst suppression on EEG

Many resuscitation studies report end points of return of sustained circulation or survival to hospital discharge. Data about neurologic outcome and predictors of neurologic outcome after adult and pediatric cardiac arrest are limited. Information about the predictive value of available tests such as clinical neurologic examinations, neurophysiologic diagnostic studies (eg, electroencephalography [EEG] or somatosensory-evoked potentials), biomarkers, or neuroimaging is also limited.

What is known is that CT is not sensitive in detecting early neurologic injury. MRI with diffusion weighting can provide valuable information about hypoxic-ischemic injury in the subacute and recovery phases. Postarrest EEG showing a burst-suppression pattern is a sensitive and specific predictor of poor neurologic outcome. Somatosensory evoked potentials may be a highly sensitive and specific predictor of outcome in pediatric patients after cardiac arrest, but these tests are not standardized, and results are difficult to interpret.

Biomarkers may be useful to predict neurologic outcome. In an adult study, neuron-specific enolase levels greater than 33 μg/L and S100b levels greater than 0.7 μg/L were highly sensitive and specific for poor neurologic outcome (defined as death or persistent unconsciousness). These tests have not been adequately evaluated in children.

How well did you know this?
1
Not at all
2
3
4
5
Perfectly
65
Q

Which one of the following characteristics of an ingested drug makes it more amenable to be cleared by dialysis?

A - Drug with large volume of distribution

B - Drugs with low-volume of distribution

C - Large molecular weight

D - Highly proteins bound drugs

E - All drugs are equally dialyzable

A

B - Drugs with low-volume of distribution

The ability of an extracorporeal therapy to remove an intoxicant is affected by the volume of distribution of the drug. Generally, drugs with volume of distribution of

How well did you know this?
1
Not at all
2
3
4
5
Perfectly
67
Q

Which one of the following brain structures is most resistant to hypoxia?

A - Brainstem

B - Cortex

C - Basal ganglia

D - Thalamus

E - Inferior colliculus

A

A - Brainstem

Oxygen consumption by the human brain averages about 3.5 ml/100 gms of brain/minute in an adult. This figure represents approximately 20% of the total body resting oxygen consumption. The brain is extremely sensitive to hypoxia and occlusion of its blood supply produces unconsciousness in as short of a period as 10 seconds. The vegetative structures in the brainstem are more resistant to hypoxia than the cerebral cortex, and patients may suffer from an accident such as cardiac arrest and other conditions causing fairly profound hypoxia, with normal vegetative functions but serious, permanent intellectual deficiencies. The basal ganglia use oxygen at a very high rate, and symptoms of Parkinson’s disease as well as intellectual deficit can be produced by chronic hypoxia. The thalamus and the inferior colliculus are also very susceptible to hypoxic changes.

How well did you know this?
1
Not at all
2
3
4
5
Perfectly
69
Q

Which one of the following conditions is associated with prolongation of Q-T interval?

a. Hyperkalemia
b. Hyperthyroidism
c. Hyponatremia
d. Hypothermia
e. Hypermagnesemia

A

d. Hypothermia

Causes of QT prolongation are as follows:

How well did you know this?
1
Not at all
2
3
4
5
Perfectly
70
Q

An 18-month-old girl presented to ED with fever of 39 °C. During examination, the patient had a generalized tonic-clonic seizure which lasted for 9 minutes. A head CT demonstrated no intracranial pathology. After lumbar puncture, the patient was admitted to PICU for observation. Her cerebrospinal fluid (CSF) results showed no white cells, glucose and protein were normal, and Gram stain was negative for bacteria, but CSF quantity was not sufficient to do routine CSF culture for bacteria. In the ED, they could not draw blood for any tests. In the PICU, patient is awake, alert, and playful. She interacts well with her parents. Her temperature is 38.2 °C, and there is no external focus of infection. Which one of the following tests will be most helpful in the acute management of this patient?

A - Electroencephalogram (EEG) and MRI of brain

B - Complete blood counts and blood culture

C - Serum metabolic and electrolyte profiles with calcium, phosphorous, magnesium, and glucose

D - Urine analysis with microscopy and urine culture if necessary

E - Repeat lumbar puncture and CSF culture to rule out bacterial meningitis

A

D - Urine analysis with microscopy and urine culture if necessary

This is a case of a toddler with simple febrile seizures. Emphasis in simple febrile seizure is minimal intervention and the routine laboratory and neurodiagnostic tests are unnecessary. Option A is incorrect because these tests will not help in identifying cause, nor will they help in predicting the outcome in simple febrile seizures. Option B is incorrect. Child is awake, alert, and there is no definite focus of infection. In this scenario, blood work will not help in the management of this patient. Option C is also incorrect because these are unnecessary tests in simple febrile seizures. Option D is correct. Even though children with febrile seizures do not have increased risk for urinary tract infections, urine analysis may help to identify cause for fever in this child. Prevalence of UTI in a febrile children aged 2 months to 2 years without any focus on examination is 5%. Option E is incorrect. This patient is not high risk for meningitis and repeat lumbar tap is unnecessary.

How well did you know this?
1
Not at all
2
3
4
5
Perfectly
71
Q

The ECG tracing shown in the figure is obtained in a 3-year-old patient who was admitted with cardiac decompensation.

What is the most likely diagnosis?

A. Myocarditis

B. Ectopic atrial tachycardia

C. Wolff-Parkinson-White syndrome

D. Anomalous left coronary artery from the pulmonary artery

E. Kawasaki disease with coronary thrombosis and myocardial infarction

A

Answer: A - Myocarditis

The ECG tracing shown is most consistent with a diagnosis of myocarditis. The voltages are diminished, and there is flattening and inversion of the T wave. Myocardial infarction pattern also can be seen in patients with myocarditis whose ECG tracing is characterized by wide Q waves and changes in the ST segment. Anomalous left coronary artery from the pulmonary artery is characterized by Q waves in leads I and aVL. Ectopic atrial tachycardia usually is characterized by an abnormal P wave axis, although this abnormality can occur in the sinus region and thus mimic sinus tachycardia. There is no evidence for myocardial infarction as seen with Kawasaki disease with coronary aneurysm and thrombosis. ECG tracings of patients with Wolff-Parkinson-White syndrome would show ventricular preexcitation, which can be a cause of ventricular dysfunction due to unrecognized sustained tachyarrhythmias.

How well did you know this?
1
Not at all
2
3
4
5
Perfectly
72
Q

A 5-year-old child with MR/CP is admitted to the PICU with pneumonia. He is noted to have a serum potassium of 2 mEq/lt. He has a complex medical history and is on multiple home medications, information of which is currently unavailable. His urine potassium is 40 mEq/lt and urine chloride is 30 mEq/lt. In view of these values, which one of the following options is the most likely cause of his hypokalemia?

a. Nasogastric drainage
b. Alkalosis
c. Diarrhea
d. Inadequate intake
e. Diuretic therapy

A

e. Diuretic therapy

Hypokalemia is a serum potassium of 25 mEq/lt) when magnesium depletion or diuretics are responsible. Magnesium depletion impairs potassium re-absorption across the renal tubules and plays an important role in promoting and sustaining potassium depletion. Extra renal potassium loss is a major cause is diarrhea. Potassium conc in stool is 75 mEq/lt, stool volume is normally 200 ml or less, thus little potassium is lost. In diarrheal states, daily volume can be as high as 10 lt and thus severe diarrhea can result in significant potassium depletion. In diarrheal losses, urinary potassium will be low.

How well did you know this?
1
Not at all
2
3
4
5
Perfectly
73
Q

A 4-year-old has been in the pediatric ICU for 3 weeks and is now in the recovery phase of acute respiratory failure secondary to influenza complicating chronic lung disease. Which of the following laboratory tests is correctly matched with its representation of the onset of protein malnutrition?

A. Albumin; 3 days

B. Prealbumin; 4 days

C. Retinol binding protein; 5 days

D. Transferrin; 1 day

A

Answer: D (Transferrin)

The onset of protein malnutrition in the nonacute phase of critical illness may be measured by a variety of laboratory tests, each with a different half-life. Albumin has the longest half-life and low values represent malnutrition as far back as 30-60 days with a half-life of 20 days. Transferrin represents malnutrition as far back as 12-16 days with a half-life of 8 days. Retinol binding protein represents malnutrition as far back as 1-2 days with a half-life of 10 hours. Prealbumin has a half-life of 2 days and may represent malnutrition 3-4 days in the past. It must be remembered that all these measures are acute phase reactants; levels may be depressed in the acute phase of critical illness, and interpretation requires contextual data.

How well did you know this?
1
Not at all
2
3
4
5
Perfectly
75
Q

A 5-month-old male patient is admitted due to cough and fever with concerns for pneumonia. Shortly after admission, he develops respiratory distress and is intubated. This is the patient’s first hospitalization. He had 2 previous doctor visits due to otitis media that resolved with appropriate antibiotics and for recurrent diarrhea and thrush. Based upon the most common defect associated with this disorder, what would you expect his flow cytometry to reveal?

A - (-) T cells, (+) B cells, (-) NK cells

B - (-) T cells, (-) B cells, (+) NK cells

C - (-) T cells, (-) B cells, (-) NK cells

D - (+) T cells, (-) B cells, (+) NK cells

E - (+) T cells, (+) B cells, (+) NK cells

A

A - (-) T cells, (+) B cells, (-) NK cells

This case illustrates severe combined immune deficiency. The 3 major types of lymphocyte are T cells, B cells, and natural killer (NK) cells. NK cells are a part of the innate immune system and play a major role in defending the host from both tumors and virally infected cells. T cells (thymus cells) and B cells (bursa-derived cells) are the major cellular components of the adaptive immune response. T cells are involved in cell-mediated immunity whereas B cells are primarily responsible for humoral immunity (relating to antibodies). Combined immunodeficiency is a defect in T and NK cell development, thus they are negative.

How well did you know this?
1
Not at all
2
3
4
5
Perfectly
76
Q

Which one of the following statements is true regarding inflicted abdominal trauma in children?

A - Death rate of abdominal trauma victims of child abuse is 5%-10%

B - All children with child abuse should be screened with abdominal CT scan

C - Trauma is not a common cause of pancreatitis in children

D - Child abuse consultants often screen for occult abdominal injury using hepatic transaminases

E - Concomitant injuries are not commonly seen in children with inflicted abdominal injuries

A

D - Child abuse consultants often screen for occult abdominal injury using hepatic transaminases

Abdominal trauma is the second most common cause of death in children with child abuse. Even though incidence of abdominal trauma is low in child abuse, mortality of inflicted abdominal injury is around 45%. Option A is incorrect because death rate of abdomen trauma after child abuse is higher due to delayed recognition of injuries. Option B is incorrect. Routine abdominal CT scan of all child abuse children has low yield (3%) and is not recommended. Option C is incorrect because trauma is the primary cause of pancreatitis in children. Option D is correct. Alanine aminotransferase or aspartate aminotransferase levels of >80 IU/L has 77% sensitivity and 82% specificity in diagnosing occult abdominal trauma. Child abuse consultants often use screen occult abdominal injury using hepatic transaminases.

How well did you know this?
1
Not at all
2
3
4
5
Perfectly
77
Q

An 18-month-old girl had repair of ventricular septal defect and returned to the ICU. She is on a ventilator and receiving tidal volume of 7 ml/kg. Her peak inspiratory pressure is 21 cm of H2O; positive end-expiratory pressure (PEEP) is 5 cm of H2O; and FiO2 is 0.3. Her O2 saturation on pulse oximeter is 99%. Her chest X-ray shows that lungs are expanded to 7 ribs. Her blood pressure on admission was 84/42 mmHg. Two hours after admission, the nurse calls you to the bedside for blood pressures readings of 60/31 mmHg. Patient’s central venous pressure reading from right subclavian line is 4 cm of H2O. Which one of the following interventions has the best chance to restore this patient’s blood pressure back to normal?

a. Five percent albumin bolus of 20 ml/kg
b. Decrease PEEP to 3 cm of H2O
c. Decrease tidal volume to 6 ml/kg
d. iNO 20 ppm
e. Milrinone infusion at 0.5 mcg/kg/minute

A

a. Five percent albumin bolus of 20 ml/kg

This patient’s blood pressure is low because of low right atrial pressure. Within normal physiological boundaries, right atrial pressure is the main factor that determines right ventricular output and in turn total cardiac output. Option A is correct because fluid bolus will increase right atrial pressure, right ventricular output, and cardiac output restoring blood pressure. Option B is incorrect. Lungs are not over-expanded on chest X-ray and decreasing PEEP in will not decrease pulmonary vascular resistance and will not improve blood pressure. Option C is incorrect. Tidal volumes within normal limits will not affect pulmonary vascular resistance or right ventricular output. Option D is incorrect because this patient does not have clinical features of pulmonary hypertension and iNO will not be useful for this patient. Option E is incorrect because in the setting of low venous return to right heart, milrinone can further decrease blood pressure. Milrinone is mainly used in the setting of low cardiac output state secondary to myocardial dysfunction after cardiac surgeries.

How well did you know this?
1
Not at all
2
3
4
5
Perfectly
78
Q

A 12-year-old boy is admitted to the ICU after he was found unconscious in a house fire. He is now intubated and on FiO2 of 0.6. His pulse oximeter reads the arterial oxygen saturations (SaO2) as 100%. His carboxyhemoglobin level is 25%. Which one of the following options best describes this patient’s SaO2 and oxyhemoglobin (oxy-Hb) dissociation curve shift?

A - SaO2 is falsely elevated and the oxygen hemoglobin dissociation curve is shifted to the right

B - SaO2 is falsely elevated and the oxy-Hb dissociation curve is shifted to the left

C - SaO2 is appropriately elevated and oxy-Hb dissociation curve is shifted to the right

D - SaO2 is appropriately elevated and oxy-Hb dissociation curve is shifted to the left

E - FiO2 of 0.6 is causing elevated SaO2 and there is no shift in the oxy-Hb dissociation curve

A

B - SaO2 is falsely elevated and the oxy-Hb dissociation curve is shifted to the left

Carbon monoxide has a higher affinity to bind with Hb as compared to oxygen. Option B is correct because binding of carbon monoxide to hemoglobin causes increased affinity of oxygen to bind to the remaining sites. This is reflected as a leftward shift of the oxy-Hb dissociation curve. Carboxyhemoglobin is interpreted as oxyhemoglobin by the pulse oximeter—hence, in this clinical scenario, SaO2 is falsely elevated. Options A and C are incorrect because the oxy-Hb curve is shifted to the left. Option D is incorrect as the SaO2 are falsely elevated, and cooximetry is imperative in clinical suspicion of carbon monoxide poisoning. Option E is incorrect because there is a leftward shift of oxy-Hb curve in carbon monoxide poisoning.

How well did you know this?
1
Not at all
2
3
4
5
Perfectly
79
Q

A severely hypoxic patient has a baseline PaO2 of 50 mmHg and PaCO2 of 80 mm Hg. An air bubble in the atrial blood gas sample would affect the results in which way?

A. The PaO2 would be decreased and the PaCO2 would be decreased.

B. The PaO2 would be decreased and the PaCO2 would be increased.

C. The PaO2 would be increased and the PaCO2 would be decreased

D. The PaO2 would be increased and the PaCO2 would be increased.

A

Answer: C

An air bubble consists of atmospheric gas with a partial pressure of oxygen theoretically PaO2 = 154 mm Hg and a PaCO2 = 0. Hence, this patient’s blood gas would be altered with dilution of the PaCO2 and spurious elevation of the PaO2. If the patient were not hypoxic, the PaO2 could be decreased if their PaO2 were greater than 154 mm Hg of atmospheric PO2; however, this patient is severely hypoxic with a low PaO2.

How well did you know this?
1
Not at all
2
3
4
5
Perfectly
80
Q

Diastolic runoff or low diastolic pressure as one pathophysiology for developing necrotizing enterocolitis occurs in which of the following cardiac congenital lesions?

A. Critical coarctation

B. Large ventricular septal defect

C. Tetralogy of Fallot

D. Truncus arteriosus

A

Answer: D - Truncus arteriosus

The pathophysiology of necrotizing enterocolitis (NEC) in cardiac infant patients presumably entails intestinal hypoperfusion. Diastolic runoff and low diastolic pressures in truncus arteriosus may contribute to intestinal hypoperfusion. NEC also can occur in critical coarctation and hypoplastic left heart syndrome because of critical flow of systemic circulation through patent ductus arteriosus manifested as circulatory shock. Splanchnic complications can occur with large ventricular septal defect if low cardiac output occurs caused by lung overflow and pulmonary hypertension.

How well did you know this?
1
Not at all
2
3
4
5
Perfectly
81
Q

Microbiology laboratory techniques that are useful in determining the correct type and dosage of an antimicrobial agent against a specific bacterial isolate include:

A. Blood culture

B. Enzyme-linked immunosorbent assay

C. Gram stain

D. Latex agglutination

E. Susceptibility tests

A

Answer: E - Susceptibility tests

Antimicrobial susceptibility is a commonly used method for determining the antimicrobial activity of a bacterial isolate. Antimicrobial susceptibility is expressed in terms of the minimum inhibitory concentration (MIC) and minimum bactericidal concentration (MBC) of an antimicrobial agent. The MIC and MBC indicate the degree to which a microorganism is susceptible to an antimicrobial agent, and thus are useful in determining the type and dosage of antibiotic to use. Latex agglutination and ELISA are used primarily to detect bacterial antigen in body fluids. The Gram stain may demonstrate the presence and morphologic characteristics of a bacteria, but are nonspecific. The blood culture may assist in identifying a bacterial isolate but does not assist in determining the susceptibility profile.

How well did you know this?
1
Not at all
2
3
4
5
Perfectly
83
Q

Which one of the following congenital cardiac abnormalities is least likely to be associated with coarctation of aorta (CoA)?

A - Supravalvular mitral stenosis

B - Parachute mitral valve

C - Subaortic stenosis

D - Transposition of great arteries

E - Pulmonary stenosis

A

E - Pulmonary stenosis

CoA may be a part of complex left-sided obstructions identified as Shone’s syndrome which consists of coarctation, supravalvular mitral stenosis, parachute mitral valve, and subaortic stenosis. Syndromes that have low pulmonary blood flow, such as tetralogy of Fallot, tricuspid atresia, pulmonary stenosis, and atresia are rarely associated with CoA.

How well did you know this?
1
Not at all
2
3
4
5
Perfectly
84
Q

A child is admitted to ICU for metabolic acidosis of unknown etiology. His measured serum osmolality is 340 mOsm/lt . His electrolytes are as follows: (1) sodium of 145 mEq/lt; (2) potassium of 3.5 mEq/lt; (3) chloride of 111 mEq/lt; (4) bicarbonate of 14 mEq/lt; (5) BUN of 28 mg/dl; (6) creatinine of 1.2 mg/dl;

a. 5
b. 10
c. 20
d. 30
e. 40

A

d. 30

Plasma osmolality is calculated by the following equation: 2× plasma Na + glucose/18 + BUN/ 2.8. Therefore, 2×145+ 180/18 + 28/2.8 is the equation in this question. 290 +10+10=310 mOsm/Kg H2O. Thus, the osmolar gap 30 mOsm/Kg H2O. Due to the fact that solutes other than sodium, chloride, glucose, and urea are present in the extracellular fluid, the measured plasma osmolality will be greater than the calculated plasma osmolality. This osmolar gap is normally up to 10 mOsm/Kg H2O. An increase in the osmolar gap occurs when certain toxins (e.g., ethanol, methanol, ethylene glycol, or the unidentified toxins that accumulate in renal failure) are in the extra cellular fluid.

How well did you know this?
1
Not at all
2
3
4
5
Perfectly
85
Q

Which one of the following options is a risk factor for propofol infusion syndrome?

a. Old age
b. Bacterial infection/meningitis
c. High carbohydrate intake
d. Low fat intake
e. Concomitant steroid use

A

e. Concomitant steroid use

Propofol infusion syndrome associated with a high dose (>4 mg/kg/hour) and prolonged use (>48 hours). Risk factors are as follows: (1) young age; (2) critical illness; (3) high fat and low carb diet; (4) inborn errors of mitochondrial fatty acid oxidation; and (5) Concomitant catecholamine infusion or steroid therapy. Finally signs and symptoms include severe metabolic acidosis, CV collapse, rhabdomyolysis, hyperlipidemia, renal failure, and hepatomegaly.

How well did you know this?
1
Not at all
2
3
4
5
Perfectly
86
Q

Which one of the following options is the correct order of organs in respect to their oxygen consumption?

a. Brain > heart > kidneys > liver > skeletal muscle
b. Heart muscle > brain > kidneys > liver > skeletal muscle
c. Heart muscle > kidneys > brain > liver > skeletal muscle
d. Skeletal muscle > liver > heart > brain > kidneys
e. Liver > brain > heart > skeletal muscle > kidneys

A

c. Heart muscle > kidneys > brain > liver > skeletal muscle

Resting oxygen consumption of various organs in a 63 kg adult male with MAP of 90 mmHg and oxygen consumption of 250 mL/min is seen in the following figure:

How well did you know this?
1
Not at all
2
3
4
5
Perfectly
87
Q

Which one of the following options is the correct order of organs in respect to their oxygen consumption?

a. Brain > heart > kidneys > liver > skeletal muscle
b. Heart muscle > brain > kidneys > liver > skeletal muscle
c. Heart muscle > kidneys > brain > liver > skeletal muscle
d. Skeletal muscle > liver > heart > brain > kidneys
e. Liver > brain > heart > skeletal muscle > kidneys

A

c. Heart muscle > kidneys > brain > liver > skeletal muscle

Resting oxygen consumption of various organs in a 63 kg adult male with MAP of 90 mmHg and oxygen consumption of 250 mL/min is seen in the following figure:

How well did you know this?
1
Not at all
2
3
4
5
Perfectly
88
Q

A 17-year-old boy, who was in a motor vehicle accident, presents at the ED. He is making incomprehensible sounds. He showed no response to command; however, on giving painful stimuli to the right shoulder, he opened his eyes and tried to reach for his shoulder. Which one of the following values represents his GCS score?

a. 7
b. 8
c. 9
d. 10
e. 11

A

c. - 9

89
Q

A 13-year-old girl has developed fulminant liver failure following ingestion of acetaminophen. Which of the following is the most likely explanation for development of cerebral edema in this patient?

A. Astrocyte swelling secondary to intracellular glutamine accumulation

B. Direct neuronal injury by acetaminophen

C. Excessive gama-aminobutyric acid (GABA) receptor stimulation by benzodiazepine-like substances

D. N-acetyl-p-benzoquinonimine (NAPQI) stimulation of N-methyl-D-aspartate (NMDA) receptors

E. Regional ischemia secondary to decreased neuronal nitric oxide production

A

Answer: A - Astrocyte swelling secondary to intracellular glutamine accumulation

The mechanism by which cerebral edema develops in hepatic failure is not fully understood. However it appears likely that a major factor leading to cerebral edema is accumulation of glutamine in the astrocyte. Increased ammonia (NH3) diffuses across the blood-brain barrier into the astrocyte where it is deaminated, and glutamate is converted to glutamine. Increased glutamine in the astrocyte promotes water uptake into the cell along an osmotic gradient. Under usual circumstances glutamine is pumped out of the astrocyte and taken up by presynaptic neurons where it is converted back to glutamate. However, elevated levels of ammonia impede export of glutamine from the astrocyte. Nerve stimulation causes release of glutamate into the synaptic cleft, where it acts as an excitatory neurotransmitter. In addition, glutamate stimulates N-methyl-D-aspartate (NMDA) receptors, probably stimulating neuronal nitric oxide synthase (nNOS), increased nitric oxide production, cerebral vasodilatation, and further predisposition to cerebral edema.

Although acetaminophen has been associated with direct renal toxicity, there is no evidence for direct neuronal injury.

Benzodiazepine-like substances do stimulate gamma-aminobutyric acid (GABA) receptors in hepatic encephalopathy, but it is not likely that these substances are associated with the development of cerebral edema. Flumazenil has been demonstrated to reverse neurologic depression seen in early hepatic encephalopathy, but the effect is transient.

N-acetyl-p-benzoquinonimine (NAPQI) is a toxic by-product of acetaminophen metabolism by the hepatic cytochrome P-450 mixed-function oxidase enzymes. Under usual circumstances it is detoxified by glutathione but when the glutathione supply is exhausted following overdose, NAPQI accumulates, binds to hepatocytes, and causes necrosis. There are no data to suggest that it stimulates NMDA receptors in the brain.

As noted above, nitric oxide production is probably increased in hepatic encephalopathy.

90
Q
A
91
Q

A patient known to have myasthenia gravis (MG) was admitted for pneumonia inadvertently administered amikacin as an antibiotic. Soon after administration of the antibiotic, a significant worsening of patient’s respiratory efforts were noted. Which one of the following options is the best antidote for this adverse effect of an aminoglycoside antibiotic?

a. Propantheline
b. Dicyclomine
c. Calcium
d. Pentolinium
e. Isoproterenol

A

c. Calcium

Aminoglycoside (AMG) associated neuromuscular blockade causing weakness of skeletal muscle is a very uncommon adverse reaction but may precipitate respiratory depression. This may be due to competitive antagonism of the acetylcholine (ACh) activity at the neuromuscular junction. Patients at particular risk to experience AMG induced neuromuscular blockade include those with MG, severe hypocalcemia, infantile botulism or those who have recently received neuromuscular blocking agents. IV infusion of calcium salt will successfully antagonize this adverse drug affect.

92
Q

A 16-year-old, 50-kg girl who is upset from a recent breakup with a boyfriend ingested at least 50 extra strength (500 mg) acetaminophen tablets and awoke this morning vomiting. In the ER, she has an acetaminophen level drawn and is administered a loading dose of 150 mg/kg N-acetylcysteine (NAC), but vomits once she arrives in the PICU. At this point, her acetaminophen level is 100 mg/L approximately 10 hours following ingestion (toxicity begins at 40 mg/L). On exam, she’s nauseated, but otherwise able to follow commands and answer questions. Her vital signs are normal except for her HR of 112. Her AST is 3280 and her ALT is 1740. Her PT is greater than 100 s and her INR is 9, and her creatinine is 2.4 mg/dL. Which one of the following therapeutic plans is the best early plan?

A - Administer IV NAC repeating the 150 mg/kg dose, PICU observation with hourly neurologic evaluation

B - Administer IV NAC repeating the 150 mg/kg dose, PICU observation with hourly neurologic evaluation and urgent liver transplant evaluation

C - Administer IV NAC repeating the 150 mg/kg dose, PICU observation with hourly neurologic evaluation, urgent liver transplant evaluation and neurosurgical evaluation for intracranial pressure monitoring

D - Continue on the oral NAC algorithm, observe in the PICU with hourly neurological evaluation

A

B - Administer IV NAC repeating the 150 mg/kg dose, PICU observation with hourly neurologic evaluation and urgent liver transplant evaluation

This is a fairly severe acetaminophen overdose based on the level and the evidence of early liver and renal failure. These latter factors make the child high risk for requiring liver transplantation and this evaluation should commence immediately. IV NAC is as effective, as oral and sensible when a patient is vomiting. The use of intracranial pressure monitoring is reserved for situations where there is no neurological exam that can be followed.

94
Q

A 2-month-old girl developed poor feeding and irritability early this morning. She was brought to ED due to respiratory distress. On examination, she looks mottled and has weak pulses. Her rhythm strip in lead II showed the following rhythm (vent tach). The emergency physician tried cardioversion with 0.5 J/kg, but rhythm on the monitor did not change. Which one of the following options is the next most appropriate intervention?

A - 25 mg/kg of IV magnesium sulphate

B - Synchronized cardioversion with 0.25 J/kg

C - Synchronized cardioversion with 2 J/kg

D - Defibrillation with 2 J/kg

E - 0.1 mg/kg of IV adenosine

A

C - Synchronized cardioversion with 2 J/kg

This is a child with unstable ventricular tachycardia. Rhythm strip shows monophasic ventricular tachycardia. In this scenario, electrical cardioversion is recommended. If lower energy fails to convert or break ventricular tachycardia, synchronized cardioversion with higher energy should be tried. Option A is incorrect because magnesium sulphate has little demonstrable effect in the acute treatment of monophasic ventricular tachycardia. Option B is incorrect because lower energy level will not help to convert this rhythm. Option C is correct. When 0.5 J/kg fails to convert ventricular tachycardia rhythm, higher energy with 1 to 2 J/kg should be used for synchronized cardioversion. Option D is incorrect because defibrillation is used in ventricular fibrillation, and this patient also has weak pulse. In this scenario, synchronized cardioversion is recommended. Options E is incorrect because IV adenosine is not beneficial in children with ventricular tachycardia.

95
Q

A 16-year-old boy arrives to the ED by ambulance following cardiac arrest that occurred following a forceful tackle during a football game. After the tackle, the boy passed out, was unresponsive, and spontaneous movement as absent. The boy was revived following cardiopulmonary resuscitation and paramedics returned the boy to normal sinus rhythm with one 200-J countershock. The patient has no prior history of heart disease, arrhythmias, or congenital anomalies. An initial workup demonstrated a normal metabolic panel and negative drug screen. Which one of the following causes of the cardiac arrest is most appropriate considering the patient’s history?

A - Myocardial infarction

B - Long-QT syndrome

C - Hypertrophic cardiomyopathy

D - Commotio cordis

E - Aneurysm

A

D - Commotio cordis

This question describes the most appropriate diagnosis of a 16-year-old male football player who went into cardiac arrest following a forceful tackle during a football game. Due to the observation that the cardiac arrest occurred following a forceful tackle, option D is correct. Commotio cordis is a condition characterized by cardiac arrest due to trauma to the chest and is a common cause of cardiac arrest in young athletes. Options A, B, C, and E are incorrect because the patient history and results of the initial workup do not indicate myocardial infarction, long-QT syndrome, or hypertrophic cardiomyopathy.

96
Q

Which one of the following statements regarding blood brain barrier (BBB) has the most accurate description of its structure or function?

a. Capillary endothelium forms the principle component of BBB along with pericytes and astrocytes
b. Water-soluble substrates cross BBB more readily compared to lipid-soluble molecules
c. Morphine induces narcotic effect more rapidly compared to fentanyl
d. Glucose require energy-dependent transporters of ATP binding cassette to cross BBB
e. Benzodiazepines do not penetrate into the cerebrospinal fluid across the intact BBB

A

a. Capillary endothelium forms the principle component of BBB along with pericytes and astrocytes

Option A is the correct response. Cellular component of the BBB comprises endothelial cells of brain capillaries, pericytes, foot process of astrocytes, and neurons. Endothelial cells are linked together by tight junctions, which prevent paracellular diffusion of substances. Option B is incorrect because lipid-soluble substrates cross BBB more readily than water-soluble compounds. Option C is incorrect because fentanyl is more lipophilic and induces narcotic effect more rapidly than morphine. Option D is incorrect because glucose is transported across BBB by energy-independent mechanism by GLUT1 transporter. Option E is also incorrect because benzodiazepines being more lipophilic penetrate readily into the cerebrospinal fluid (CSF).

97
Q

In an otherwise healthy 16-year-old male which of the following bodily fluids contains the highest sodium concentration?

a. Urine
b. Gastric secretions
c. Diarrhea
d. Small bowel secretions
e. Sweat

A

d. Small bowel secretions

Sodium concentration in the body fluids:

98
Q

A 15-year-old boy who has had a 4-day history of progressive weakness in the lower extremities is now unable to walk. He appears anxious but otherwise has a normal mental status. Physical examination reveals symmetric distal muscle weakness, absent ankle jerks, a weak knee-jerk reflex, and a normal sensory pattern. Which of the following is the most important next test in the assessment of his clinical status?

A. Creatine kinase level

B. Echocardiography

C. MRI of the brain

D. Negative inspiratory force and vital capacity

E. Train-of-four testing

A

Answer: D - NIF and vital capacity

This patient most likely has Guillain-Barré syndrome (GBS), which is defined as a postinfectious polyneuropathy that causes demyelination of motor nerves, although sensory nerves also can be affected. Respiratory failure associated with GBS is life-threatening and requires early recognition.

Pulmonary function tests, such as measurement of negative inspiratory force and vital capacity, are helpful in detecting impending respiratory failure due to intercostal or phrenic nerve involvement. Reductions in inspiratory force often precede blood gas abnormalities and should be monitored frequently in patients with acute progressive muscle weakness. Intubation and mechanical ventilation should be considered in the following circumstances:

  • Forced vital capacity
  • Maximum negative inspiratory force > –20 to –30 cm H2O
  • Hypoventilation leading to PaCO2 >50 mm Hg
  • Pulmonary secretions that cannot be managed by chest physiotherapy • Loss of protective airway reflexes
  • Progressive atelectasis

Onset of paralysis usually follows a nonspecific illness by about 10 days. Symptoms of the original infection may have been limited to the respiratory tract (Mycoplasma pneumoniae) or gastrointestinal tract (Campylobacter jejuni). Weakness associated with GBS classically begins in the lower extremities and moves up, progressively involving the trunk, the upper limbs, and finally the bulbar muscles. Bulbar involvement occurs in half of patients and may result in respiratory insufficiency. Dysphagia and facial weakness are often impending signs of respiratory failure. The Miller Fisher variant is characterized by ophthalmoplegia, ataxia, and areflexia with relatively little weakness. Other variants exist, including descending GBS (facial or bulbar muscles involved first). Autonomic dysfunction is a major risk factor for mortality in patients with GBS. Fatal cardiovascular collapse can occur. Risk is highest in patients with respiratory failure, quadriplegia, or bulbar involvement.

Cerebrospinal fluid studies show albuminocytologic dissociation with elevated protein level and normal cell count usually by the second week of illness and are not definitive in diagnosis. Rapidly progressive ascending paralysis is treated with IV immunoglobulin. Plasmapheresis also has been used; both therapies are equally effective.

99
Q

Which one of the following statements is correct regarding blood flow through blood vessels?

a. RBC and plasma proteins contribute equally to the viscosity of blood
b. Resistance of fluid to changes in flow rate is a property known as density
c. Velocity of blood flow decreases as blood vessels narrows
d. Viscosity of blood decreases as the blood moves into the small blood vessels
e. A decrease in Hct is associated with a proportional decrease in cardiac output

A

d. Viscosity of blood decreases as the blood moves into the small blood vessels

Resistance of fluid to changes in flow rate is known as viscosity. The viscosity of whole blood is almost entirely due to cross-linking of circulating erythrocytes by plasma fibrinogen. The principle determinant of whole blood viscosity is the concentration of circulating erythrocytes. The velocity of blood flow increases as the blood vessels narrow, the viscosity of the blood will also decrease as the blood moves into the small blood vessels in the periphery. The decrease in viscosity occurs because the velocity of plasma increases more than the velocity of erythrocytes, so relative plasma volume increases in the small blood vessels. This process is called “sheer thinning.” Since a decrease in hematocrit is associated with a decrease in viscosity, and from Poiseuille equation, as viscosity decreases the blood flow increases. The progressive decrease in Hct is associated with a steady rise in cardiac output.

100
Q

A 3-year-old boy weighing 14 kg (31 lbs) is admitted to the pediatric ICU with respiratory distress and a right upper lobe infiltrate. The next day, he develops diffuse infiltrates on chest radiograph and is intubated for impending respiratory failure. Blood cultures grow Streptococcus pneumoniae. Conventional ventilator settings are as follows: synchronized intermittent mandatory ventilation/pressure support; set rate, 30/min; peak inspiratory pressure, 28 cm H2O; positive end-expiratory pressure, 8 cm H2O; and delivered tidal volume, 100 mL. What is this patient’s respiratory elastance?

A. 0.15 cm H2O/mL

B. 0.20 cm H2O/mL

C. 0.28 mL/cm H2O

D. 2.0 mL/cm H2O

E. 5.0 mL/cm H2O

A

Answer: B (0.20 cm H20/mL)

Elastance is defined as the inverse of compliance. Compliance is the change in volume (ie, tidal volume) divided by the change in pressure (ie, delta-P). Thus, elastance is the change in pressure divided by the change in volume. In this case, elastance is 20 cm H2O divided by 100 mL, or 0.2 cm H2O/mL.

100
Q

Which of the following inotropic agents can be used to improve cardiac output without significantly increased myocardial consumption?

A. Dobutamine, 15 μg/kg/min

B. Dopamine, 12 μg/kg/min

C. Epinephrine, 0.1 μg/kg/min

D. Isoproterenol, 0.3 μg/kg/min

E. Milrinone, 0.75 μg/kg/min

A

Answer: E (Milrinone)

Milrinone is a phosphodiesterase-3 inhibitor that usually does not increase heart rate or blood pressure, which are the principal determinants of myocardial oxygen consumption. Epinephrine, dopamine, dobutamine, and isoproterenol are all sympathomimetic agents that stimulate beta1 receptors in the heart. All of these agents increase heart rate and/or blood pressure, resulting in increased myocardial oxygen consumption.

101
Q

Which of the following is the most common clinical occurrence in the adolescent following repair of coarctation of the aorta?

A. Left ventricular dysfunction

B. Ischemia of the artery of Adamkiewicz during aortic cross-clamp time

C. Rebound hypertension

D. Mesenteric arteritis

E. Gradient of 25 mm Hg between upper extremity and lower extremity

A

Answer: C - Rebound hypertension

Coarctation of the aorta in the older child causes high obstruction of systemic blood flow and leads to left ventricle overload and pulmonary edema. This obstruction causes aortic collaterals to be present. The relief of this obstruction can cause a rebound hypertension postoperatively. In the immediate postoperative phase this is believed to be due to both catecholamine release from manipulation of the aortic arch and the stimulation of the renin-angiotensin system. Hypertension is controlled through beta-blockade, vasodilators, analgesia, and sedation. Spinal cord injury due to lack of collaterals and ischemia to the spinal cord by not perfusing the anterior spinal artery can cause ischemic shock to the cord and paralysis. Mesenteric arteritis can occur as the intestinal vasculature responds to the introduction of pulsatile flow by vasoconstriction. This vasoconstriction can cause intestinal ischemia, which may cause bowel infarction.

Left ventricular dysfunction may be seen in the neonatal period, but rarely in the adolescent population.

102
Q

A 16-year-old girl took an unknown amount of oil of Wintergreen after a fight with her boyfriend. Serum salicylate level 4 hours after ingestion was 60 mg/dl. Which one of the following options is the primary site of excretion of salicylate in kidneys?

a. Glomerular filtration
b. Proximal tubular secretion
c. Passive diffusion in the ascending limb off loop of Henle
d. Distal convoluted tubules
e. Collecting duct

A

b. Proximal tubular secretion

Salicylic acid is a weak acid that exists in charged (deprotonated, Sal-) and charged (protonated, H+) form H+ + sal- ↔ HS. Uncharged molecule (HS), unlike charged molecules (sal- ), can move easily across cellular barriers, including the blood-brain barrier (BBB) and the epithelium of the renal tubule. Metabolic acidosis drives this equation to the right and increases plasma concentration of HS, thereby promoting diffusion across BBB into the CNS. Salicylic acid is highly protein bound, it enters the urine primarily via secretion by the organic anion secretary pathway in proximal tubule, rather than by glomerular filtration. Alkalization of the urine converts urinary salicylic acid to ionized salicylate, minimizing the back diffusion of the secreted salicylic acid out of the tubular lumen. Raising the urine pH from 6.5 to 8.1 by administration of sodium bicarbonate can increase total salicylic excretion more than 5 fold.

104
Q

A 5-year-old child, with septic shock, is being monitored by both noninvasive cuff blood pressure (BP) recording and invasive arterial line (through a radial arterial line). You are explaining the principles behind calculation of blood pressure by both the modalities to the residents. Which one of the following statements would be most apt for this setting?

A - In automated noninvasive BP reading, systolic and diastolic are measured values, and mean arterial pressure (MAP) is calculated—while in invasive BP, MAP is measured value and systolic and diastolic are calculated values

B - In automated noninvasive BP, MAP is a measured value, while systolic and diastolic are calculated values; in invasive BP, systolic and diastolic are measured values while MAP is a calculated value

C - All values are measured in both noninvasive and invasive blood pressure

D - Both modalities measure MAP and calculate systolic and diastolic values

E - Both modalities measure systolic and diastolic values and calculate MAP

A

B - In automated noninvasive BP, MAP is a measured value, while systolic and diastolic are calculated values; in invasive BP, systolic and diastolic are measured values while MAP is a calculated value

Automated devices have simplified and standardized the measurement of blood pressure, but the method of BP measurement using these devices is different from the traditional auscultation methods. Quite simply, the automated cuff inflates and deflates automatically, detecting the presence of the amplitude of pulsations in the arm. The peak amplitude is the mean arterial pressure; the systolic and diastolic pressures are then calculated from an analysis of the peak amplitude of pulsation, and rate of increase and decline of its pulsations. Therefore, MAP is the measured value—and systolic and diastolic readings are only calculated values. The invasive pressure monitoring does the opposite. It directly measures the systolic and diastolic pressures, and calculates the mean from those measurements.

104
Q

Intracerebral pressure recordings through a ventricular catheter are characterized by many well defined wave forms. Which one of the following waveforms is considered most ominous?

a. A-waves
b. B-waves
c. C-waves
d. D-waves
e. E-waves

A

A - A-waves

Ventricular waveforms, recorded through an ICP transducer, show the following waves: A-waves or plateau waves – amplitude of 50-100 mmHg, lasting 5-20 minutes. These waves are always associated with intracranial pathology. During such waves it is common to observe evidence of early herniation, including bradycardia and hypertension. They are thought to occur when cerebral perfusion pressure is inadequate to meet metabolic demands. B-waves – oscillating waves, up to 50 mmHg in amplitude and a frequency of 0.5-2 per minute. They are thought to be due to vasomotor center instability. C-waves – oscillating waves, up to 20 mmHg in amplitude and a frequency of 4-8 per minute. These waves have been documented in healthy individuals and are thought to occur because of interaction between cardiac and respiratory cycles. Both A and B waves require intervention to reduce ICP and preserve CPP. However A-waves (or plateau waves) are considered more ominous.

105
Q

A 15-year-old male is in the pediatric ICU with resolving traumatic brain injury. He has had resolution of his acute phase of injury that was managed with hyperosmolar therapy, intracranial pressure/cerebral perfusion pressure–targeted therapy, and barbiturate burst-suppression. He is now out of barbiturate burst-suppression and remains intubated awaiting neurologic recovery. He remains on total parenteral nutrition due to ischemic bowel injury and hollow viscus rupture at the time of his motor vehicle accident. His glucose infusion rate is 15 mg/kg/min and his calculated glucose infusion needs are 4 mg/kg/min. Which of the following most likely represents his respiratory quotient?

A. ≥1

B. 0.8

C. 0.5

D. ≤0.1

A

Answer: A

This patient’s glucose infusion rate far exceeds his energy needs as well as his glucose as an energy source. The recommended glucose infusion rate is 4-5 mg/kg/min. Any energy intake in excess of his energy expenditure needs, especially excess glucose, will be converted to lipid stores. This occurs through hepatic lipogenesis and is associated with increased carbon dioxide production. A respiratory quotient greater than or equal to 1 best represents the state of anabolism described here.

107
Q

A 10-year-old boy presents to the ED with a red rash with central clearing on his right inner thigh, a headache, and general malaise. The mother of the child states that the rash developed 2 days prior and has spread to its current baseball-like size. Infection by which one of the following pathogens is responsible for the rash?

A - B. burgdorferi

B - S. aureus

C - D. variabilis

D - I. scapularis

E - F. tularensis

A

A - B. burgdorferi

This question describes an infection with a pathogen that caused a red rash with central clearing on the thigh of boy. Option A is correct because B. burgdorferi causes a characteristic rash called erythema migrans in two-thirds of those infected. Option B is incorrect because although S. aureus infection of the skin can result in a circular red area, it does not cause the characteristic erythema migrans. Options C, D, and E are incorrect because the tick-borne pathogens D. variabilis, I. scapularis, and F. tularensis cause Rocky Mountain spotted fever, anaplasmosis, and tularemia, respectively—but, they do not result in erythema migrans.

108
Q

If ketamine is administered as an intramuscular injection, rather than intravenously, a larger dose will be necessary to induce sedation due to which one of the following factors?

A - Incomplete absorption

B - Slower absorption

C - Tissue metabolism

D - Tachyphylaxis

E - Up-regulation of drug receptors

A

B - Slower absorption

Ketamine has a bioavailability of 93% when it is given via an intramuscular injection. Its absorption rate is 2-17 minutes which is in contrast to intravenously injected ketamine which has a rapid onset. Furthermore, intramuscular injection does not appear to be affected by tissue metabolism, tachyphylaxis, or up-regulation of the drug receptors.

109
Q

A critically ill 2-year-old is admitted to the pediatric ICU with 30% partial- and full-thickness burns. Which of the following hormones in this child is most likely to be decreased from a normal baseline state?

A. Cortisol

B. Epinephrine

C. Growth hormone

D. Insulin

E. Thyroxine

A

Answer: E - Thyroxine

Neurendocrine responses in critically ill children tend to follow a similar response irrespective of the etiology for critical illness. While cortisol and growth hormone responses have been of debate during the acute phase of critical illness, thyroxine has been shown to be decreased in the majority of studies done to date. In these children, free thryoxine is depressed but without an increase in thyroid-stimulating hormone (sick euthyroid state). There is an increase in reverse triiodothyronine (rT3), but not in bioavailable thyroxine. The clinical impact of this abnormality is still highly debated, without consensus on the subject of supplementation and repletion during critical illness.

111
Q

A 5-year-old child is transferred to your pediatric intensive care unit from a rural hospital after being found wandering and disoriented in a corn field. His father, who is with him, states that he did not witness any ingestions, but that he is on oxycodone and diazepam for recent lower back pain and his wife takes an antidepressant of which he is uncertain of the proper name. On exam, his HR is 67, his BP is 128/82, his RR is 23, and his saturations are 93% on room air. He has 1-mm pupils and experiences significant drooling. He is slightly agitated but can follow commands, though his words are nonsensical. The remainder of his exam is unremarkable. Which one of the following next steps is most appropriate given this information?

A - Immediate intubation for airway protection

B - Placement of nasogastric tube with charcoal lavage

C - Administration of naloxone and flumazenil IV in consultation with poison control while awaiting toxicology results

D - Administration of atropine and potentially pralidoxime in consultation with poison control

E - Await toxicology results and continued pediatric intensive care unit observation

A

D - Administration of atropine and potentially pralidoxime in consultation with poison control

This child is presenting with acetylcholine excess which likely represents organophosphate (insecticide) poisoning, based on the given circumstance. Immediate treatment with atropine can help symptoms; however, the use of “oximes” to prevent toxin curing is more controversial. At this point there is no clear indication for airway control and the other agents described would be ineffective in treating this intoxication.

112
Q

What are the four lung capacities?

A

Lung Capacities (capacity = sum of >2 lung volumes)

1) Inspiratory capacity (IC): tidal volume + inspiratory reserve volume
~3500 mL

2) Functional residual capacity (FRC): expiratory reserve volume + residual volume
~2400 mL
-FRC is volume in lungs after a normal tidal volume (after normal expiration) and can be thought of as the equilibrium volume of the lungs

3) Vital capacity (VC): inspiratory capacity + expiratory reserve volume
~4700
-Maximum volume that can be expired after maximal inspiration
-Value increases with body size, male gender and physical conditioning, and decreases with age

4)Total lung capacity (TLC): vital capacity + residual volume
~5900 mL
Total volume in lungs after a maximal inspiration

114
Q

Which of the following patients most likely has an elevated right atrial pressure of 14 mm Hg and a normal left atrial pressure?

A. A 3-month-old infant with Ebstein anomaly and moderate tricuspid valve regurgitation

B. A 4-month-old infant with a residual right ventricular outflow tract gradient of 60 mm Hg after tetralogy of Fallot repair

C. A 5-year-old child with restrictive cardiomyopathy

D. A 12-year-old child with chronic dilated cardiomyopathy

A

Answer: B

Restrictive cardiomyopathy is characterized by elevated right and left atrial pressures and pulmonary hypertension. Similarly, chronic dilated cardiomyopathy is also associated with elevated right and left atrial pressures. An infant with Ebstein anomaly and moderate tricuspid valve regurgitation will have a relatively compliant right atrium. These patients can have regurgitation and an enlarged right atrium without a significantly elevated right atrial pressure. A child with tetralogy of Fallot will have right ventricular hypertrophy and a stiff right ventricle, which is exacerbated by residual right ventricular outflow tract obstruction. These patients will have elevated right atrial pressure but normal left atrial pressure.

116
Q

A 5-day-old infant with hypoplastic left heart syndrome (mitral atresia, aortic atresia) is admitted to the cardiac ICU after a stage 1 palliation operation with a 3.5-mm Blalock-Taussig shunt 18 hours ago. The operation was uneventful; however, because of bleeding after separation from cardiopulmonary bypass, his chest was left open. On rounds on postoperative day 1, temperature is 36.5°C (97.6°F), HR is 170/min, central venous pressure via right internal jugular vein line is 16 mm Hg, common atrial pressure via right atrial line is 15 mm Hg, BP is 73/40 mm Hg, and SaO2 is 85%. The patient is sedated and paralyzed, has cool peripheries and bounding central pulses, and urine output is minimal despite a furosemide infusion started 12 hours postoperatively. Mixed venous oxygen saturation is 45%, serum lactate level is 4 mmol/L, and hematocrit is 40%. Arterial blood gas results on ventilation with FIO2 of 0.21, ventilator rate of 14/min, and tidal volume of 10 mL/kg are pH of 7.32, PaCO2 of 50 mm Hg, PaO2 of 50 mm Hg, and base excess of –7 mEq/L. The infant is supported with dopamine, 5 μg/kg/min. Chest radiography done this morning shows well-expanded lungs. Assuming the pulmonary vein saturation is 98%, the next step in managing this patient is to:

A. Administer bicarbonate to correct base deficit

B. Administer blood to increase hematocrit toward 50%

C. Increase dopamine to 10 μg/kg/min and consider adding norepinephrine

D. Increase ventilator rate to decrease PaCO2

E. Start milrinone infusion at 0.5 μg/kg/min

A

Answer: E - Start milrinone infusion

The patient in this scenario has a high Qp:Qs and is stealing from his systemic circulation towards his pulmonary circulation. In patients with shunted single-ventricle physiology,

Qp:Qs = (Systemic SaO2 – Mixed Venous [Superior Vena Cava] SaO2)/(Pulmonary Vein SaO2 – Pulmonary Artery SaO2).

In single-ventricle circulation, Systemic SaO2 = Pulmonary Artery SaO2. Thus, Qp:Qs = (85 – 45)/(98 – 85) = 3.1.

Thus, the patient has large pulmonary blood flow. The Qp:Qs is generally a balance of pulmonary vascular resistance and systemic vascular resistance (SVR). The patient is already on low ventilator rate and FIO2 of 0.21, so the next step is introducing an inodilator such as milrinone to decrease SVR. Increasing dopamine infusion and adding epinephrine may increase SVR further and worsen Qp:Qs. His bleeding has stopped and he appears adequately volume expanded based on his right atrial pressure. Bicarbonate administration will not help correct metabolic acidosis that is due to imbalance of Qp:Qs in shunted single-ventricle physiology.

116
Q

A 1-day-old, full-term neonate has cyanosis, tachypnea, and an oxygen saturation of 82% in room air by pulse oximetry. Chest radiograph shows streaky bilateral interstitial opacities. Cyanotic congenital heart disease is suspected. A hyperoxia test is performed with PaO2 levels obtained from both preductal and postductal sites with the patient breathing 100% oxygen. Which of the following is the most likely diagnosis with a preductal PaO2 of 250 mm Hg and a postductal PaO2 of 80 mm Hg?

A. Primary pulmonary hypertension

B. Severe pulmonary parenchymal disease

C. Congenital cyanotic heart disease with right-to-left intracardiac shunting

D. Heart disease with complete intracardiac mixing lesion without obstruction of pulmonary blood flow

A

Answer: A - Primary pulmonary hypertension

The hyperoxia test consists of having the patient breathe 100% oxygen followed by blood gas analysis. A PaO2 higher than 250 mm Hg in both preductal and postductal sites makes critical structural cyanotic heart disease very unlikely (often referred to as “passing” the hyperoxia test). An arterial PaO2 less than 100 mm Hg in the absence of obvious lung disease (“failed” hyperoxia test) is most likely due to intracardiac right-to-left shunting and is virtually diagnostic of cyanotic congenital heart disease. If arterial PaO2 is between 100 and 250 mm Hg, results are equivocal; patients may have structural heart disease with complete intracardiac mixing and greatly increased pulmonary blood flow, as is sometimes seen with single-ventricle complexes. Markedly higher oxygen content in the upper compared with the lower part of the body (ie, differential cyanosis) is consistent with a diagnosis of left ventricular outflow obstruction, critical aortic arch obstruction, or persistent pulmonary hypertension.

117
Q

A child is admitted to ICU for metabolic acidosis of unknown etiology. His measured serum osmolality is 340 mOsm/lt . His electrolytes are as follows: (1) sodium of 145 mEq/lt; (2) potassium of 3.5 mEq/lt; (3) chloride of 111 mEq/lt; (4) bicarbonate of 14 mEq/lt; (5) BUN of 28 mg/dl; (6) creatinine of 1.2 mg/dl; (7) glucose of 180 mg/dl; and (8) calcium of 9.0 mg/dl. Which one of the following values represents his osmolar gap?

A - 5

B - 10

C - 20

D - 30

E - 40

A

D - 30

Plasma osmolality is calculated by the following equation: 2× plasma Na + glucose/18 + BUN/ 2.8. Therefore, 2×145+ 180/18 + 28/2.8 is the equation in this question. 290 +10+10=310 mOsm/Kg H2O. Thus, the osmolar gap 30 mOsm/Kg H2O. Due to the fact that solutes other than sodium, chloride, glucose, and urea are present in the extracellular fluid, the measured plasma osmolality will be greater than the calculated plasma osmolality. This osmolar gap is normally up to 10 mOsm/Kg H2O. An increase in the osmolar gap occurs when certain toxins (e.g., ethanol, methanol, ethylene glycol, or the unidentified toxins that accumulate in renal failure) are in the extra cellular fluid.

118
Q

A 2-year-old girl is on venovenous extracorporeal membrane oxygenation (VV ECMO) for acute hypoxic respiratory failure. During morning rounds, patient is on ECMO flows of 100 ml/kg with premembrane O2 saturation of 75%, postmembrane O2 saturation of 100%, and cephalad catheter O2 saturation of 65%. You do not make any changes to extracorporeal membrane oxygenation (ECMO) parameters. In the afternoon, patient’s premembrane O2 saturation increases to 90% and cephalad O2 saturation decrease to 55%. ECMO flows are still 100 ml/kg with post membrane O2 saturations of 100%. Which one of the following options is the most likely cause for these changes in O2 saturations?

A - Thrombus formation in the membrane oxygenator

B - Improved cardiac output of the patient

C - Decreased intravascular volume of the patient

D - Cephalad catheter affecting ECMO flows

E - Partial occlusion of ECMO catheter

A

C - Decreased intravascular volume of the patient

These changes of O2 saturations on ECMO reflect increased recirculation fraction from 28%-77%. This will adversely affect the oxygen supply of the patient. The factors that affect recirculation on ECMO are pump flow, catheter position, cardiac output, and right atrial size (intravascular volume). Option A is incorrect. Thrombus formation in the membrane oxygenator will decrease post membrane O2 saturations and can affect flows. In this case, both have not changed. Option B is in correct because if patient’s native cardiac output has increased it will decrease premembrane O2 saturations. Option C is the correct response because decreased intravascular volume or right atrial size will increase recirculation and hence increase premembrane O2 saturations and decrease mixed venous saturations. Option D is incorrect. Cephalad catheter augments the amount of venous drainage and will not increase recirculation. Option E is incorrect. Partial occlusion of catheter may decrease ECMO flows and will not affect recirculation.

119
Q

Your patient is a previously healthy, 7-year-old girl with community-acquired pneumonia. She has been on vancomycin and cefotaxime for 4 days. Her fever curve is unchanged and she continues to spike fevers up to 40°C (104°F). Her C-reactive protein level improved from 25 to 20 mg/L with the initiation of antibiotics but has been stable at 20 mg/L for the past 3 days. She continues to require high-flow nasal cannula at 12 L/min and an FIO2 of 0.5 for the past 3 days. With this treatment her oxygen saturations have remained in the 92%-97% range. Arterial blood gas studies done this morning show pH of 7.38, PaCO2 of 45 mm Hg, and bicarbonate level of 29 mEq/L. Chest radiograph is shown in the figure. Ultrasonography shows absence of septations. The most important next step would be to:

A. Change antibiotics to linezolid as the organism is most likely not sensitive to the current antibiotic regimen.

B. Consult surgery for an open thoracotomy.

C. Drain the effusion and consider tPA through the chest tube if it is not draining well.

D. Initiate bilateral positive airway pressure to help with atelectasis, which should improve her fevers.

E. Schedule bronchoscopy with bronchoalveolar lavage to ensure treatment of the correct organism.

A

Answer: C - Drain the effusion and consider tPA through the chest tube if it is not draining well

The patient has a large left pleural effusion that needs to be drained in order to effectively treat the infection. The empyema is the likely cause for continued fevers and lack of clinical or laboratory improvement. It is unlikely that the organism is resistant to her current antibiotics, so there is no reason to change antibiotics or perform a bronchoscopy with bronchoalveolar lavage unless draining the effusion does not improve her clinical picture. As the patient is oxygenating and ventilating well with her current support, and her chest radiograph is more consistent with a

large effusion than atelectasis, bilateral positive airway pressure would not be as beneficial as draining the effusion. A surgical thoracotomy may be necessary eventually. However, the best first step would be to drain the effusion and consider using tPA to aid in draining the fluid if it is purulent. This may save your patient from needing a more invasive procedure.

120
Q

A 3-year-old girl who weighs 20 kg is admitted to PICU with diagnosis of status asthmaticus. She receives 5 mg/kg of aminophylline loading dose over 20 minutes. Blood level of aminophylline immediately after loading dose is 12 mcg/ml. Assuming no elimination or clearance has occurred, which one of the following values represents the correct volume of distribution of aminophylline in this patient?

A - 8.33 L

B - 0.42 L

C - 0.83 L

D - 60 L

E - Not able to determine from the given information

A

A - 8.33 L

The formula to calculate volume of distribution (Vd) is as follows: Vd = total amount of drug in the body (drug) / concentration of drug in the blood or plasma (Cp). In this patient total drug given is 5 mg x 20 (weight of the patient) is 100 mg. Substituting the values in the formula Vd = 100 / 12 = 8.33 L. Option A is the correct response because this is the value obtained from the formula. Option B is not correct. If it were to be volume of distribution/kg then Vd would have been 0.42 L/kg. Options C, D, and E are all incorrect.

121
Q

At a recent symposium you learned that a commonly used anticonvulsant may be useful for patients who require mechanical ventilation by reducing their requirements for propofol, benzodiazepines, and other sedatives. The medication has been widely prescribed as an anticonvulsant for years and has a good safety profile. You would like to use it off-label in your ventilated patients for the next year and compare the doses of sedation used in these patients to historical controls from your ICU. Based on the preliminary data that you have reviewed, you believe this is a clinically reasonable option that should not require the specific consent of the patient or surrogate, and you proceed without submission for institutional review board (IRB) review. Which of the following statements best reflects adherence to established ethical practice in human subjects research?

A. Because the intervention meets the requirement for “minimal risk,” informed consent is not required.

B. Because you intend to collect data on the patients and pursue publication, this constitutes human subject research and requires IRB review.

C. If you were not planning on studying it, you would not need to obtain specific informed consent for use of this drug, so it is not necessary in this case.

D. No consent is required because this is not a randomized controlled trial and no patients are receiving placebo.

E. The prescription of any drug for an off-label indication requires specific consent from the patient or surrogate under the guidance of the IRB.

A

Answer: B

Physicians are not required to obtain institutional review board (IRB) review or specific informed consent for off-label use of medications with prior FDA approval. The fact that you intend to prescribe a drug to your patients based on preliminary information you learned at a symposium is not prohibited. However, if a patient were to have a complication related to administration of the drug, you would be responsible for defending your assessment of the risks versus the benefits of the drug under the circumstances.

In this case, however, since you are intending (a priori) to collect data and compare them to historical controls, this is human subject research. As such, your patients are now research subjects, and the process requires IRB review. This is true regardless of the study design, and even if the patients are not randomized. Under some circumstances institutional review boards may grant a waiver of consent if the research involves minimal risk, but this is not determined by the investigator alone.

122
Q

Which one of the following options is a side effect of propofol?

A - Increase in respiratory drive

B - Lactic acidosis

C - Elevation of blood pressure

D - Hypoglycemia

E - Hypotriglyceridemia

A
123
Q

Which one of the following statements is accurate regarding epidural hematomas?

a. Occurs from traumatic events with significant force, most often involving motor vehicles
b. Often associated with primary brain injury and carry a much worse prognosis than SD hematoma
c. On CT scan, they have a characteristic crescentic appearance
d. Two-thirds of epidural hematomas result from tear of middle meningeal artery
e. Due to risk of clot expansion, almost all epidural hematomas need surgical evacuation

A

d. Two-thirds of epidural hematomas result from tear of middle meningeal artery

Epidural hematoma occurs when a skull fracture or surface contact force traverses an epidural vessel with sufficient force to rapture the vessel. These injuries can occur from low height falls, especially in infants and young children whose skulls are thin and malleable. Because an arterial epidural hematoma can expand rapidly, a potentially life-threatening injury can occur from household falls and should be kept in mind when this mechanism is reported. Two-thirds of epidural hematomas result from tear of the middle meningeal artery, and thus cannot be managed without surgical evacuation and cauterization of this vessel. Epidural hematoma, when treated with immediate surgical evacuation, carries a relatively good prognosis with less than 10% mortality rate. Subdural hematoma – usually occur from traumatic events with significant force, most often involving motor vehicles. Acute subdural hematomas models are generally evacuated urgently, but because of large forces involved, are often associated with significant underlying primary brain injury. They carry much worse prognosis than epidural hematomas. In infants, the most common etiology of acute subdural hematoma is nonaccidental injury.

125
Q

You are running a code blue of a 10-month-old child who had sudden cardiopulmonary arrest in the pediatric floor. Patient is now intubated and being bagged with 100% oxygen with good chest rise. He has received total bolus of 60 ml/kg of normal saline. He is getting effective chest compressions and his pulse oximeter reading is 99%. He has intraosseous (IO) needle placed in his left tibia. When chest compressions are paused for assessment, his heart rate on the monitor is 56 with normal sinus rhythm. He has cold extremities with capillary refill of 4 seconds. According to American heart association (AHA) guidelines, which one of the following steps in management should ensue with this patient?

a. Normal saline bolus of 20 ml/kg
b. Atropine 0.02 mg/kg through IO
c. Epinephrine 0.01 mg/kg through IO
d. Transthoracic pacing with rate of 100/minute
e. Stop chest compressions and observe the child

A

c. Epinephrine 0.01 mg/kg through IO

This patient has bradycardia with cardiopulmonary compromise and needs continued CPR. Option A is not correct because this patient has already received adequate volumes of fluids and needs pharmacological intervention for bradycardia with cardiorespiratory compromise. Option B is also not correct. Atropine is the not the first-line drug for symptomatic bradycardia. Atropine can be considered for increased vagal tone or primary AV block. Option C is correct because (according AHA guidelines) epinephrine is the drug of choice for persisting symptomatic bradycardia. Option D is not the correct response. Transthoracic pacing is not the first line of treatment in this scenario. Option E is incorrect. This patient is still not resuscitated completely and needs continued CPR.

126
Q

Which one of the following options is a common electrolyte abnormality seen with use of tacrolimus?

a. Hypokalemia
b. Hyponatremia
c. Hypocalcemia
d. Hypomagnesia
e. Hypophosphatemia

A

d. Hypomagnesia

Hyperkalemia and hypomagnesaemia can be a significant problem on chronic tacrolimus therapy, with reported incidence of up to 45%. Potassium sparing diuretics must be used with caution in patients receiving tacrolimus.

127
Q

Which one of the following muscles is last to be paralyzed following administration of an nondepolarizing neuromuscular blocker?

a. Forearm muscles
b. Muscles of vocal cord
c. Facial muscles
d. Intercostal muscles
e. Diaphragm

A

E - Diaphragm

Neuromuscular transmission requires the binding of 2 acetylcholine (Ach) molecules (1 Ach molecule binds to each alpha subunit); thus, even if only 1 Ach binding site is occupied by the nondepolarizing neuromuscular blocker, Ach activation is effectively inhibited. However, approximately 90%-95% of the Ach receptors must be blocked before neuromuscular transmission is completely inhibited. The diaphragm is more densely populated with Ach receptors than are other muscles, so the diaphragm may continue to function even after the muscles of the hand and upper airway have been effectively paralyzed.

128
Q

Which one of the following options is a side effect of permissive hypercapnia?

A - Decrease in cerebral perfusion

B - Increased myocardial contractility

C - Increased sympathetic activity

D - Decreased GI motility

E - Decreased renal blood flow

A

C - Increased sympathetic activity

Carbon dioxide is a powerful vasodilator of cerebral vessels. Increasing CO2 levels can result in cerebral edema and increased ICP, which can aggravate cerebral disorders, such as cerebral trauma or hemorrhage and cerebral occupying lesions. For this reason, the use of permissive hypercapnia is contraindicated in the presence of such disorders as head and intracranial disease. Permissive hypercapnia is relatively contraindicated in preexisting cardiovascular instability. Circulatory effects of permissive hypercapnia can include decreased myocardial contractility, arrhythmia, vasodilation, and increased sympathetic activity. A common finding in a patient receiving permissive hypercapnia is increased cardiac output, normal systemic blood pressure, and pulmonary hypertension.

129
Q

In a 6 month old with a history of bronchopulmonary dysplasia (BPD), which one of the following changes in ventilator settings would potentially have the most effect on decreasing the patient’s arterial carbon dioxide level?

A - Decreasing the inspiratory cycle off to 10%

B - Increasing the inspiratory cycle off to 40%

C - Increasing the set PEEP level to below the intrinsic PEEP level

D - Slow the inspiratory rise time

E - Speed up the inspiratory rise time

A

A - Decreasing the inspiratory cycle off to 10%

Changing the inspiratory cycle off can shorten or lengthen the inspiratory cycle. In a patient with BPD, they commonly have increased airway resistance that may lead to carbon dioxide retention when placed on positive pressure ventilation. Decreasing the inspiratory cycle off to 10% would shorten the inspiratory cycle and allow longer time for expiration and CO2 removal. Increasing the inspiratory cycle off has the opposite effect and usually is used in patients with stiff lungs to prolong inspiration. PEEP can be increased above the intrinsic PEEP level to splint open the airways and improve CO2 removal and air trapping. In a patient with increased airway resistance, changing inspiratory cycle off so that more time is allowed during inspiration may cause turbulence and affect triggering or increase inspiratory work of breathing.

131
Q

According to existing literature, in approximately what percentage of patients with anaphylaxis can you expect a biphasic reaction?

a. 1%
b. 20%
c. 40%
d. 60%
e. 90%

A

b. 20%

Anaphylaxis refers to a systemic, immediate hypersensitivity reaction that is potentially life-threatening. Features of anaphylaxis are onset of signs and symptoms within seconds to minutes, multiple organ systems involvement, and involvement of systems distant from the site of exposure. Biphasic anaphylaxis is defined as a return of symptoms after initial remission, occurs in 5%-20% of cases.

132
Q

A patient in the catheterization lab has the measurements shown in the figure. Pressures are represented as mm Hg, oxygen saturation as percent saturation.

(Abbreviations: RA, right atrial; LV, left ventricular, CI, cardiac index; PCWP, pulmonary capillary wedge pressure)

Which of the following is the best interpretation of these data?

A. Ventricular septal defect

B. Pulmonary hypertension

C. Pulmonary stenosis

D. Left heart failure

E. Normal

A

Answer: C - Pulmonary stenosis

The right ventricular pressure is suprasystemic and there is a pressure gradient from the right ventricle to the pulmonary artery, which would be consistent with some obstruction to flow, such as pulmonary stenosis. Pulmonary hypertension would require a much higher pulmonary artery pressure. This patient’s pulmonary artery pressure is about one-fourth the systemic pressure, which is in the range of normal. Left ventricular failure with a normal cardiac index and wedge pressure is unlikely, as is a ventricular septal defect with relatively equal saturations from the right atrium to the pulmonary artery.

133
Q

A 2-year-old child status-post cardiac transplant is undergoing follow-up cardiac catheterization. His weight is 12 kg. His latest CBC shows WBC 6.5, Hb 14, platelets 250. His arterial saturation on FiO2 30% is 95% and mixed venous oxygen saturation is 65%. Assuming an oxygen consumption of 180 ml/min/m2, what will be his calculated cardiac output?

A - 2.5 lt/min

B - 3.0 lt/min

C - 3.5 lt/min

D - 4.0 lt/min

E - 4.5 lt/min

A

B - 3.0 lt/min

In the absence of any shunt, the pulmonary blood flow and systemic cardiac output are the same and may be measured as part of investigation of patients with impaired cardiac function—notably, as part of transplant assessment. One of the simplest methods of measuring cardiac output is by using Fick method. Fick principle requires estimation of oxygen consumption; assumed values based on age, sex, and heart rate are often substituted. In cardiac output estimation by Fick principle, the following apply: (1) Blood flow is calculated by measurement of the oxygen content of venous blood and hence estimating the difference between the 2, which represents the tissue oxygen utilization; and (2) If oxygen consumption is known, the blood flow is calculated by the simple equation which follows: Q = VO2/ A-V O2 difference (Where Q = blood flow in lts/min) Or Q = VO2/CaO2 - CvO2. CaO2 = 140 × 0.95 × 1.34 = 180 mL/lt. CvO2 = 140 × 0.65 × 1.34 = 120 ml/lt. Q = 180/60. Finally, Q = 3 lt/min.

134
Q

In Henoch Schönlein purpura (HSP) intussusception is the most common and best described abdominal complication. Which one of the following is the most common site of intussusception in this condition?

A - Ileoileal

B - Ileocolic

C - Colocaecal

D - Colocolic

E - There is no specificity of location in intussusception in HSP

A

A - Ileoileal

Intussusception is the most common and best described complication of HSP. The lead point is the edematous bowel wall secondary to the intramural vasculitis and hemorrhage. Unlike spontaneous intussusception, which is usually ileocolic, intussusception in HSP usually involves the entire small bowel. This distinction is important as contrast enema is not useful when pathology is above the ileocaecale valve.

136
Q

A 4-year-old, 22-kg boy with influenza pneumonia develops worsening respiratory failure and requires intubation and mechanical ventilation for severe hypoxemia. It’s noted that he is being ventilated in volume-control SIMV mode with a rate of 20 (which he is overbreathing to 22), a tidal volume of 220 ml (with resultant plateau pressures of 32-35), an end expiratory pressure of 10 cm H2O, and an FiO2 of 1. His recent ABG indicates a pH of 7.41, PaCO2 of 32, and PaO2 of 58. Which one of the following changes in management would most likely reduce his mortality on the basis of existing animal and adult clinical data on ARDS?

A - Transition to high-frequency oscillating ventilation

B - Increase in positive end-expiratory pressure to 16 to 20 cm H2O to optimize recruitment

C - Sedation and paralysis (if needed) to reduce his overbreathing and create mild hypercapnia

D - Extracorporeal membrane oxygenation

E - Reduction of tidal volumes to reduce plateau pressures

A

E - Reduction of tidal volumes to reduce plateau pressures

In a large randomized trial of adult ARDS patients, low tidal volume ventilation (6 ml/kg) has been proven to reduce mortality compared to higher tidal volumes (12 ml/kg). Additional animal and clinical studies have correlated ventilator induced lung injury with excessive tidal volumes and plateau pressures. No one mode of ventilation is known to be superior for management to another (e.g., HFOV is not better than conventional). Although a recent adult study did reveal a mortality benefit from 24-hour neuromuscular blockade, this was in conjunction with low tidal volume ventilation and not due to the effect of respiratory rate or CO2. Transfer to an ECMO center in the UK provided superior outcomes in a recent adult ARDS trial in the UK, but this result was present in both patients who received ECMO and those who did not; thus, bringing into question whether it was in fact additional management strategies, such as low tidal volume ventilation, which underlay the benefit.

138
Q

A 5-year-old child is admitted to the ICU for septic shock. He has an arterial line in dorsalis pedis artery to continuously monitor his blood pressure. Which one of the following considerations would lead to a higher systolic blood pressure?

a. Short tubing
b. Air bubble
c. Transducer higher than the catheter tip
d. Over damped system
e. Arterial line in femoral artery rather than in dorsalis pedis

A

e. Arterial line in femoral artery rather than in dorsalis pedis

Vascular pressures are recorded by fluid-filled plastic tubes that connect arterial catheter to the pressure transducers. Fluid-filled recording systems can produce artifacts that distort arterial pressure waveform. Failure to recognize recording system artifacts can lead to errors in interpretation. Under damped system: When frequency of an incoming signal approaches the resonant frequency of the system, the resident oscillation adds to the incoming signal to amplify it. This system is called under damped system. It results in higher systolic peaks. A longer connecting tubing leads to under damping. Over damping: Reduces the gain of system and is reported by an attenuated systolic peak and narrow pulse pressure air bubbles trapped in the system lead to poor damping. To accurately measure the pressure, the transducer must be at the same height as the catheter tip. If the catheter tip is higher than the transducer, the monitor reads higher than actual pressure due to the fluid pushing downstream. As pressure waves move towards periphery, the systolic pressure gradually increases. Systolic pressure can increase as much as 20 mmHg, from proximal aorta to radial or femoral artery.

139
Q

Pulmonary arterial wedge pressure should be measured at which of the following time points in the respiratory cycle?

A. End of expiration in a mechanically ventilated patient

B. End of inspiration in a mechanically ventilated patient

C. End of inspiration in a spontaneously ventilating patient

D. Midpoint between inspiration and expiration

A

Answer: A (End of expiration)

The pulmonary arterial wedge pressure should always be measured at the end of expiration because pleural pressure will be closest to zero at this time and therefore an accurate estimate of transmural pressure can be obtained. If pleural pressure is measured at the end of inspiration, then pleural pressure will either be negative if the patient is spontaneously breathing or positive if the patient is being ventilated with positive pressure ventilation.

140
Q

A 4-year-old boy was admitted to PICU 24-hours ago after motor vehicle accident for traumatic brain injury. He is intubated, sedated, paralyzed, and is on a ventilator. He has intracranial pressure (ICP) monitoring in place. He has received mannitol and 3% saline to control raised ICP. His temperature is 37.2 °C, heart rate is 120/min, and blood pressure is 80/40 mmHg with mean of 52 mmHg. His current ICP reading is 23 mmHg. His serum sodium is 160 mEq/L, glucose is 165 mg/dL and osmolality is 345 mOsm/L. Which one of the following statements regarding management of this patient is most accurate?

a. Mannitol should be used to treat intracranial hypertension in this patient instead of 3% saline
b. His ICP and cerebral perfusion pressures (CPP) are in the acceptable range
c. Prophylactic phenytoin may be used to reduce incidence of early post traumatic seizures
d. Cooling this patient to 32 °C will have favorable effect on his outcome
e. Decompressive craniectomy will not benefit this patient

A

c. Prophylactic phenytoin may be used to reduce incidence of early post traumatic seizures

This is a patient with acute severe traumatic brain injury who has intracranial hypertension and his CT scan is showing diffuse cerebral edema. Option A is incorrect because when serum osmolality is >320 mOsm/L, 3% saline is preferred over mannitol to treat intracranial hypertension. Option B is incorrect. This patient’s CPP (mean arterial pressure –ICP) is 29. ICP >20 mmHg and CPP

141
Q

A 5-month-old boy returns to the ICU after repair of tetralogy of Fallot. HR has progressively increased to 162/min, the right atrial pressure is 8 mm Hg, and BP is 68/42 mm Hg. He is currently on dopamine, 5 μg/kg/min, and milrinone, 0.5 μg/kg/min. What is the most appropriate intervention at this time?

A. Initiate an epinephrine infusion

B. Give a volume bolus of 10 mL/kg

C. Give a bolus of amiodarone over 30 to 60 minutes

D. Increase the milrinone infusion to 0.75 μg/kg/min.

A

Answer: B - Give a volume bolus of 10mL/kg

A patient after tetralogy of Fallot repair has restrictive right ventricular pathophysiology. The right ventricle is noncompliant and has significant diastolic dysfunction. This physiology often requires a filling pressure greater than 10 mm Hg and judicious volume resuscitation to improve cardiac output. Junctional ectopic tachycardia is a possibility, and an ECG or atrial electrogram can be obtained, but volume should be the first consideration. Milrinone infusion is efficacious in this type of patient because of its lusitropic properties. However, volume status should be optimized before the milrinone infusion is increased. If the patient does not respond to interventions, echocardiography can be used to assess right ventricular volume and rule out significant pericardial fluid collection.

142
Q

Which one of the following anatomic or physiologic terms explains why a previously-healthy 4 year old is at lower risk for atelectasis on conventional ventilation than a previously healthy neonate?

A - Dead space ventilation

B - Lambert’s channels

C - Persistent fetal circulation

D - Pores of Kohn

E - Inter-bronchiolar channels

A

D - Pores of Kohn

The adult lung contains anatomic channels that permit ventilation distal to an area of obstruction. Two channels have been identified in normal human lungs, interalveolar (pores of Kohn) and bronchoalveolar (Lambert’s channels). Inter-bronchiolar channels are not found in healthy lungs but develop during disease processes. Pores of Kohn appear as holes in alveolar walls in the first and second years of life. Lambert’s channels are found after 6 years of age. The absence of these collateral pathways places infants and children at risk for the development of atelectasis and resulting ventilation/perfusion inequality

144
Q

A 1-day-old, full-term neonate with a prenatal diagnosis of hypoplastic left heart syndrome (HLHS) is transferred to the ICU on a prostaglandin infusion. The patient is quite cyanotic, and the chest radiograph is consistent with pulmonary edema. Which of the following is the most likely cause of the patient’s clinical and radiographic presentation?

A. Significant tricuspid regurgitation

B. A closing or already closed ductus

C. A restrictive atrial septum and respiratory distress syndrome

D. Congenital pneumonia

A

Answer: C - A restrictive atrial septum and respiratory distress syndrome

Optimal preoperative management of infants with hypoplastic left heart syndrome (HLHS) requires maintaining relatively well-balanced pulmonary to systemic blood flow (Qp/Qs of ~1).

Overcirculation: A newborn with arterial oxygen saturation greater than 90% accompanied by tachypnea, hepatomegaly, cool extremities, oliguria, and metabolic acidosis has excessive pulmonary blood flow and inadequate systemic blood flow (Qp/Qs >1).

Decreased pulmonary blood flow is manifested clinically by hypoxemia with SaO2 less than 75%. In this patient, Qp/Qs is less than 1, which may be due to restricted flow across a small patent ductus arteriosus, increased pulmonary vascular resistance due to lung disease, or increased pulmonary artery pressure due to obstructed pulmonary venous return or a restrictive atrial septum.

In infants with HLHS and a severely restrictive atrial septum, heart size may be relatively normal; however, there is significant pulmonary venous congestion. Radiographic findings may be misinterpreted as lung disease, potentially leading to a delay in diagnosis. If the atrial septum is nonrestrictive, the chest radiograph will show pulmonary overcirculation with cardiomegaly. The right atrial border may be prominent with absence of the ascending aortic shadow.

145
Q

According to Poiseuille’s law, which one of the following statements is true if we wish to maintain a constant flow of gases?

A - Halving the tube radius will require a 16-time increase in driving pressure

B - Alveolar recruitment has its greatest effect on flow

C - Large jumps in driving pressure is needed to overcome airway narrowing

D - The driving pressure varies directly with the airway circumference

E - Resistance is proportional the length of the airway

A

A - Halving the tube radius will require a 16-time increase in driving pressure

Poiseuille’s law states that resistance is directly proportional to the length of the tube and the density of the liquid or gas. Resistance is inversely proportional to the radius of the tube to the fourth power causing a 16-time increase for a halving of the radius. Alveolar recruitment affects elastic changes and subsequent compliance but has little effect on resistance. When resistance increases, there is a pressure drop across that resistance—not an increase; it is not related to the airway circumference. Resistance is proportional to the length of the airway and density but it is not related to the constant gas flow.

146
Q

An 8-month-old infant is admitted with a history of poor feeding, lethargy, and diaphoresis. On examination, he is poorly perfused, tachycardic, and tachypneic. Echocardiogram shows moderately depressed right ventricular function and severely depressed left ventricular function. It is appropriate to intubate him in anticipation that mechanical ventilation will improve cardiac status by reducing work of breathing (ie, oxygen consumption) and lead to which of the following other changes?

A. Decreased right ventricular (RV) preload

B. Increased RV preload

C. Decreased left ventricular (LV) afterload

D. Increased LV preload

E. Increased RV afterload

A

Answer: C (Decreased LV afterload)

In addition to reducing a patient’s oxygen consumption/work of breathing, positive pressure ventilation assists the left ventricle by reducing its afterload. Positive pressure ventilation leads to an increase in mean intrathoracic pressure, a resultant decrease in left ventricular (LV) transmural pressure, and thus a reduction in LV afterload. In terms of the right side of the heart, positive pressure ventilation (through an increase in mean intrathoracic pressure) decreases right ventricular (RV) preload and may decrease RV output. One may speculate that decreasing RV preload may improve LV performance by decreasing its preload. However, any improvement in LV function by this mechanism is likely to be less significant than the improvement related to the reduction in LV afterload (ie, the decrease in LV afterload represents the more significant change in physiology). Overall, positive pressure ventilation reduces RV preload, may increase or decrease RV afterload depending on lung volume, has a variable effect on LV preload depending on the status of the RV (ventricular interdependence), and decreases LV afterload. The degree of these physiologic changes depends on the mean intrathoracic pressure generated (ie, ventilator settings), baseline ventricular function, and intravascular volume status.

147
Q

Hepatic encephalopathy is a potentially reversible form of cerebral dysfunction that occurs in both acute and chronic liver disease. Brain edema in hepatic encephalopathy has been attributed to which one of the following bioactive mediators?

a. GABA
b. Acetylcysteine
c. Dopamine
d. Glutamate
e. Serotonin

A

D - Glutamate

Acute liver failure often leads to an encephalopathy, which progresses from altered mental status to stupor and coma over a period of days. Progressive cerebral edema and increased ICP result in mortality rates as high as 50%-90%. Hyperammonemia plays a crucial role in the pathogenesis of cerebral edema in acute liver failure. Cerebral herniation is closely associated with plasma ammonia levels of more than 150 mmol/lt. Ammonia is detoxified in the brain by formation of glutamine mainly in the astrocytes. The osmotic effects of intracellular accumulation of glutamine in astrocytes is supposed to be a mechanism of cytotoxic cerebral edema in acute liver failure.

149
Q

The pressure-volume loop in the figure demonstrates pulmonary overdistension.

Pulmonary overdistension is defined as:

A. Cdyn/Cstat

B. Cstat/Ctotal

C. C20/Ctotal

D. C50/Ctotal >1.0

E. Cdyn/Ctotal >1.0

A

Answer: C (C20/Ctotal

Pulmonary overdistension occurs when the volume limit of regional lung units is reached, and these portions of the lung can no longer expand as pressure continues to rise. Such a situation is seen in a pressure-volume loop as a decrease in compliance or a flattening of the slope of the inspiratory limb during the latter portion of inspiration. Pulmonary overdistension is objectively defined as a decrease in compliance of the last 20% of the breath (C20) in comparison to total compliance. Thus, pulmonary overdistension is represented as C20/Ctotal

150
Q

A 6-year-old African American boy with newly diagnosed acute lymphoblastic leukemia is being transferred to pediatric ICU for close monitoring of tumor lysis syndrome. His white blood cell count is 150,000 per mm3. Which one of the following statements regarding the treatment of this patient’s condition is most accurate?

A - Alkalization of urine with systemic bicarbonate use increases the solubility of uric acid, xanthine, and hypoxanthine to promote excretion

B - Allopurinol is the drug of choice to prevent tumor lysis syndrome in this patient

C - This patient should be screened for G6PD deficiency before starting medication

D - Wait and watch approach is entirely appropriate for this patient

E - Rasburicase lowers uric acid by competitively inhibiting the enzyme xanthine oxidase

A

C - This patient should be screened for G6PD deficiency before starting medication

This is a patient who is at high risk for developing tumor lysis syndrome. In high-risk patients, rasburicase is the preferred drug to prevent tumor lysis syndrome. African American children have high incidence of G6PD and are at risk for hemolytic anemia with rasburicase treatment. Option A is incorrect, as xanthine and hypoxanthine solubility decrease with urinary alkanization. There is lack of evidence for use of bicarbonate in tumor lysis syndrome and hence alkalization of urine is not recommended in the treatment of tumor lysis syndrome. Option B is incorrect, as this is a high-risk patient, and for a high-risk patient, rasburicase is recommended unless there are contraindications for its use. Option C is the correct response. This patient can develop hemolytic anemia with rasburicase use if he has G6PD deficiency. Therefore, it is important to screen this patient for G6PD deficiency before starting rasburicase. Option D is incorrect because the wait and watch approach is appropriate only for low-risk patients. Option E is incorrect because rasburicase acts by oxidation of uric acid into allantoin.

151
Q

In determining brain death in a 2-year-old pediatric patient, which one of the following tests is considered to be required to confirm evidence of brain death?

A - Two neurologic examinations by different examiners confirming absence of neurologic function

B - Electroencephalogram

C - Nuclear cerebral blood flow study

D - During an apnea test, evidence of no respiratory effort after PaCO2 rises to greater than 80 mmHg

E - Doppler flow study of brain to document reverse diastolic flow

A

A - Two neurologic examinations by different examiners confirming absence of neurologic function

Brain death is the permanent absence of cerebral and brainstem functions. Recent recommendations have required that 2 neurologic exams be performed and must include an apnea test in both exams. However, the same examiner can do the apnea tests. Cerebral blood flow study and EEG are supplemental tests—but never replace the 2 neurologic exams. Doppler flow study with reversed diastolic flow can be associated with loss of forward flow similar to a cerebral blood flow study but has never gained wide acceptance. During the apnea test, the PaCO2 is only required to rise to a level of 20 above baseline or greater than 60 mmHg.

153
Q

Which one of the following options is a vasodilator in skeletal muscle?

A - Oxygen

B - Alkalosis

C - Hypothermia

D - Potassium

E - Adenosine

A

D - Potassium

The metabolic changes that produce vasodilation include, in most tissues, decrease in oxygen tension and pH. Increase in CO2 tension and osmolality also dilate the vessels (most pronounced in skin and brain). A rise in temperature exerts a direct vasodilator effect. Potassium accumulates locally in skeletal muscles and has a demonstrated dilator activity. Adenosine may play a vasodilator role in cardiac muscle but not in skeletal muscle

154
Q

A 14-year-old boy is admitted to PICU after cardiac catheterization as a part of workup for right heart failure. His catheterization results showed mean pulmonary artery pressure of 40 mmHg, capillary wedge pressure of 10 mmHg, pulmonary vascular resistance index of 3.5 wood units/m2. Both inhaled nitric oxide (iNO) of 20 parts per million and IV epoprostenol dropped his mean pulmonary artery pressures to 20 mmHg without any change in cardiac output. Which one of the following statements is most accurate regarding this patient’s condition?

A - Chronic iNO is well established treatment for children with this condition

B - Oral chronic calcium-channel blockade is efficacious and has favorable survival rate in children

C - This patient will most likely require heart-lung transplant in less than 5 years

D - Epoprostenol (prostacyclin) IV infusion is preferred treatment for this patient

E - Atrial septostomy should be performed early in the disease process of these patients

A

B - Oral chronic calcium-channel blockade is efficacious and has favorable survival rate in children

This patient has idiopathic reactive pulmonary hypertension. Children whose pulmonary artery pressure drops to >20% with epoprostenol, iNO, IV adenosine, or inhaled iloprost are considered to have reactive pulmonary hypertension. In children with reactive pulmonary hypertension, oral calcium-channel blockers are efficacious with a 10-year survival rate of >80%. Option A is incorrect because iNO is shown to be beneficial for short-term treatment of pulmonary hypertension. Its role for long-term management of pulmonary hypertension is still under investigation. Option B is the correct response. One, 5, and 10 year survival rates of children with reactive pulmonary hypertension with oral calcium-channel blockade are 97%, 97%, and 81% respectively. Option C is incorrect because this patient has a very good chance of 10-year survival; he is very unlikely to need lung transplantation within 5 years. Option D is incorrect because epoprostenol infusion is technically difficult to administer and is associated with serious complication and is not a first-line treatment for this patient. Option E is also incorrect because atrial septostomy is considered when other treatments have failed or unavailable for patients who are in need of palliative treatment.

155
Q

Describe how the shape of the oxyhemoglobin dissociation curve influences the uptake and delivery of oxygen. Define P50.

A

P50: is the PO2 where hemoglobin is 50% saturated; or where 2 of the 4 heme groups are bound to O2

  • An increase in P50 reflects a decrease in the affinity for O2; thus facilitating the unloading of O2 in tissues
  • A decrease in P50 reflects an increase in the affinity for O2; thus more difficult unloading of O2 in tissues
  • The sigmoidal shape of the oxygen hemoglobin dissociation curve implies that as more O2 binds to hemoglobin, the affinity of hemoglobin for O2 increases (positive cooperativity)
  • This means that in the lungs, where PO2 is 100 mm Hg, positive cooperativity leads to the highest hemoglobin affinity for O2, so oxygen is readily taken up by hgb
  • In peripheral tissues, where PO2 is around 40 mm Hg, hemoglobin has a lower affinity for O2, so it is readily released from hemoglobin and taken up by the tissues
156
Q

Which one of the following microorganism is the most commonly associated with ventilator associated pneumonia?

A - Pseudomonas

B - Enterococcus

C - Streptococcus pneumonia

D - Hemophilias

E - Klebsiella

A

A - Pseudomonas

Aspiration of pathologic organisms from the oropharynx is believed to be the inciting event in most cases of hospital acquired (nosocomial) and ICU acquired pneumonias. The organisms that most often colonize the oropharynx in hospitalized patients are enteric gram –ve bacilli (particularly pseudomonas) and staph aureus, which are implicated in most cases of Ventilator associated pneumonias.

157
Q

A 4-year-old boy is on high-frequency oscillatory ventilation (HFOV) for acute hypoxic respiratory failure following viral pneumonia. He is on day 7 of mechanical ventilation. His HFOV settings are mean air way pressure (MAP) of 36; FiO2 of 0.6; frequency of 5 Hz. His pulse oximeter saturation is reading 92%. Arterial blood gas shows pH of 7.30; PaO2 of 51 mmHg; PaCO2 of 52. He is on IV infusion of dopamine of 10 mcg/kg/min and epinephrine of 0.05 mcg/kg/min. Echocardiogram shows ejection fraction of 44%. You are called to the bedside for decreased pulse oximeter readings of 83%. Which one of the following options is the next the most appropriate action for this patient?

a. Venovenous extra corporeal membrane oxygenation (VV ECMO)
b. Venoarterial extra corporeal membrane oxygenation (VA ECMO)
c. iNO at 20 ppm
d. Intratracheal surfactant administration
e. Place the child on prone position

A

a. Venovenous extra corporeal membrane oxygenation (VV ECMO)

This is a patient with severe acute hypoxic respiratory failure with oxygenation index (OI) of 42 and on moderate inotropic support. OI of >40 in acute hypoxic respiratory failure is associated with >70% mortality. Key is early us of ECMO in these patients. Option A is the correct answer. This patient has OI of 42 and is not improving. His ionotropic score is 15 and he has reasonable cardiac function—thus, VV ECMO is most appropriate treatment for this patient. Option B is incorrect because given stable cardiac function and primary cause for hypoxia is respiratory illness VA ECMO is not warranted at this time. VA ECMO is associated with carotid artery ligation. Option C is incorrect because iNO has not shown to be beneficial in acute hypoxic respiratory failure. Option D is incorrect because surfactant administration is still investigational modality of treatment. Option E is incorrect because prone position may temporarily help but it will not alter the course of the disease.

158
Q

A 28-month-old white girl with precursor B-cell acute lymphoblastic leukemia had IV methotrexate 7 days ago. WBC count is 700/μL, hemoglobin level is 6 g/dL, and platelet count is less than 10,000/μL. She has a fever to 38.9°C (102°F), mild epistaxis, hypotension, and has just been transferred to the pediatric ICU. Which of the following types of packed red blood cells should this child receive?

A. Irradiated

B. Leukoreduced

C. Leukoreduced and irradiated

D. Normal

E. Volume-reduced

A

Answer: C - Leukoreduced and irradiated

Consent is required for the administration of blood products. Risks and benefits of blood administration should be explained to patient and family at that time. All hospitals have different guidelines for consent and these should be known to clinicians prior to writing orders for blood products. As for leukoreduction, the units that are cellular (packed red blood cells and platelets) contain white blood cells that can cause febrile, nonhemolytic transfusion reactions, human leukocyte antigen alloimmunization, and cytomegalovirus transmission; thus, leukoreduction in this setting is appropriate (really should be done for all children despite their immune status). In the immune-deficient patient, irradiation is required as leukoreduction alone will not stop transfusion-associated graft-versus-host disease (TA-GVHD). T-cell sterilization by irradiation will stop the proliferation of these cells. TA-GVHD is a rare complication of transfusion, but it is associated with greater than 90% mortality.

160
Q

Which of the following is the minimal amount of change from baseline in near-infrared spectroscopy cerebral regional oxygen saturation that needs further investigation or intervention?

A. 5%

B. 15%

C. 20%

D. 40%

A

Answer: C - 20%

Targets and thresholds are expressed in rSO2 numerical values and percentage changes from baseline. A change in the near-infrared spectroscopy cerebral regional oxygen saturation of 20% from baseline or an rSO2 less than 50 are common intervention triggers and are supported by outcome data.

162
Q

A 7-year-old boy, with relapsed ALL, is admitted to ICU with septic shock. He is on dopamine at 10 mcg/kg/min and epinephrine at 0.05 mcg/kg/min; his CVP is 4-5 cm of H2O and has been having decreased urine output. His serum albumin that morning is 2.1 mg/dl. You suspect sepsis induced capillary leak and resulting hypoalbuminemia. You decide to give him 1 g/kg, 25% albumin infusion over 3 hours. His weight is 25 kg. At the end of the infusion, which one of the following options do you expect the expansion of his intravascular volume would be?

A - 50 ml

B - 100 ml

C - 200 ml

D - 300 ml

E - 400 ml

A

E - 400 ml

Colloid fluids contain large molecules that do not readily move out of the vascular compartment. These molecules create an osmotic pressure called the colloid osmotic pressure (COP) that favors the retention of water in the vascular compartment. Colloid fluids differ in their ability to increase the plasma volume and this difference is a function of the COP of each fluid. Colloid fluids that have a COP that is much higher than the COP of plasma, draw interstitial fluid into the plasma. 5% albumin has a COP of 20 mm Hg (equivalent to plasma), it has 70% retention in plasma, while 25% albumin has a COP of 70 mm Hg, as a result 25% albumin draws fluid from the interstitial space, and the increment in plasma volume can be 4-5 times greater than the volume infused.

164
Q

A 12-year-old girl is struck by a car while riding her bicycle. She is transported to your ICU for evaluation. The primary survey reveals an intact airway, adequate respiratory effort, normal heart tones, and strong femoral pulses. The neurological assessment shows the ability to localize painful stimuli but failure to follow commands, spontaneous eye opening, and talking in sentences that do not make sense. The most appropriate Glasgow Coma Scale score for this patient is:

A. 7

B. 9

C. 11

D. 13

E. 15

A

Answer: D - 13

The Glasgow Coma Scale score is a tool designed to classify a patient’s level of consciousness at any given time. The patient’s eye, verbal, and motor responses are observed by the clinician and scored on a scale from 3 (completely comatose) to 15 (fully awake). Spontaneous movements and gestures are scored as a higher value than responses made after the clinician’s prompting. The eye movement category is scored from 1 to 4. A score of 4 is given for spontaneous eye opening, a score of 3 is given for eye opening after verbal stimuli from the clinician, a score of 2 is given for eye opening after painful stimuli is given, and a score of 1 is given for no eye opening. The verbal response category is scored on a 1 to 5 scale. A score of 5 is given for speech that is oriented to time and place, a score of 4 is given for confused speech but speaking in comprehensible words, a score of 3 is given for speech that consists only of inappropriate words, a score of 2 is given for speech that consists of incomprehensible sounds without words, and a score of 1 is given for no verbal response. The motor response is scored on a scale of 1 to 6. A score of 6 is given when the child can follow commands, a score of 5 is given when the child can localize painful stimuli, a score of 4 is given for a withdrawal response to painful stimuli, a score of 3 is given for abnormal flexion responses to painful stimuli, a score of 2 is given for abnormal extension responses to painful stimuli, and a score of 1 is given for no motor response. In the current vignette, the child scores 4 for eye opening, plus 4 for verbal response, plus 5 for motor response, equaling 13.

165
Q
  1. Which one of the following statements is true regarding inflicted abdominal trauma in children?
    a. Death rate of abdominal trauma victims of child abuse is 5%-10%
    b. All children with child abuse should be screened with abdominal CT scan
    c. Trauma is not a common cause of pancreatitis in children
    d. Child abuse consultants often screen for occult abdominal injury using hepatic transaminases
    e. Concomitant injuries are not commonly seen in children with inflicted abdominal injuries
A
166
Q

A patient receives an initial IV dose of a medication and serum concentration levels are measured. The drug has a volume of distribution of 0.5 L/kg and serum concentrations of 5 mg/dL are the target for therapy. What is the loading dose needed for a 10-kg child to achieve this target?

A. 25 mg

B. 50 mg

C. 250 mg

D. 500 mg

A

Answer: C - 250mg

The formula relating these variables is:

Change in Concentration = Dose/Volume of Distribution.

So, to change concentrations from 0 to 5 mg/dL in a 0.5 L/kg volume of distribution for a 10-kg child, the necessary dose would be:

(5 mg/dL) × (0.5 L/kg) × (10 kg) × (10 dL/L) = 250 mg loading dose or 25 mg/kg.

168
Q

Which one of the following statements is accurate regarding brain abscesses in children with congenital heart disease?

a. Increased risk of brain abscesses in conditions with left-to-right shunt
b. Degree of hypoxia has not been shown to have a direct correlation with the risk of brain abscesses
c. Among congenital heart diseases, the one most commonly associated with brain abscesses is hypoplastic left heart syndrome
d. Most commonly seen in distribution of posterior cerebral artery
e. Often occur in multiple intracerebral locations in the same patient

A

e. Often occur in multiple intracerebral locations in the same patient

Intracranial collection of pus is a serious and potentially life-threatening condition. Majority of cases of brain abscess currently occur in children with congenital heart disease. The overall annual incidence of brain abscesses in children with congenital heart disease has been estimated to be 0.45%. In their first 2 decades, patients with tetralogy of Fallot—the most common association with brain abscesses—have a risk of brain abscess of 12.1%. In congenital heart disease, the risk of brain abscess correlates with the degree of hypoxia. Children with congenital heart disease are at risk of developing microscopic areas of brain infarction due to serious hypoxemia, coupled with increased viscosity of polycythemic blood, in particular when reduced blood flow in brain microcirculation becomes critical during periods of dehydration or cardiac dysfunction. Episodes of low-grade bacteremia are common, as right-to-left shunting of blood bypasses the pulmonary capillary bed filter, and seeding of the devitalized areas establishes foci of cerebritis. In these conditions, abscesses are often multiple and may be located anywhere, although most commonly found in the distribution of middle cerebral artery.

169
Q

An 8-year-old boy with known history of asthma is admitted to the ICU for asthma exacerbation. On physical examination, in addition to his pulmonary findings of tachypnea and bilateral wheezing, you notice an abscess measuring 3 cm in diameter on his thigh. He has been afebrile and complains of tenderness at the site. You decide to incise and drain the abscess. Which one of the following statements best describes ketamine as the drug of choice for this procedure?

A - No effect on bronchial size or airway reflexes

B - Causes bronchoconstriction and protective airway reflexes lost

C - Causes bronchoconstriction and protective airway reflexes preserved

D - Causes bronchodilation and protective airway reflexes lost

E - Causes bronchodilation and protective airway reflexes preserved

A

E - Causes bronchodilation and protective airway reflexes preserved

Ketamine binds to the phencyclidine receptor in the N-methyl-D-aspartate channel. It causes dissociative amnesia and has analgesic properties. It is used for short, painful procedures in ICU setting. It has bronchodilatory properties and preserves the protective airway reflexes. Increased secretions are a potential side effect.

170
Q

Which one of the following statements is accurate regarding epidural hematomas?

a. Occurs from traumatic events with significant force, most often involving motor vehicles
b. Often associated with primary brain injury and carry a much worse prognosis than SD hematoma
c. On CT scan, they have a characteristic crescentic appearance
d. Two-thirds of epidural hematomas result from tear of middle meningeal artery
e. Due to risk of clot expansion, almost all epidural hematomas need surgical evacuation

A

d. Two-thirds of epidural hematomas result from tear of middle meningeal artery

Epidural hematoma occurs when a skull fracture or surface contact force traverses an epidural vessel with sufficient force to rapture the vessel. These injuries can occur from low height falls, especially in infants and young children whose skulls are thin and malleable. Because an arterial epidural hematoma can expand rapidly, a potentially life-threatening injury can occur from household falls and should be kept in mind when this mechanism is reported. Two-thirds of epidural hematomas result from tear of the middle meningeal artery, and thus cannot be managed without surgical evacuation and cauterization of this vessel. Epidural hematoma, when treated with immediate surgical evacuation, carries a relatively good prognosis with less than 10% mortality rate. Subdural hematoma – usually occur from traumatic events with significant force, most often involving motor vehicles. Acute subdural hematomas models are generally evacuated urgently, but because of large forces involved, are often associated with significant underlying primary brain injury. They carry much worse prognosis than epidural hematomas. In infants, the most common etiology of acute subdural hematoma is nonaccidental injury.

170
Q

How do pulmonary capillaries compare to systemic capillaries?

A

Compared to other capillaries, they have:

1) large diameters,
2) low resistance,
3) high flow rate and
4) low pressure. This low pressure is key as it prevents pulmonary edema by keeping osmotic pressure greater than hydrostatic pressure.

171
Q

Spinal muscular atrophy (SMA) is a common genetic disease affecting anterior horn cells of the spinal cord, leading to symmetric flaccid paralysis. Which one of the following options is the mode of inheritance of this condition?

a. Autosomal recessive
b. Autosomal dominant
c. X-linked recessive
d. X link dominant
e. Spontaneous mutation

A

A - Autosomal recessive

SMA is inherited as an autosomal recessive disease. It is caused by mutation of the survival motor neuron (SMN) gene in an unstable and genetically complex region of chromosome 5q13. There are 2 copies of the SMN gene within this region: SMN1 (which is closer to the telomere) and SMN2 (which is closer to the centromere). Mutations in the SMN1 gene are responsible for the vast majority of SMA cases.

172
Q

An infant has asplenia following a palliative procedure that included an aortopulmonary shunt and repair of total anomalous pulmonary venous connection. HR has gradually increased to 184/min despite adequate volume resuscitation. Adenosine failed to convert the tachycardia. Which of the following is the most likely diagnosis?

A. Ectopic atrial tachycardia

B. Ventricular tachycardia

C. Reentrant-type supraventricular tachycardia

D. Junctional ectopic tachycardia

E. Junctional ectopic tachycardia with aberrancy

A

Answer: A - Ectopic atrial tachycardia

Infants with asplenia and those who undergo repair of total anomalous pulmonary venous return have an increased incidence of ectopic atrial tachycardia. This condition has a characteristic acceleration with onset of tachycardia, and slowing may precede cessation. It also may be mistaken for sinus tachycardia because if its insidious nature.

Supraventricular tachycardia typically has an abrupt onset and would respond to adenosine, which is effective in converting typical reentrant-type supraventricular tachycardia to sinus rhythm. Junctional ectopic tachycardia is usually seen in infants who have had surgery that involves closure of a ventricular septal defect (such as tetralogy of Fallot or truncus arteriosus). Postoperative ventricular tachycardia is usually associated with myocardial ischemia.

173
Q

A 5-year-old child is status-post a cardiac transplant for dilated cardiomyopathy. Which one of the following statements is true regarding his heart rate and use of inotropic medications in postoperative period?

A - Transplanted hearts have a lower basal rate due to denervation

B - Dopamine has an accentuated inotropic effect on the transplanted heart

C - Atropine is ineffective for the treatment of bradycardia in immediate post-cardiac transplant

D - Stress and exercise induced heart rate increase remains preserved

E - Permanent pacing wires are very frequently required in transplanted hearts

A

C - Atropine is ineffective for the treatment of bradycardia in immediate post-cardiac transplant

Loss of cardiac innervation in heart transplant patients results in significant changes in patient’s ability to respond to inotropic medications. The donor sinoatrial (SA) node—which controls the rate of the transplanted heart—does not receive vagal stimulation and thus fires at a higher basal rate. The stress and exercise induced increase in heart rate are blunted relative to normal. There is decreased cardiac response to dopamine; however, epinephrine and NE tend to have an accentuated inotropic effect on a denervated heart. The vagolytic effects of atropine are ineffective for the treatment of bradycardia. Temporary pacing wires are frequently used in the early postoperative period to maintain an adequate heart rate and cardiac output. Permanent pacing is rarely required.

175
Q

A 16-year-old, previously healthy child was admitted to the ICU with concerns of surgical abdomen due to intestinal perforation. His physical examination is significant for tachycardia, abdominal tenderness in the right lower quadrant, and cold extremities. Hypovolemic shock is supected, and intravenous fluid bolus (20 ml/kg) is started and he is given 4 mg of morphine for pain. He remains tachycardic. The surgical team is ready for the patient, and they want to operate on him immediately. The anesthesiologist is told about his clinical condition and advised to hydrate him before induction of inhaled anesthesia. Which one of the following options explains the cardiovascular effects of inhaled halogenated alkane anesthetic agents?

A - Depressed cardiac output and hypotension

B - Increased cardiac output and hypotension

C - Depressed cardiac output and hypertension

D - Increased cardiac output and hypertension

E - No known cardiovascular effects

A

A - Depressed cardiac output and hypotension

Option A is correct because inhaled halogenated alkane anesthetic agents are well known to depress cardiac output and cause vasodilation. This leads to hypotension, which is accentuated in hypovolemic patients like the one in the vignette. Option B and D are incorrect, as inhaled anesthetic agents do not increase cardiac output. Option C is incorrect because depressed cardiac output causes hypotension and not hypertension. Option E is incorrect, as the cardiovascular effects of inhaled anesthetic agents are well documented.

177
Q

Which one of the following options is the rationale for the use of helium/oxygen (heliox) in patients with respiratory distress?

A - Heliox causes bronchodilation in children with severe respiratory distress

B - Antiinflammatory properties of heliox decrease the mucous production in the bronchioles

C - Helium and oxygen share atomic similarities, making helium a good substitute for oxygen

D - The low density of heliox allows greater laminarity of gas flow through high-resistance airways

E - Methemoglobinemia is only seen when high concentrations of helium are used, making it safe at lower concentrations

A

D - The low density of heliox allows greater laminarity of gas flow through high-resistance airways

Helium has an extremely low density when compared to air or oxygen. Gas flow through relatively a small cross-sectional area is usually turbulent. Gas flow across a large cross-sectional area is laminar. The lower density of helium improves gas flow through high-resistance airways. As gas flow becomes less turbulent in the affected airways, flow velocity is reduced, transitioning to laminar flow. Option D is correct as the low density of heliox allows this transition from turbulent flow to laminar flow—hence, increased delivery to distal airways. Option A is incorrect as heliox does not have any effect on the diameter of airways. Option B is incorrect because helium is a biologically inert gas. Option C is incorrect, as oxygen and helium do not have similar atomic structure. Option E is incorrect because helium does not cause methemoglobinemia at any concentrations.

178
Q

A 3-year-old male child is found playing with an open bottle of his grandfather’s pain medications (opioids). He is sleepy, so his mother rushes him to the ED, which is not far from her house. Upon arrival to the ED, he has perioral cyanosis and the arterial saturations are reading 88% on room air. The rest of his vital signs are within normal range except for a respiratory rate of 12 breaths per minute. His lung examination is normal and his neurological examination is significant for lethargy. Which one of the following options best describes his alveolar arterial oxygenation (A-a) gradient and arterial partial pressure of CO2 (PaCO2) levels?

A - Normal A-a gradient and increased PaCO2

B - Increased A-a gradient and increased PaCO2

C - Normal A-a gradient and decreased PaCO2

D - Increased A-a gradient and normal PaCO2

E - Normal A-a gradient and normal PaCO2

A

A - Normal A-a gradient and increased PaCO2

The patient in this scenario has ingested pain medication which has depressed his respiratory drive. Opioids are known to depress respiratory drive, leading to hypoventilation. Hypoventilation leads to an increase in PaCO2 (making options B, C, D, and E all incorrect). His SaO2 is low because of arterial hypoxemia. Arterial hypoxemia indicates limitation of pulmonary gas exchange. There are 4 reasons for limitations of pulmonary gas exchange: (1) hypoventilation; (2) shunts; (3) diffusion abnormalities; (4) ventilation/perfusion mismatch. The A-a gradient is increased in all conditions except for hypoventilation (making options B and D incorrect). Hence, this child will have a normal A-a gradient and increased PaCO2 (making option A the correct response option).

179
Q

A child involved in a high-speed motor vehicle collision has an initial evaluation with a Glasgow Coma Scale (GCS) score of 13. Five minutes after the initial neurological assessment, the child only opens eyes to painful stimuli, talks only in incomprehensible sounds, and demonstrates abnormal flexion responses to painful stimuli. The most appropriate next step is:

A. CT of the head followed by neurosurgical consultation

B. Intubation and rapid administration of hyperosmolar therapy

C. Neurosurgical consultation to determine if brain CT is indicated

D. Doing nothing further since there is no change in GCS

E. Mannitol infusion of 0.25 g/kg over 20 minutes

A

Answer: B - Intubation and rapid administration of hyperosmolar therapy

The rapid deterioration of a child’s mental status after traumatic injury indicates an evolving intracranial injury that is life threatening in character. This finding demands the prompt treatment of suspected intracranial hypertension in order to diminish the risk of cerebral herniation, making option B the correct answer. Therapies to lower intracranial pressure include immediate endotracheal intubation to stabilize the airway, removal of cerebrospinal fluid (if an intracranial pressure monitor were in place), controlled hyperventilation (PCO2 to 30-35 mm Hg), delivery of osmotic agents (mannitol, hypertonic saline), or administration of barbiturates. Option E is incorrect because low-dose mannitol infusion over a long period of time (20 minutes rather than IV push) is an insufficient response to the life-threatening clinical situation. While neurosurgical care will most likely be needed in cases like this, medical management of intracranial pressure can effectively stave off herniation until definitive surgical procedures can be performed, making option C incorrect. Option A represents the next steps after the initial lifesaving stabilization represented by option B.

180
Q

A 5-year-old child presents for deep sedation for an MRI for persistent headaches. During your pre-sedation evaluation, you are informed that the child has a family history of early cardiac death and has asthma which currently requires occasional doses of an inhaler. Which one of the following categorizations best describes the ASA physical status classification for this patient?

A - ASA class 1

B - ASA class 2

C - ASA class 3

D - ASA class 4

E - ASA class 5

A

B - ASA class 2

This child has a history of asthma which would be classified as single organ with no functional limitations. Therefore, it would be categorizes as an ASA class 2. Meanwhile, ASA class 1 refers to a healthy patient with no systemic disease. In regards to ASA class 3, the patient would have significant systemic disease. ASA class 4 patients have a major illness which is a constant threat to life. Finally, ASA class 5 refers to a patient who is moribund and may die with or without the procedure.

181
Q

A 15-year-old male is in the pediatric ICU with resolving traumatic brain injury. He has had resolution of his acute phase of injury that was managed with hyperosmolar therapy, intracranial pressure/cerebral perfusion pressure–targeted therapy, and barbiturate burst-suppression. He is now out of barbiturate burst-suppression and remains intubated awaiting neurologic recovery. He remains on total parenteral nutrition due to ischemic bowel injury and hollow viscus rupture at the time of his motor vehicle accident. His glucose infusion rate is 15 mg/kg/min and his calculated glucose infusion needs are 4 mg/kg/min. Which of the following most likely represents his respiratory quotient?

A. ≥1

B. 0.8

C. 0.5

D. ≤0.1

A

Answer: A (>1)

This patient’s glucose infusion rate far exceeds his energy needs as well as his glucose as an energy source. The recommended glucose infusion rate is 4-5 mg/kg/min. Any energy intake in excess of his energy expenditure needs, especially excess glucose, will be converted to lipid stores. This occurs through hepatic lipogenesis and is associated with increased carbon dioxide production. A respiratory quotient greater than or equal to 1 best represents the state of anabolism described here.

183
Q

A 60-kg (132 lbs), 10-year-old boy has a 1.5- × 2-cm mass in the suprasellar position presumed to be a craniopharyngioma. He is taken to the operating room and the tumor resected through a left temporal approach craniotomy. During the case he has an estimated blood loss of 50 mL, makes 1 mL/kg/h of urine, and is given 1,200 mL of normal saline. He returns to the ICU extubated and awake with no focal deficits on cranial nerve testing. Over the subsequent 8 hours he receives normal saline at 100 mL/h as his only fluid and makes urine at 250 mL/h. His morning laboratory values include a sodium level of 160 mEq/L. The most likely etiology of his hypernatremia is:

A. Administration of hypertonic fluids

B. Cerebral salt-wasting

C. Diabetes insipidus

D. Syndrome of inappropriate antidiuretic hormone

A

Answer: C - Diabetes insipidus

Patients with untreated central diabetes insipidus will have high urinary flows with excessive free water losses resulting in hypernatremia. This is due to a decreased excretion of antidiuretic hormone (vasopressin) from the hypothalamus, resulting in excessive free water urinary losses. This patient’s likely craniopharyngioma and subsequent resection put him at high risk for this condition. Option D is incorrect because patients with syndrome of inappropriate antidiuretic hormone secretion (SIADH) have low to no urine output and dilutional hyponatremia. Option A is incorrect because the patient was given isotonic (to normal values) fluids both intraoperatively and postoperatively. Option B is incorrect as patients with cerebral salt-wasting have hyponatremia in conjunction with high urinary flows. Although the exact physiologic etiology of this syndrome is unknown, it occurs with both cranial trauma and cranial surgery. Urine losses of sodium are very high and the patient develops hyponatremia through these sodium losses.

184
Q

Early food allergy to which one of the following foods is commonly outgrown?

A - Peanuts

B - Seafood

C - Tree nuts

D - Shellfish

E - Eggs

A

E - Eggs

This question describes a type of food that can cause an allergy early on in children, but which children often outgrow. Options A, B, C, and D are incorrect as food allergies to peanuts, seafood including shellfish, and tree nuts are not typically outgrown. Option E is correct, as early egg and milk allergies may be outgrown.

185
Q

When designing a study with a predictor variable which is dichotomous and an outcome variable which is continuous, which one of the following tests is best for estimating sample size?

A - Z statistic

B - Correlation coefficient

C - Chi square statistic

D - Spearman rank correlation

E - T test

A

E - T test

In estimating sample size in an analytical study or experiment, the investigator has to select the appropriate statistical test based on the type of predictor variable and outcome variable in the hypothesis. The t-test can be used when the predictor variable is dichotomous and outcome variable is continuous. It can also be used when the predictor variable is continuous and the outcome variable is dichotomous. In contrast, the z statistic is used to compare 2 groups that have dichotomous outcomes. The Chi square is the same as the z statistic for contingency. A correlation coefficient is used when both the predictor and outcomes are continuous. A Spearman rank correlation is only used when the predictor variable is not normally distributed and the outcome variable is continuous and normally distributed.

186
Q

A 17-year-old child has a lower extremity PICC line, with the tip ending above the diaphragm and an internal jugular line ending in superior vena cava (SVC). If the SVC oxygen saturation is 78% and IVC saturation is 70%, what would be his mixed venous saturation?

a. 78%
b. 76%
c. 74%
d. 72%
e. 70%

A

b. 76%

Mixed venous oxygen saturation (sat MV) was referred to the oxygen saturation of venous blood returning to heart; in practice, SVC saturation is often used, but a value intermediate between SVC and IVC may be preferred, as the 2 may be significantly different. The following formula is often used to calculate mixed venous oxygen saturation:Sat MV = 3 × Sat SVC + 1 × Sat IVC/4. This can also be simplified as follows: Sat MV = Sat SVC - (Sat SVC - Sat IVC/4). In regards to the above question, the correct calculations are as follows: Sat MV = 78 - (78-70/4); Sat MV = 78-2; and Sat MV = 76%.

187
Q

Which one of the following statement regarding phosphodiesterase inhibitors is accurate?

A - Milrinone inhibits PDE3, resulting in a decrease in cAMP in myocardial and vascular muscles

B - Milrinone has been shown to have antiinflammatory effects in septic shock

C - Milrinone is light sensitive and should be mixed in glucose

D - Milrinone is the inotrope of choice for severe left ventricular outflow obstruction

E - Dose-dependent thrombocytopenia is a common SE of amrinone

A

B - Milrinone has been shown to have antiinflammatory effects in septic shock

Amrinone and milrinone are excellent following cardiac surgery to augment inotropy and decrease systemic vascular resistance during periods of low cardiac output. Amrinone and milrinone are bipyridine derivatives which inhibit cyclic nucleotide phosphodiesterase III, resulting in an increased cAMP in myocardial and vascular muscle. Various animal experiments have documented antiinflammatory effects of milrinone in septic shock. Amrinone (not milrinone) is light sensitive and should not be mixed in glucose. Milrinone and amrinone should be avoided in patients with severe left and right heart outflow tract obstruction because of risk that fixed outflow obstruction in the presence of vasodilation may compromise coronary perfusion. Amrinone has been associated with thrombocytopenia. This is mediated by nonimmune peripheral platelet destruction, and there is no correlation between thrombocytopenia and amrinone dose, plasma concentration, or duration of administration.

188
Q

Which one of the following options is a risk factor for propofol infusion syndrome?

a. Old age
b. Bacterial infection/meningitis
c. High carbohydrate intake
d. Low fat intake
e. Concomitant steroid use

A

e. Concomitant steroid use

Propofol infusion syndrome associated with a high dose (>4 mg/kg/hour) and prolonged use (>48 hours). Risk factors are as follows: (1) young age; (2) critical illness; (3) high fat and low carb diet; (4) inborn errors of mitochondrial fatty acid oxidation; and (5) Concomitant catecholamine infusion or steroid therapy. Finally signs and symptoms include severe metabolic acidosis, CV collapse, rhabdomyolysis, hyperlipidemia, renal failure, and hepatomegaly.

189
Q

Which one of the following statements is true regarding errors in hypothesis testing?

A - Decreasing sample size decreases the chances of both type 1 and type 2 errors

B - Type 1 and type 2 errors are the same

C - Type 1 error occurs when the null hypothesis is incorrectly accepted

D - Type 2 error occurs when the null hypothesis is incorrectly rejected

E - Type 1 error occurs when the null hypothesis is incorrectly rejected

A

E - Type 1 error occurs when the null hypothesis is incorrectly rejected

Option E is correct because type 1 or alpha errors occur when the null hypothesis is incorrectly rejected, which means even though there was no difference between the treatment group and the placebo group, the study concluded that there was a difference. Option A is incorrect because the only way to decrease the chances of type 1 and type 2 errors is to increase the sample size. Option B is incorrect because type 1 and type 2 errors are different. Option C and D are incorrect because type 2 errors occur when the null hypothesis is incorrectly accepted.

191
Q

Which one of the following is a correct pairing of antimicrobial and adverse effect?

a. Aminoglycosides – apnea, respiratory depression
b. Amphotericin – hypotension due to histamine release
c. Vancomycin – Stevens-Johnson syndrome
d. Acyclovir – hyponatremia
e. Chloramphenicol – liver failure

A

a. Aminoglycosides – apnea, respiratory depression

Examples of clinically significant, unusual side effects associated with antiinfective agents:

192
Q

A 4-year-old boy was involved in a motor vehicle collision and transferred to your pediatric critical care unit. He was intubated at the emergency department for decreased level of consciousness (Glasgow Coma Scale score of 7). He received atropine, fentanyl, and succinylcholine for the rapid-sequence intubation. On examination, he has tachypnea (45/min), tachycardia (190/min), fever (40.3°C [104.5°F]), and generalized muscle rigidity and jaw stiffness. An arterial blood gas study shows: 7.12/60/80/-7

The best next step in the management of this patient is to:

A. Administer 2.5 mg/kg of dantrolene

B. Administer 0.1 mg/kg of lorazepam

C. Obtain a CT of the head

D. Start a 20-mL/kg bolus of lactated Ringer solution

E. Start maintenance IV fluids with 60 mEq/L of sodium bicarbonate

A

Answer: A - Administer dantrolene

The patient in this case had malignant hyperthermia (MH) syndrome. MH is a pharmacogenetic syndrome characterized by a hypermetabolic state of skeletal muscle, typically induced by exposure to specific anesthetic agents including all the halogenated volatile agents as well as succinylcholine. The syndrome typically presents with fever, profound acidosis, rhabdomyolysis, and, if untreated, death in the majority of cases.

In the past, agents known to trigger MH included ether, halothane, and enflurane. Currently, sevoflurane, isoflurane, desflurane, and succinylcholine are known to trigger episodes of MH. The clinical presentation of MH is highly variable and may at times be confused with other conditions such as rhabdomyolysis. The classic features of the disorder are nonspecific and can be found in association with many other conditions. The combination of a highly variable presentation with nonspecific signs and symptoms in a rare disorder explains the continued association of MH with poor outcomes, including death. It is believed that MH is associated with only a few conditions, including central core disease, Evans myopathy, and King–Denborough syndrome.

Successful management of an acute MH episode depends on early recognition and swift application of a treatment protocol that has been well thought out in advance of the need. Specific management includes discontinuation of triggering agents and institution of therapy with dantrolene. Nonspecific therapy is aimed at the effects of acidosis, fever, rhabdomyolysis, and consequent hyperkalemia, including correction of acidosis with hyperventilation and sodium bicarbonate; active cooling using cooling blankets, iced saline lavage, cooled fluids, etc; maintenance of urine output with mannitol and furosemide; and management of hyperkalemia with bicarbonate, glucose, and insulin. Additional therapies directed against dysrhythmia such as procainamide and amiodarone may be useful; however, rhythm disturbance is almost certainly secondary to hyperkalemia, and therapy should be primarily directed at reducing serum potassium.

194
Q

A 9-month-old girl was admitted to PICU with provisional diagnosis of meningitis. Patient was vomiting for 7 days with irritability and bulging fontanel. Patient is on breast feeding along with vitamin supplementation. Mother was giving 3 ml per day of vitamin supplementations instead of recommended 1 ml/day. On examination, child is awake and irritable. There were no fever spikes in the hospital. Anterior fontanel is open and bulging. There is no papilledema or retinal hemorrhages on funduscopic examination. Lumbar tap reveals opening pressure of 25. CSF analysis shows glucose of 65 mg/dL, protein of 60 mg/dl, and white cell count of 0. Serum glucose was 90. Gram-stain of CSF does not show any organism and bacterial culture results are still pending. MRI of brain of this child is normal. Which one of the following medications is useful to treat this condition?

a. Vancomycin and ceftriaxone
b. Dexamethasone
c. Furosemide
d. Acetazolamide
e. Enoxaparin subcutaneously

A

d. Acetazolamide

This is a child with pseudotumor cerebri probably from hypervitaminosis D. Option A is incorrect because child does not have any evidence of meningitis. Option B is incorrect because corticosteroids are not routinely administered unless patient has severe intracranial hypertension and is at risk of losing visual function or awaiting surgical decompression. Option C is incorrect because furosemide is not used to treat elevated intracranial pressure. Option D is correct response because acetazolamide 10-30 mg/kg/day is an effective regimen to treat pseudotumor cerebri. Option E is incorrect because enoxaparin is used in children with sinus venos-thrombosis and this patient does not have venous thrombosis.

195
Q

Verrucous endocarditis (Libman-Sacks endocarditis) describes vegetations seen in association with SLE. Which one of the following options is the most common site of these vegetations?

a. Subaortic vulvar
b. Aortic valve leaflets
c. Aortic valve roots
d. Mitral valve
e. Tricuspid valve

A

d. Mitral valve

The valvular vegetations are seen in association with endocarditis in SLE (also referred to as verrucous endocarditis or Libman-Sacks endocarditis). Most commonly these involve the mitral valve, but also the aortic, pulmonic, and tricuspid valves in order of decreasing incidence. The lesion consists of deposits of immune complexes, cellular debris, and fibrin.

196
Q

A 16-year-old male who works in his family’s jewelry manufacturing company was found unresponsive 30 minutes after returning home. BP is 40 mm Hg/palpable and HR is 100/min. Physical examination is remarkable for coma and diffuse rales. Arterial blood gas results in 100% oxygen show pH of 7.21, PaCO2 of 19 mm Hg, and PaO2 of 151 mm Hg. ECG demonstrates sinus bradycardia without prolongation of PR, QRS, or QTc intervals. Which of the following is the most likely toxin?

A. Amitriptyline

B. Cyanide

C. Diazepam

D. Isopropanol

E. Salicylates

A

Answer: B - Cyanide

Cyanide leads to hypoxic poisoning by inhibition of cytochrome oxidase, which halts oxidative phosphorylation. Sudden loss of consciousness, cardiorespiratory collapse, hyperoxia, and metabolic acidosis are key features. A history of possible cyanide exposure is helpful in the diagnosis. Exposure can occur with electroplating of metals, extraction of gold or silver, incomplete combustion during fires, plastic processing, almond extract preparation, and other occupational settings. A cyanide antidote kit is used as follows:

Amyl nitrite pearls as an immediate source of nitrite to induce methemoglobinemia 10% sodium nitrite IV
25% sodium thiosulfate IV to enhance conversion of cyanide to thiocyanate

Hydroxocabalamin has been used in Europe and use of hyperbaric oxygen is controversial.

Isopropanol and diazepam do not result in metabolic acidosis. Salicylates can cause altered mental status, pulmonary edema, and metabolic acidosis, but severe hypotension is uncommon. The short time course and metabolic acidosis are not characteristic of cyclic antidepressants, especially with normal intervals on ECG.

197
Q

Which of the following drugs is metabolized primarily by plasma pseudocholinesterase?

A. Cisatracurium

B. Pancuronium

C. Succinylcholine

D. Vecuronium

A

Answer: C - Succinylcholine

Cisatracurium is broken down by Hofmann degradation; it is least affected by organ dysfunction and more affected by pH and temperature.

Pancuronium is primarily metabolized in the kidneys and excreted in the urine; therefore, it is more likely to build up in patients with renal failure.

Vecuronium is primarily metabolized in the liver and will be metabolized more slowly in patients with hepatic failure.

Succinlycholine is hydrolyzed in the plasma by pseudocholinesterase.

198
Q

A newborn is transferred to PICU for perinatal diagnosis of truncus arteriosus. He has low-set ears, downturned eyes, micrognathia, and widely-spaced eyes. His ionized calcium is 1.00 and no thymus is visible on CXR. Which one of the following options is the most likely chromosomal abnormality associated with his condition?

a. Ch 22q11.2 deletion
b. Ch 22q11.1 deletion
c. Ch 22q22.2 deletion
d. Ch 22q11.2 translocation
e. Ch 22q22.2 translocation

A

a. Ch 22q11.2 deletion

The constellations of symptoms that define DiGeorge syndrome include congenital T-cell immune deficiency, conotruncal cardiac anomaly, hypocalcemic tetany, unusual facies, and hypoplastic thymus gland. The association of DiGeorge syndrome with chromosome 22 defects was first defined in 1981. The hemizygous chromosome 22q11.2 microdeletion is responsible is many but not all of the manifestation in affected children.

199
Q

A 3-month-old infant with tetralogy of Fallot and severe subpulmonary stenosis is admitted to the ICU with a hypercyanotic episode. He is awake and crying and has an oxygen saturation of 53%. Which of the following is the most appropriate medication for these cyanotic spells?

A. Phenylephrine IV

B. Milrinone IV

C. Isoproterenol

D. Sodium bicarbonate IV

A

Answer: A - Phenylephrine IV

A hypercyanotic (“Tet”) spell in patients who have tetralogy of Fallot (TOF) is due to increased right-to-left shunting that occurs with either increased pulmonary vascular resistance (PVR) or decreased systemic vascular resistance (SVR). Patients with TOF have a harsh systolic ejection murmur due to pulmonary stenosis; during a hypercyanotic episode, the murmur is absent or much softer than baseline. Patients may present with agitation or irritability, hyperpnea, profound cyanosis, and syncope.

Treatment focuses on decreasing PVR and increasing SVR. Oxygen is of limited benefit but can improve oxygenation by increasing alveolar oxygen concentration and potentially decreasing PVR and improving pulmonary blood flow. Sedation may also decrease PVR, particularly if the child is agitated. Sedation will also decrease tissue metabolic demands. Volume expansion with saline solution or packed red blood cells can increase overall cardiac output, which can improve flow through the narrowed right ventricular outflow tract and thus improve systemic oxygenation. Packed cells will also increase oxygen-carrying capacity, which is important in patients with severe hypoxemia. Phenylephrine is an alpha-agonist, which will increase SVR and force more flow into the pulmonary circulation. A beta-blocker such as esmolol can be used to decrease heart rate and possibly decrease the right ventricular outflow tract narrowing.

200
Q

A 7-year-old boy, who recently immigrated to the US, has been diagnosed with coarctation of aorta (CoA). In the cardiology clinic, he is noted to be hypertensive, and is admitted to the ICU for control of his blood pressure prior to the surgery. Which one of the following antihypertensive medication is the most appropriate for this patient?

a. Nipride
b. Hydralazine
c. Nifedipine
d. Furosemide
e. Propranolol

A

e. Propranolol

Preexisting hypertension occurs in many patients with CoA. All such patients should receive beta-blockers to control hypertension prior to surgery. Failure to provide preoperative beta-blockade may lead to marked hypertensive responses to intubation. Vasodilators are contraindicated before surgery in a hypertensive child with CoA. Left ventricular function may not increase cardiac output sufficiently to compensate for vasodilation because of relatively fixed obstruction from CoA. Tachycardia associated with vasodilation may also lead to myocardial ischemia.

201
Q

A 14-month-old girl with type I spinal muscular atrophy is in the PICU with respiratory syncytial virus (RSV) bronchiolitis, which presented as fever and copious secretions. She has an RR of 56 with deep subcostal retractions and is started on noninvasive positive-pressure ventilation with inspiratory pressure of 15 cm H2O and expiratory pressure of 5 cm H2O. Several hours later, her retractions are much improved and her RR is 30; however, her saturations are only 92% on 40% oxygen and her chest radiograph reveals bibasilar atelectasis. Which one of the following treatment plans will best recruit her collapsed lung?

A - Change to high flow nasal cannulae at 3 L /min

B - Increase the expiratory pressures to 10 cm H2O and the inspiratory pressures to 20 cm H2O

C - Glycopyrrolate or scopolamine

D - Chest physiotherapy with mechanical insufflation-exsufflation

E - Intubation and mechanical ventilation

A

D - Chest physiotherapy with mechanical insufflation-exsufflation

This child with neuromuscular weakness and copious secretions will need frequent breaks from the ventilator to remove these secretions. She will also need assistance expelling these secretions, which can be provided by chest physiotherapy and cough assist. Use of these techniques improves strength of cough and helps resolve atelectasis in clinical trials. Increase in expiratory and inspiratory pressures may temporarily help, but ultimately will fail due to the lack of secretion clearance. The use of mechanical ventilation is not indicated this early on and would place the child at risk for numerous subsequent adverse events.

202
Q

A 3-year-old has been on venoarterial extracorporeal membrane oxygenation (ECMO) through the cervical vessels for 4 hours for severe status asthmaticus. She is doing well until the venous return alarm on the ECMO circuit begins to ring. She is crying and the venous line does not appear to be kinked. Her blood pressure, heart rate, and perfusion are normal. The most likely cause of the alarm is:

A. Hypervolemia after receiving blood

B. Hypovolemia related to capillary leak syndrome

C. Increased intrathoracic pressure with crying

D. Pericardial tamponade

A

Answer: C (Increased intrathoracic pressure with crying)

The Valsalva movement of crying will increase intrathoracic pressures and potentially reduce venous return to the right heart. In the scenario of high-flow venoarterial extracorporeal membrane oxygenation, reduced venous return to the heart will reduce venous return to the circuit and create an alarm.

203
Q

Which one of the following statements regarding mechanism of drug action/receptor is most accurate?

A - Pharmacologic effect of sildenafil result from selective inhibition of phosphodiesterase III enzyme

B - ß2 receptors are absent in the heart and have no role in myocardial function

C - Phosphorylation of phospholamban by protein kinase A cause’s lusitropy in myocardium

D - Drug induced thrombocytopenia is a major limiting factor for milrinone use in children

E - Nesiritide increases cardiac index in cardiac failure patients through its direct ionotropic effect

A

C - Phosphorylation of phospholamban by protein kinase A cause’s lusitropy in myocardium

Option A is incorrect because sildenafil is a selective inhibitor of phosphodiesterase V enzyme. Option B is incorrect. ß2 receptors are present in the heart and couple with both Gs and Gi proteins. However, response to ß2 activation in the heart is controversial. Option C is correct. One of the actions of ß1-adrenergic receptors in the heart is activation of protein kinase A (PKA). PKA phosphorylates phospholamban, relieving the disinhibitory effect of the un-phosphorylated form on calcium channels in the sarcoplasmic reticulum. The accumulation of calcium by the sarcoplasmic reticulum is thus enhanced increasing the rate of sarcomere relaxation (lusitropy). Option D is incorrect because thrombocytopenia is not associated with milrinone. Option E is incorrect because increase in cardiac index by nesiritide is due to reduction in both preload and afterload.

205
Q

A 5-year-old boy with myasthenia gravis is admitted to ICU for pneumonia. Which one of the following antibiotics is the safest for treating his condition?

A - Amikacin

B - Ciprofloxacin

C - Ampicillin

D - Clindamycin

E - Vancomycin

A

E - Vancomycin

Clinician must always be cautious when initiating medications in the patient with myasthenia gravis. Many drugs interfere with the neuromuscular junction; the best-known are the aminoglycoside medications. Other antibiotics that have been implicated in the worsening of myasthenic symptoms include ampicillin, ciprofloxacin, clindamycin, erythromycin, sulfonamide, tetracycline, and the peptide antibiotics (polymyxin A and B and colistin).

206
Q

Describe the changes in pulmonary vascular resistance when the pressures in pulmonary arteries and pulmonary veins increase.

A

As pulmonary arterial/venous pressure increases, pulmonary vascular resistance DECREASES to preserve blood flow

Mechanism:
At baseline: some, but not all capillaries are open to receive blood flow
When pulmonary vascular pressure increases…
closed capillaries open for conduction (recruitment)
MOST IMPORTANT IN REDUCING RESISTANCE
Think back to LO4, the relationship that closes capillaries shifts with increased arterial or venous pressure
individual capillaries increased in caliber (distension)
helps decreases resistance after all capillaries are opened (aka very high pressures)

207
Q

The internal diameter of the trachea of an infant is 8 mm. As a result of airway edema, consequent to viral infection (croup), the diameter is now reduced to 4 mm. Which one of the following options will be its effect on the resistance to airflow in the airways?

a. Remain the same
b. Double
c. Increase 4 times
d. Increase by 8 times
e. Increase by 16 times

A

E - Increase by 16 times

The relation between flow of air through a tube and radius of the tube was described by French physician Poiseuille. In straight circular tubes, the volume flow rate is given by the following equation: V = Pπr4 / 8 ηL. Where P is driving pressure, r = radius, η = viscosity, and L = length. Because flow resistance is driving pressure divided by flow, the equation can be rearranged as the following: R= 8 ηL / πr4. Tube radius is of critical importance, if the radius is halved, the resistance increases 16 fold. However, doubling the length only doubles the resistance.

208
Q

Which one of the following diuretics has the most proximal tubular site of action?

A - Acetazolamide

B - Furosemide

C - Mannitol

D - Bumetanide

E - Amiloride

A

C - Mannitol

Loop diuretics affect either the proximal part or distal part of the loop of Henley. Furosemide and bumetanide work at the distal part of the loop as long as sodium is available at the site of their action. Acetazolamide is a carbonic anhydrase inhibitor which works in the tubule; however, it is not considered a diuretic. Finally, Amiloride is a potassium-sparing diuretic which works in the late distal convoluted tubule.

209
Q

Which one of the following is a correct statement regarding C. difficile infection?

A - Stool culture although rarely done are the gold standard for diagnosis

B - C. difficile toxin assay is highly sensitive but has poor specificity

C - CT abdomen shows marked edema of bowel wall of small intestine

D - Bloody diarrhea is seen in only 5%-10% of cases

E - Almost all patients who harbor C. difficile in their stools are symptomatic

A

D - Bloody diarrhea is seen in only 5%-10% of cases

C. difficile is a spore-forming, gram +ve anaerobic bacillus, which is not a prominent bowel inhabitant in healthy subjects, but proliferates when the normal bowel flora of the lower GI tract is altered by antibiotic therapy. It is not an invasive organism, but it elaborates cytotoxins that incite inflammation in the bowel mucosa. Severe cases of mucosal inflammation are accompanied by raised plaque-like lesions on the mucosal surface called pseudomembranes. Although C. difficile is found in less than 0.5% of healthy adults in the community, it can be seen in as many as 40% of the hospitalized patients. More than half of patients who harbor C. difficile in their stools are asymptomatic. The most common manifestation of symptomatic C. difficile infection is fever, abdominal pain, and watery diarrhea. Bloody diarrhea is seen in 5%-10% of cases. Stool culture for C. difficile is unreliable as they do not distinguish toxigenic from non-toxigenic strains of the organisms. Most lab use an ELISA method to detect the cytotoxins. The sensitivity of this test is 85% for one stool sample and 92% for 2 stool samples. Specificity is 98%. CT abdomen shows marked thickening of the wall of colon; however, small bowel is not affected. These findings are characteristic for inflammatory process involving the large bowel (an enterocolitis), but they are not specific for C. difficile enterocolitis

210
Q

A 1-week-old neonate is postop day 2 from coarctation of aorta repair. The bedside nurse calls you because the child looks mottled and has decreased urine output for the last hour. Vital signs are heart rate of 165 beats per minute, respiratory rate of 34/ minute, mean arterial pressure (MAP) of 45 mmHg, Sa02 of 100 % on room air, and he is afebrile. His physical examination is significant for mottling of skin, well healing incision site on chest, strong bilateral pulses, normal cardiac exam, and pale conjunctiva. The nurse had sent a recent mixed venous oxygen saturation (SvO2), which is 55%—a drop of 20 points from the previous one. Which one of the following options is most consistent with this clinical picture?

A - Congenital mixing cardiac lesion which was missed on the initial echocardiogram

B - Postoperative cardiac failure due to unsuccessful repair

C - Hypoventilation due to pain medication

D - Multiple blood draws leading to decreased Hb concentration

E - Septic shock due to urinary tract infection

A

D - Multiple blood draws leading to decreased Hb concentration

This clinical scenario is based on the concept of mixed venous oxygen saturation (SvO2) and factors affecting it. SvO2 is used to measure the balance between oxygen demand and supply. Conditions decreasing the SvO2 are conditions decreasing oxygen delivery. Anemia, decreased cardiac output, and hypoxemia are some of the common causes of decreasing oxygen delivery and hence SvO2. Option D is correct, as this patient has signs of anemia. Congenital mixing cardiac lesion is unlikely because of normal examination, normal saturations, and very sensitive echocardiograms (option A). Option B is incorrect because the child has normal cardiac examination, normal blood pressure, and bilateral equal pulses making unsuccessful repair unlikely. Option C is incorrect, as there are no indications of hypoventilation. Sepsis usually causes increase in SvO2 because of mitochondrial failure of uptake of oxygen. This patient does not have any signs of sepsis or septic shock (option E).

212
Q

Which one of the following options represents the mechanism of action of thiazide diuretics?

a. Inhibit Na+ K+ 2 Cl- co-transporter in the medullary thick ascending limb of loop of Henle
b. Inhibit Na+ K+ exchange in collecting duct
c. Inhibit Na+ Cl- co-transport in early portion of the distal tubule
d. Decreases tubular reabsorption of sodium and increases GFR
e. Decreases H+ secretion with resultant increase in Na+ and K+ excretion

A

c. Inhibit Na+ Cl- co-transport in early portion of the distal tubule

Mechanism of action of various diuretics:

213
Q

A 15-year-old boy is admitted to the ICU for respiratory distress. His vital signs are temperature of 102.2 °F, respiratory rate of 35/min, heart rate 110/min, blood pressure of 124/70 mmHg, and arterial oxygen saturation (SaO2) of 88% on room air. His physical examination is significant for moderate distress, and crackles in the right base. His chest radiograph shows right lower lobe pneumonia with effusion. His initial CBC is significant for elevated WBC count and anemia with Hb of 6.2 g/dl. Along with preparing for intubation, it is decided to transfuse packed red blood cells (PRBC). The pediatric resident wants to know the rationale for transfusion. Which one of the following options is the best rationale for transfusion?

A - PRBC transfusion will improve the SaO2 and improve his outcome

B - PRBC transfusion will improve the SaO2 and decrease his respiratory distress

C - PRBC transfusion will improve his arterial oxygen content and improve tissue oxygen delivery

D - PRBC transfusion will improve his arterial oxygen content and prevent acute respiratory distress syndrome (ARDS)

E - PRBC transfusion will improve his outcome, as it will prevent his progression to ARDS

A

C - PRBC transfusion will improve his arterial oxygen content and improve tissue oxygen delivery

The scenario in this vignette elucidates the concept of oxygen carrying capacity. The arterial oxygen content (CaO2) is product of Hb (g/100ml), arterial oxygen saturation (SaO2), and Hb oxygen carrying capacity (1.39ml O2/g Hb).CaO2 = Hb × SaO2 × 1.39. PRBC transfusion is going to increase the Hb, and thus increase the arterial oxygen content. Improving the arterial oxygen content will improve oxygen delivery to the tissues. As such, option C is correct. Options A and B are incorrect because PRBC transfusion will not increase the SaO2. PRBC transfusion has not shown to improve outcomes in pneumonia or ARDS; there is no evidence it will prevent ARDS, either (making options D and E incorrect).

214
Q

The blood smear indicates 15% parasitemia indicative of infection with P. falciparum and laboratory testing reveals thrombocytopenia, elevated bilirubin, and elevated creatinine. Which one of the following treatment regimens should be initiated?

A - IV quinidine and clindamycin in the PICU

B - Oral quinidine and clindamycin

C - Oral chloroquine

D - Oral chloroquine plus primaquine

E - Oral primaquine

A

A - IV quinidine and clindamycin in the PICU

This question describes the best treatment option for a pediatric patient with malaria caused by P. falciparum acquired in sub-Saharan Africa. Option A is correct because the patient’s disease burden is high, which requires treatment with quinidine or quinine in addition to an antibiotic such as clindamycin or doxycycline. The patient should be admitted to the PICU for observation during initial treatment, as quinidine can prolong QTc causing dysrhythmias, as well as asymptomatic hypoglycemia and respiratory distress. Option B is incorrect because initial therapy should include intravenous administration of quinidine and clindamycin due to the severity of the disease, and the patient should be monitored in the hospital. Options C and D are incorrect because chloroquine is not indicated in this case because the patient acquired the P. falciparum infection in an area endemic for chloroquine-resistance. Options D and E are incorrect because primaquine is not required for P. falciparum infection; it is used to treat malaria caused by P. vivax and P. ovale as primaquine eradicates the dormant phase of these parasites within the liver.

215
Q

Which one of the following statements is accurate regarding the pressure generated at the inlet of the ventilator in an intubated spontaneously breathing pediatric patient being supported on mechanical ventilation?

A - Affected by the pressure generated by the patients muscular effort

B - Affected by the product of the flow generated by the patient and the volume

C - Affected by the product of the patient’s pulmonary elastance and the flow generated by the patient

D - Independent of the level of intrinsic PEEP

E - Independent of the pressure generated by the ventilator

A

A - Affected by the pressure generated by the patients muscular effort

Newton’s equation of motion dictates the relationship between pressure generated at the airway and what factors affect that pressure. With a spontaneously breathing patient, the pressure generated is the sum of the pressure generated by the patient and that generated by the ventilator. Those pressures are affected by the resistive and elastic nature of the system. Resistance of the system is the determined by the product of the flow and total resistance of the respiratory system. Elastic forces of the system are determined by the product of volume and elastance of the respiratory system. The final factor is the level of intrinsic PEEP will affect the level of pressure measured at the inlet.

216
Q

A 6-year-old child with spastic quadriplegia has been brought to the ED by his parents for vomiting/diarrhea for 3 days and decreased PO intake. His weight is 16 kg, while his weight in his pediatrician’s office 2 weeks ago was 18 kg. His admission serum chemistries reveal sodium of 160 mEq/lt, potassium of 4.0 mEq/lt, chloride of 124 mEq/lt, bicarbonate of 20 mEq/lt, BUN of 60 mg/dl,

a. 1000 ML
b. 1500 ML
c. 2000 ML
d. 2500 ML
e. 3000 ML

A

a. 1000 ML

Free water deficit is calculated by the following equation: H2O deficit (in lt)= (estimated body water fraction)× (current weight in kgs)× (plasma sodium/normal plasma sodium-1). H2O deficit= 0.6×16×160/145-1. H2O deficit= 9.6× 15/145. Finally, H2O deficit= 144/145= 1

217
Q

You were called to see a child in the postoperative recovery room, who just arrived s/p appendectomy for a perforated appendix. The nurse has noted him to be tachypneic, tachycardic, hypotensive with mottling and diaphoreses. His blood gas is 7.1/60/80/-8 with lactate of 10. He has severe muscle rigidity and is febrile to a temperature of 41 °C. Review of his chart revealed that he was given sevoflurane and SCh for anesthesia. Which one of the following options is the mechanism of action of the most common drug used to treat this condition?

a. It prolongs the voltage sensitive neuronal sodium channels and increase refractory period of neurons
b. Causes GABAa receptor, mediated synaptic inhibition
c. It binds to ryanodine receptors and inhibits sarcoplasmic reticulum calcium release
d. Causes augmentation of release of inhibitory transmitter GABA by inhibiting its degradation
e. Blocks 5-HT 2A receptors resulting in inhibition of K+ channels and slow depolarization of neurons

A

c. It binds to ryanodine receptors and inhibits sarcoplasmic reticulum calcium release

Malignant hyperthermia is a disease associated with certain anesthetic agents, including inhalational anesthesia such as halothane, sevoflurane, and depolarizing neuromuscular blockers such as succinylcholine. Malignant hyperthermia has a variable clinical presentation including tachypnea, tachycardia, blood pressure abnormalities, cyanosis, mottling, and diaphoresis. Hypoxia, hypercapnia, metabolic acidosis, and muscular rigidity resulting in serious rhabdomyolysis can also occur. Treatment of malignant hyperthermia includes immediate discontinuation of offending agent, hyperventilation with 100% oxygen, and administration of dantrolene. Dantrolene interacts with amino acids 590-609 in the N-terminal fragment of the RYR1. By decreasing calcium release from the sarcoplasmic reticulum through RYR1, dantrolene decreases excitation contraction coupling. It does not act on the neuromuscular junction or on the passive or active electrical properties of the surface membranes of muscle fibers.

218
Q

In the setting of shock and cardiovascular compromise, intubation and mechanical ventilation assists with which of the following aspects of physiology/pathophysiology

A. Increased left ventricular afterload

B. Increased left ventricular preload

C. Reduced systemic vascular resistance

D. Reduced VO2

A

Answer: D (Reduced VO2)

Work of breathing may increase cardiac output demands and VO2 of the respiratory apparatus by 40%. Mechanical ventilation (positive pleural pressures) may reduce left ventricular afterload and reduce right ventricular preload (hence left ventricular preload) and does not affect systemic vascular resistance.

219
Q

You are intubating a previously healthy 4 year old for respiratory failure secondary to pneumonia. The best estimate of the appropriate depth of endotracheal tube insertion which one of the following values?

A - 10 cm

B - 12 cm

C - 14 cm

D - 16 cm

E - 18 cm

A

C - 14 cm

The depth of ET tube insertion should be determined (in a normal sized child) based on either age or size of endotracheal tube. If basing on age, the following equation may be used: depth of insertion = 12 + (age in years/2). One may also determine approximate appropriate depth of insertion by multiplying ET tube diameter by 3. In a 4 year old, a 4.5 or 5.0 mm ET tube would likely be used, resulting in an insertion depth of 15 cm. Answer C (14 cm) is the best available selection.

220
Q

Which one of the following antibiotics has a concentration dependent antibacterial action?

A. Penicillin

b. Linezolid
c. Macrolides
d. Clindamycin
e. Ciprofloxacin

A

e. Ciprofloxacin

Antibiotics, currently in use, can be sorted into 2 groups based on their pharmacologic interactions with bacteria; they are either time-dependent or concentration dependent killers. The activity of the first group (beta lactams, linezolid, macrolides, clindamycin) is determined by the time that the antibiotic concentration at the site of the infection exceeds the MIC of the infecting bacteria (denoted as T>MIC). For the second group (aminoglycosides, fluoroquinolones, metronidazole), the functional activity is driven by the relationship between the concentration achieved at the site of the infection relative to the MIC.

222
Q

A 10-month-old, 10-kg boy with pneumonia required intubation for respiratory failure 2 days ago. Currently, he is in volume control ventilation, synchronized intermittent mandatory ventilation, with rate of 30/min, tidal volume of 120 mL, FIO2 of 0.95, positive end-expiratory pressure (PEEP) of 5 cm H2O, and inspiratory time of 1.0 second. He is reaching peak inspiratory pressures of 35 cm H2O. His most recent arterial blood gas results are pH of 7.34, PaCO2 of 50 mm Hg, PaO2 of 52 mm Hg, and oxygen saturation of 90%. To improve oxygenation in this patient, which of the following interventions is most appropriate?

A. Decrease rate to 20/min.

B. Increase inspiratory time to 2 seconds

C. Increase PEEP to 10 cm H2O.

D. Reduce tidal volume to 60 mL.

E. Transition to pressure control ventilation.

A

Answer: C (Increase PEEP to 10 cm H2O)

Although the desire to promote lung-protective ventilation (low tidal volumes, low peak inspiratory pressure [PIP], and permissive hypercapnia) is correct, those changes would not necessarily immediately improve oxygenation. To improve oxygenation the mean airway pressure (MAP) must be altered. The components of MAP include PIP, positive end-expiratory pressure (PEEP), inspiratory time, and frequency. Reducing the rate would reduce MAP, and reducing the tidal volume would reduce PIP, hence reduce MAP. Increasing the inspiratory time to 2 seconds would increase MAP, but would be an inappropriate initial maneuver with the PEEP at only 5 cm H2O. The increased inspiratory time of 2 seconds would induce air trapping without simultaneous alteration of the rate, as the entire respiratory cycle is 2 seconds. Increasing the PEEP to 10 cm H2O would have the greatest effect on oxygenation by its doubling of the MAP.

223
Q

Which of the following curves best demonstrates first-order kinetics of drug administration? (P = plasma concentration)?

A. A

B. B

C. C

D. D

A

Answer: B

First-order kinetics= linear =constant fraction of drug is eliminated per unit time. Half-life (t1/2) and clearance are independent of drug dose and are constant. This curve is demonstrated by graph B with a steady(constant) fraction of elimination. Zero-order kinetics = nonlinear = constant amount (not fraction) of drug is eliminated per unit time. The t1/2 and clearance are dependent on drug dose and vary with concentration. Choice D represents zero-order kinetics for most drugs.

225
Q

A one-year-old patient with biliary atresia has failed a Kasai procedure and is in the hospital awaiting liver transplantation. She is increasingly lethargic in the morning and is transferred to the pediatric intensive care unit for periodic desaturations. On exam, you find her with a T of 36.8, HR of 122, BP of 90/61, RR of 14, and pulse oximetry of 98% on room air with periodic drops to 92% at the lowest. She grimaces slightly during sternal rub and noxious stimuli in the extremities. Her respirations are very deep but not labored; however, you note a slight snore in this patient. Her heart exam is normal. Her abdomen is distended with a fluid wave but not tender. Her labs are notable for total bilirubin of 42 g/dL, PT of 19.2 with INR of 2.1, WBC of 8.3, albumin of 2.1, and sodium level of 128. Which one of the following next steps is best?

A - Order STAT ammonia level and arterial blood gas with plan to start lactulose and/or intubate based on results

B - Supplemental oxygen and lactulose via nasogastric tube which you place

C - Intubation for airway protection followed by lactulose via nasogastric tube

D - Noninvasive positive pressure with subsequent arterial blood gas

E - Administer 3% NaCl 3-5 ml/kg via a central line to increase Na to > 130

A

C - Intubation for airway protection followed by lactulose via nasogastric tube

This child with known cirrhosis is presenting with what is most likely hepatic encephalopathy. Although respiratory acidosis may be contributing to the obtundation, it is unlikely to be the cause given the lack of respiratory findings. The depth of coma and potential for impending respiratory failure (early airway obstruction, abnormal breathing pattern, etc.) dictate airway protection via intubation. Furthermore, the treatment for the encephalopathy consists of lactulose regardless of what the present ammonia levels are (as these often correlate poorly with clinical status). Hyponatremia may cause mental status changes and seizures, but is a rare cause of coma at these given levels.

226
Q

A 3-year-old girl suffered a severe traumatic brain injury. An intraventricular catheter was placed on the day of admission. The patient is intubated and sedated. HR is 130/min and BP is 96/54 mm Hg. You are called to the bedside because of a change in the brain wave tracing on the monitor, shown in the figure.

Which of the following is the best next management step?

A. Increasing the FIO2 to 100%

B. Giving 3 mL/kg of 3% saline

C. Administration of 20 mL/kg of normal saline

D. Starting the patient on an epinephrine drip

A

Answer: B - Giving 3 mL/kg of 3% saline

The patient described in the vignette has a traumatic brain injury and probably has had an increase in intracranial pressure (ICP). The ICP waveform has 3 upstrokes in one wave. P1 reflects arterial pulsation, P2 represents intracranial compliance, and P3 reflects the closure of the aortic valve. If the brain compliance is normal, then P1 has the highest upstroke, but in cases of decreased brain compliance (traumatic brain injury, intracranial bleeding, hydrocephalus), P2 would have the highest upstroke.

227
Q

In a child with hypoplastic left heart syndrome (HLHS) and parallel circulation, assuming a pulmonary venous saturation of 95%, which of the following is the appropriate arterial saturation (SaO2) and mixed venous saturation (SvO2) to achieve a Qp:Qs of 1:1 based on the Fick equation?

A. SaO2 of 85% and SvO2 of 65%

B. SaO2 of 85% and SvO2 of 50%

C. SaO2 of 80% and SvO2 of 65%

D. SaO2 of 80% and SvO2 of 50%

E. SaO2 of 75% and SvO2 of 55%

A

Answer: C - SaO2 of 80% and SvO2 of 65%

Patients with hypoplastic left heart syndrome (HLHS) ideally have a Qp/Qs of 1:1. SaO2 is typically used to guide treatment of unbalanced pulmonary-to-systemic blood flow ratio. The Fick equation allows calculation of the Qp:Qs ratio based on oxygen content values at different points in the circulation:

Qp:Qs = (Ao – SVC)/(PV – PA)

Ao, SVC, PV, and PA are the values for oxygen content at the aorta (Ao), superior vena cava (SVC), pulmonary vein (PV), and pulmonary artery (PA). Since these patients have complete mixing and parallel circulations, Ao and PA oxygen content can be assumed to be the same. In this question, PV saturation was assumed to be 95%. Therefore, the calculation for answer C is as follows:

Qp:Qs= (Ao – SVC)/(PV – PA) Qp:Qs= (80 – 65)/(95 – 80) Qp:Qs= 15/15
Qp:Qs= 1

It should be noted that this calculation can be problematic. SVC saturation is not always available, and pulmonary venous saturation is an assumed number. If the SVC saturation was 50%, the calculation would be as follows:

Qp:Qs= (Ao – SVC)/(PV – PA) Qp:Qs= (80 – 50)/(95 – 80) Qp:Qs= 30/15
Qp:Qs= 2

However, in the perioperative period, total cardiac output and metabolic demand frequently may be mismatched as a result of the inherent instability of parallel circulation as described above, and variability in the Qp/Qs, total flow (Qt), and oxygen consumption (VO2) occurs. Impaired systemic oxygen delivery can occur with SaO2 closely maintained in the target 75%-85% range.

229
Q

Which one of the following statements regarding tacrolimus is accurate?

a. Should be measured in serum
b. Inhibit B cell proliferation
c. Dialysis treatment of choice for acute toxicity
d. Mean half-life 48-72 hours
e. Nephrotoxicity and neurotoxicity are the most common adverse effects

A

e. Nephrotoxicity and neurotoxicity are the most common adverse effects

Tacrolimus is widely referred by its experimental designation FK506. Tacrolimus inhibits proliferative responses in-vitro to alloantigen and mitogen stimulation and inhibits cytotoxic T cell proliferation and differentiation. Tacrolimus is a water-insoluble lipophilic agent that is highly bound to erythrocytes and therefore whole blood measurement is the appropriate method for evaluating pharmacokinetics. Peak blood concentration after oral administration occurs in 1-3 hours and terminal elimination half-life is 4-41 hours (with a mean half-life of 12 hours, 8 hours in children). Tacrolimus is metabolized by hepatic cytochrome P450 enzyme system and less than 1% of the drug is excreted in urine and it is not dialyzable. Nephrotoxicity and neurotoxicity remain the most frequent potentially serious complications.

230
Q

Which of the following curves describes the relationship of lung volume to pulmonary vascular resistance?

A. Curve A

B. Curve B

C. Curve C

D. Curve D

A

Answer: B

The relationship between lung volume and pulmonary vascular resistance is characterized by a U-shaped curve. When lung volume is too low, pulmonary vascular resistance will be elevated. When lung volume is excessive, increased pulmonary vascular resistance also will be present. At optimal lung volume, pulmonary vascular resistance will be at a minimum.

230
Q

A 1-day-old, full-term male neonate with hypoplastic left heart syndrome (HLHS) has an SaO2 of 60%. The echocardiogram shows good right ventricular function, trivial tricuspid regurgitation, and a somewhat restrictive atrial septum. Over the next 6 hours, the patient’s condition deteriorates. His PaO2 levels are in mid to high 20s mm Hg with lactic acidosis. He is being given prostaglandin E1 (PGE1) at 0.05 μg/kg/min, intubated with positive end-expiratory pressure (PEEP) of 6 cm H2O, FIO2 of 0.5, and mean airway pressure of 13 cm H2O. Which of the following is the most appropriate next step in management?

A. Administer nitric oxide

B. Increase PEEP

C. Increase FIO2

D. Increase the PGE1 infusion

E. Initiate intervention for restrictive atrial septum

A

Answer: E - Initiate intervention for restrictive atrial septum

Echocardiography is usually diagnostic of a restrictive atrial septum. In a patient with a severely restrictive atrial septum, urgent cardiac catheterization with balloon septostomy, dilation, or stent placement may be lifesaving. Some centers may prefer immediate surgical intervention and palliation in this setting.

231
Q

A 12 year old in respiratory failure is intubated and on conventional ventilation. Her ventilator settings are as follows: PC/AC PIP 31 PEEP 10 rate 20 Ti 1 second FiO2 100%. Most recent ABG showed a PaO2 of 85. Her oxygenation index is closest to which one of the following values?

A - 15

B - 31

C - 24

D - 17

E - 20

A

E - 20

Where MAP = mean airway pressure, oxygenation index is calculated by the equation OI = [(MAP x FiO2)/PaO2)] x 100. MAP can be calculated from the above ventilator settings with simple math: Given a PEEP of 10 and a total inspiratory time per minute of 20 seconds (rate = 20 and Ti = 1 second), with a peak pressure of 31 for each of those 20 seconds, and a flat pressure waveform (pressure mode of ventilation), the MAP = 17. The equation used here is as follows: MAP = [(PIP - PEEP) x (Ti/Ti + Te)] + PEEP. The OI in this case is thus: [(17 x 1)/85] x 100 = 20.

232
Q

A 10-year-old patient is brought to the ER following a gunshot wound to the thorax at the level of the right fifth intercostal space in the anterior axillary line. On examination, the patient is awake and alert. Vital signs reveal a pulse rate of 121 bpm, respiratory rate of 28 per minute, and blood pressure of 98/58 mmHg. Laboratory studies demonstrate a hematocrit of 34%. Furthermore, an X-ray of the chest reveals a right hemothorax, a right pneumothorax, and a bullet fragment in the left upper thorax. Which one of the following factors is the most compelling indication for thoracic surgical exploration in this patient?

A - The missile path traversing the mediastinum

B - A gunshot wound involving the thorax

C - A right hemothorax

D - A right pneumothorax

E - The probability of hypovolemic shock

A

A - The missile path traversing the mediastinum

Surgical exploration has become more conservative with improved abilities to do noninvasive evaluations. However, any penetrating injury which traverses the mediastinum and the vascular structures in a stable patient will require exploration or vascular imaging to look for vascular injury. In this example, Options B, C, D, and E are not indications for surgical exploration.

233
Q

Which one of the following factors triggers the diving reflex in pediatric patients?

A - Central hypoxemia

B - Peripheral vasoconstriction

C - Stimulation of the hypoglossal nerve

D - Stimulation of the trigeminal nerve

E - Extreme bradycardia

A

D - Stimulation of the trigeminal nerve

The diving reflex is a physiologic response to stimulation of the trigeminal nerve usually by submerging the face in cold water. Once the trigeminal nerve is stimulated, the following physiologic response occurs with bradycardia occurring first. Bradycardia is then followed by peripheral vasoconstriction and blood shift which occurs to maintain integrity of vascular functions under increased pressure, as seen during deep-level diving.

234
Q

According to the American Society of Anesthesiologists (ASA) Physical Status Classification, a patient with severe systemic disease or a controlled disease of more than one body system or one major system and no immediate danger of death, would be classified as ASA level:

A. I

B. II

C. III

D. IV

E. V

A

Answer: C - III

As pediatric critical care providers are increasingly involved in providing sedation and analgesia for patients undergoing various types of procedures, whether in the pediatric care unit or in other hospital settings; pediatric critical care providers should be familiar with preoperative assessment tools. One of those tools is the American Society of Anesthesiologists (ASA) Physical Status Classification; this metric is an assessment of the child’s preexisting disease. However, the ASA classification should be used with caution as a predictive tool, as it was not designed for use in children or as a predictor of their perioperative outcome, and there is poor interrater reliability in the pediatric setting.

ASA Physical Status 1—A normal healthy patient
ASA Physical Status 2—A patient with mild systemic disease
ASA Physical Status 3—A patient with severe systemic disease
ASA Physical Status 4—A patient with severe systemic disease that is a constant threat to life ASA Physical Status 5—A moribund patient who is not expected to survive without the operation
ASA Physical Status 6—A declared brain-dead patient whose organs are being removed for donor purposes

235
Q

Intracerebral pressure recordings through a ventricular catheter are characterized by many well defined wave forms. Which one of the following waveforms is considered most ominous?

a. A-waves
b. B-waves
c. C-waves
d. D-waves
e. E-waves

A

a. A-waves

Ventricular waveforms, recorded through an ICP transducer, show the following waves: A-waves or plateau waves – amplitude of 50-100 mmHg, lasting 5-20 minutes. These waves are always associated with intracranial pathology. During such waves it is common to observe evidence of early herniation, including bradycardia and hypertension. They are thought to occur when cerebral perfusion pressure is inadequate to meet metabolic demands. B-waves – oscillating waves, up to 50 mmHg in amplitude and a frequency of 0.5-2 per minute. They are thought to be due to vasomotor center instability. C-waves – oscillating waves, up to 20 mmHg in amplitude and a frequency of 4-8 per minute. These waves have been documented in healthy individuals and are thought to occur because of interaction between cardiac and respiratory cycles. Both A and B waves require intervention to reduce ICP and preserve CPP. However A-waves (or plateau waves) are considered more ominous.

236
Q

In an infant with a recent palliative procedure that included an aortopulmonary shunt and repair of total anomalous pulmonary venous connection, HR abruptly increases to 184/min despite adequate volume resuscitation. Adenosine is given and succesfully converts the tachycardia. Which of the following is the most likely diagnosis?

A. Reentrant-type supraventricular tachycardia

B. Ventricular tachycardia

C. Ectopic atrial tachycardia

D. Junctional ectopic tachycardia

A

Answer: A - Reentrant-type SVT

Supraventricular tachycardia typically has an abrupt onset and would respond to adenosine, which is effective in converting typical reentrant-type supraventricular tachycardia to sinus rhythm. Ventricular tachycardia, ectopic atrial tachycardia, and junctional ectopic tachycardia are types of arrhythmia that do not respond to adenosine treatment.

237
Q

Which one of the following statements regarding systematic reviews is not accurate?

A - Systematic review contains metaanalysis

B - In designing systematic review, formulating a hypothesis is not necessary

C - The quality of studies included in the systematic review affect the quality of the systematic review

D - A systematic review avoids conducting a randomized controlled trial

E - For systematic review, studies should be located from a single database

A

C - The quality of studies included in the systematic review affect the quality of the systematic review

Systematic reviews are necessary when larger trials are not feasible or when studies have inconsistent results. Systematic reviews may or may not contain metaanalysis, thus, option A is incorrect. As with any study, the first step is to formulate a hypothesis or question, thus, option B is incorrect. Option C is correct because the quality of systematic review depends upon the quality of studies included in the review process. Biases in the original studies do not disappear, but systematic reviews attempt to minimize the selection bias. Option D is incorrect because systematic review is not a substitute for randomized controlled studies. Option E is incorrect because studies from multiple databases can be included in the review.

238
Q

A 2-year-old child status-post cardiac transplant is undergoing follow-up cardiac catheterization. His weight is 12 kg. His latest CBC shows WBC 6.5, Hb 14, platelets 250. His arterial saturation on FiO2 30% is 95% and mixed venous oxygen saturation is 65%. Assuming an oxygen consumption of 180 ml/min/m2, what will be his calculated cardiac output?

a. 2.5 lt/min
b. 3.0 lt/min
c. 3.5 lt/min
d. 4.0 lt/min
e. 4.5 lt/min

A

b. 3.0 lt/min

In the absence of any shunt, the pulmonary blood flow and systemic cardiac output are the same and may be measured as part of investigation of patients with impaired cardiac function—notably, as part of transplant assessment. One of the simplest methods of measuring cardiac output is by using Fick method. Fick principle requires estimation of oxygen consumption; assumed values based on age, sex, and heart rate are often substituted. In cardiac output estimation by Fick principle, the following apply: (1) Blood flow is calculated by measurement of the oxygen content of venous blood and hence estimating the difference between the 2, which represents the tissue oxygen utilization; and (2) If oxygen consumption is known, the blood flow is calculated by the simple equation which follows: Q = VO2/ A-V O2 difference (Where Q = blood flow in lts/min) Or Q = VO2/CaO2 - CvO2. CaO2 = 140 × 0.95 × 1.34 = 180 mL/lt. CvO2 = 140 × 0.65 × 1.34 = 120 ml/lt. Q = 180/60. Finally, Q = 3 lt/min.

240
Q

You are evaluating a 1-week-old child with cyanotic heart disease with usual form of transposition of great arteries with atrioventricular concordance, ventriculoarterial discordance, and subpulmonary conus. Patient is scheduled to have surgery later today. His chest X-ray is shown below. During rounds, a pediatric resident asks you what the correct nomenclature of this heart disease is. Which one of the following responses is correct?

a. Atrial situs solitus, d-loop, situs solitus normally related great arteries {S, D, S}
b. Atrial situs solitus, d-loop, d-transposition of the great arteries {S, D, D}
c. Atrial situs inversus, l-loop, situs inversus normally related arteries {I, L, I}
d. Atrial situs solitus, l-loop, l-transposition of the great arteries {S, L, L}
e. Atrial situs solitus, d-loop, l-malposition of the aorta {S, D, L}

A

b. Atrial situs solitus, d-loop, d-transposition of the great arteries {S, D, D}

This is a patient with usual transposition of great arteries with apex to left side (left ventricle is on left side) and d-transposition of great arteries. Option A is incorrect because this nomenclature is of normal heart without any malformation. Option B is correct because d-loop suggest that right ventricle is on right side and left ventricle is on left side. X-ray clearly shows apex (left ventricle) to be on the left side. Option C is incorrect. This nomenclature is of mirror image of normal heart with apex to right side and patient will not be cyanotic. Option D is incorrect because this is congenitally corrected transposition of great arteries and most patients are asymptomatic. Option E is also incorrect. This anomaly is part of Taussig-Bing heart. This usually has both pulmonary artery and aorta committed to right ventricle (DORV-double outlet right ventricle).

241
Q

What is the correct relationship between pericardial volume (x-axis) and pericardial pressure (y- axis)?

A. Curve A

B. Curve B

C. Curve C

D. Curve D

A

Answer. C

Pericardial pressure does not increase with volume until a certain threshold volume is reached, and then the pressure rapidly increases with further increases in volume. Curve C accurately reflects the pericardial volume-pressure relationship. Pericardial pressure varies with the cardiac cycle—maximum near end-diastole and minimum near end-systole. When there is tamponade physiology, pressure increases as the ventricles are attempting to fill.

242
Q

According to Poiseuille’s law, which one of the following statements is true if we wish to maintain a constant flow of gases?

A - Halving the tube radius will require a 16-time increase in driving pressure

B - Alveolar recruitment has its greatest effect on flow

C - Large jumps in driving pressure is needed to overcome airway narrowing

D - The driving pressure varies directly with the airway circumference

E - Resistance is proportional the length of the airway

A

A - Halving the tube radius will require a 16-time increase in driving pressure

Poiseuille’s law states that resistance is directly proportional to the length of the tube and the density of the liquid or gas. Resistance is inversely proportional to the radius of the tube to the fourth power causing a 16-time increase for a halving of the radius. Alveolar recruitment affects elastic changes and subsequent compliance but has little effect on resistance. When resistance increases, there is a pressure drop across that resistance—not an increase; it is not related to the airway circumference. Resistance is proportional to the length of the airway and density but it is not related to the constant gas flow.

243
Q

You have just finished placing a 4Fr 5cm femoral venous catheter in a 2 week old with septic shock. There was some resistance to guide-wire advancement during line placement, but line advanced smoothly over guide wire. There is return of blood on aspiration through both lumens, but it is somewhat sluggish. Of the following methods of confirmation, which one will best confirm that the line is appropriately positioned?

A - Transducing a CVP

B - Sending a venous blood gas

C - Cross table lateral abdominal radiograph

D - Visualization of dark venous blood return

E - Measurement of venous oxygen saturation

A

C - Cross table lateral abdominal radiograph

Malposition of femoral catheters in the ascending lumbar vein is an infrequent complication but if left in place can result in tetraplegia… Ultrasonography, lateral radiography, or venogram is required when the location of the catheter tip is in question. Warning signs that may indicate catheter malposition include (1) loss of blood return on aspiration; (2) subtle lateral deviation, or hump, of the catheter at the level of L4 or L5 on frontal abdominal radiographs in catheters placed from the left side…(3) a catheter path directly overlying the vertebral column rather than the expected path to the right of the midline for a catheter in the inferior vena cava; and (4) resistance to guide wire advancement during insertion. A lateral abdominal radiograph may confirm the posterior position of the catheter.”

244
Q

A 12-year-old patient is on venovenous ECMO via dual lumen bicaval internal jugular cannula. He is 18% fluid overloaded and has failed to maintain a negative or even fluid balance despite furosemide infusion of 0.2 mg/kg/h. The most appropriate action at this time would be which one of the following options?

A - Add chlorothiazide q6h and increase furosemide infusion to 0.3 mg/kg/h

B - Add bumetanide infusion at 0.05 mg/kg/h

C - Place femoral dialysis catheter, after holding heparin infusion for 1 hour prior to procedure. Initiate continuous renal replacement therapy via new catheter

D - Place dialysis filter in-line with ECMO circuit and remove fluid as aggressively as hemodynamics and electrolytes will tolerate

E - Continue current therapy

A

D - Place dialysis filter in-line with ECMO circuit and remove fluid as aggressively as hemodynamics and electrolytes will tolerate

Fluid overload >10% on ECMO has been associated with mortality in pediatric literature, thus fluid overload of 18% on ECMO should be aggressively treated, and further increases in diuretics are unlikely to provide enough benefit. An additional catheter is unnecessary as well as unsafe (all vascular access should be avoided on ECMO if at all possible given the risk of severe bleeding from puncture sites) as the hemofilter can be placed directly in line with the ECMO circuit.

245
Q

A 2-year-old, previously healthy girl presents with respiratory failure. She has a history of fever, cough, and runny nose. Her younger sister is hospitalized with influenza. The patient was seen by her primary physician and started on oral amoxicillin for an ear infection. Chest radiograph reveals a left-sided consolidation with effusion. She is now intubated and receiving fluid boluses and inotropic support for poor systemic perfusion. Which of the following represents the best agent to include in initial treatment?

A. Ceftriaxone

B. Cefuroxime

C. Clindamycin

D. Trimethoprim/sulfamethoxazole

E. Vancomycin

A

Answer: E - Vancomycin

Although it is currently unproven that the patient has invasive bacterial disease, she has life- threatening disease with respiratory and cardiovascular failure. Additionally, she has received prior antibiotics, making antimicrobial resistance a potential complication. Vancomycin is indicated in the initial treatment of life-threatening illness potentially due to a gram-positive organism with altered antibiotic sensitivities. Although cefuroxime, ceftriaxone, or clindamycin may be used in combination with vancomycin initially, these would not represent the best agents in initial empiric therapy. Additonally, these agents may be indicated after antimicrobial sensitivity patterns are established. The use of trimethoprim/sulfamethoxazole in this patient would require a high suspicion for Pneumocystis jiroveci.

While Streptococcus pneumoniae is the most common cause of community-acquired pneumonia and bacterial superinfection following influenza, the incidence of Staphylococcus aureus (especially methicillin-resistant) is increasing and is the leading cause of mortality with influenza. Therefore, the choice of vancomycin is the most appropriate to treat S pneumoniae (including penicillin-resistant) and S aureus.

246
Q

Which one of the following vasoactive materials pass through the lungs without significant gain or loss of activity?

a. Bradykinin
b. Norepinephrine
c. Vasopressin
d. Serotonin
e. Angiotensin

A

C - Vasopressin

The lung has important metabolic functions in addition to gas exchange. A number of vasoactive substances are metabolized by the lung. Because the lung is the only organ—except the heart—that receives the whole circulation, it is uniquely situated for modifying bloodborne substances. The following table documents the fate of substances in the pulmonary circulation:

247
Q

A 4-year old child presents with 3-4 day history of high fever, headache, abdominal pain and vomiting. He also has a blanching erythematous macular rash, that had started on wrist and ankles, it has now spread all over the body and involves palms and soles. There is no history off tick bite. His white cell count is 8000 and platelet count 80,000. Serum chemistry is normal except sodium of 130 meq/lt. While antibody titers are pending, which one of the following options is the most appropriate management strategy?

a. Start doxycycline
b. Start chloramphenicol
c. Start ceftriaxone
d. Start vancomycin
e. Withhold antibiotics until confirmation of diagnosis

A

a. Start doxycycline

Rocky Mountain spotted fever (RMSF) is a tick-borne infection with the potential for causing multisystem disease. RMSF is caused by Rickettsia Rickettsii, which are small gram-ve coccobacilli carried by dog or wood ticks. The classic triad of RMSF consists of fever, rash, and headache with an h/o tick bite. It is present in only 60% of patients and approximately 30%-40% of patients do not recall having a tick bite. The rash often begins by day 4, with blanching maculopapular rash classically starting on wrists and ankles and then appearing on palms and soles and trunk. R. Rickettsii invade and multiply in the endothelial cells of the vasculature, resulting in cytopathic vascular injury that is responsible for the morbidity and mortality. Among laboratory values, the WBC count is variable; however, most patients develop decreased platelets as disease progresses. Approximately 20% of patients have hyponatremia and hypoalbuminemia resulting from massive capillary leak. Because R. Rickettsii often is not cultured, diagnosis usually is clinical. Serum antibodies do not become positive until 7-10 days into the illness and pending results should not delay treatment. The DOC for treatment is doxycycline.

248
Q

A 12-year-old boy is admitted to the ICU after he was found unconscious in a house fire. He is now intubated and on FiO2 of 0.6. His pulse oximeter reads the arterial oxygen saturations (SaO2) as 100%. His carboxyhemoglobin level is 25%. Which one of the following options best describes this patient’s SaO2 and oxyhemoglobin (oxy-Hb) dissociation curve shift?

A - SaO2 is falsely elevated and the oxygen hemoglobin dissociation curve is shifted to the right

B - SaO2 is falsely elevated and the oxy-Hb dissociation curve is shifted to the left

C - SaO2 is appropriately elevated and oxy-Hb dissociation curve is shifted to the right

D - SaO2 is appropriately elevated and oxy-Hb dissociation curve is shifted to the left

E - FiO2 of 0.6 is causing elevated SaO2 and there is no shift in the oxy-Hb dissociation curve

A

B - SaO2 is falsely elevated and the oxy-Hb dissociation curve is shifted to the left

Carbon monoxide has a higher affinity to bind with Hb as compared to oxygen. Option B is correct because binding of carbon monoxide to hemoglobin causes increased affinity of oxygen to bind to the remaining sites. This is reflected as a leftward shift of the oxy-Hb dissociation curve. Carboxyhemoglobin is interpreted as oxyhemoglobin by the pulse oximeter—hence, in this clinical scenario, SaO2 is falsely elevated. Options A and C are incorrect because the oxy-Hb curve is shifted to the left. Option D is incorrect as the SaO2 are falsely elevated, and cooximetry is imperative in clinical suspicion of carbon monoxide poisoning. Option E is incorrect because there is a leftward shift of oxy-Hb curve in carbon monoxide poisoning.

249
Q

A 3-year-old African American girl with trisomy 21 was seen in the emergency department prior to admission to the pediatric ICU. She presented with a 3-day history of poor feeding and abdominal distension. She was diagnosed with Hirschsprung disease at birth and a distal colonic resection was performed at 1 month of age. Three weeks ago, she received a 15-day course of trimethoprim/sulfamethoxazole for pneumonia. Her abdomen is tender and mildly distended with a shiny appearance. Bowel sounds are present and bloody mucoid stools are felt in the rectal vault. Stools are obtained for cytotoxic assay. On the issue of antibiotic coverage and source control, the preferred therapeutic approach to this patient’s management is:

A. Consideration of administration of enteral vancomycin

B. Consideration of antitoxin therapy for possible Clostridium difficile in addition to IV metronidazole

C. IV clindamycin with ceftriaxone and a rectal tube insertion to evacuate the stools

D. Keeping the child nil per os in anticipation of surgical exploration in the operating room

E. Placement of a nasogastric tube for gastric decompression and the administration of IV vancomycin and ceftriaxone

A

Answer: A - Consideration of administration of enteral vancomycin

This case emphasizes the proper choice and route of administration of antibiotics in a case of enterocolitis in which a strong suspicion for Clostridium difficile exists.

Clindamycin is not the appropriate antibiotic coverage for the treatment of C difficile.

Vancomycin is the appropriate antibiotic for C difficile, but the IV route is not preferred. Both a nasogastric tube and a rectal tube decompression may be considered based on the discretion of the practitioner but are not considered a good form of source control.

A surgical exploration may be considered later for bowel decompression and correction of possible toxic megacolon but is not the preferred therapeutic intervention for source control in this case.

The enteral administration of vancomycin is the preferred therapeutic option for patients with severe enterocolitis. Because the patient appears to be septic with signs suggesting hemodynamic instability, early antibiotic treatment within the first hour must be considered.

Antitoxin therapy and IV metronidazole are not acceptable therapeutic modalities to effectively treat infection with C difficile.

250
Q

Both transfusion-related acute lung injury (TRALI) and transfusion-associated circulatory overload (TACO) are associated with which of the following clinical characteristics?

A. Antibody-mediated disease processes

B. Decreased oxygen saturation and bilateral lung infiltrates

C. Frequent occurrence in clinical practice

D. Noncardiogenic pulmonary edema

E. Pulmonary edema that responds acutely to diuretics

A

Answer: B - Decreased oxygen saturation and bilateral lung infiltrates

Both transfusion-related acute lung injury (TRALI) and transfusion-associated circulatory overload (TACO) exhibit similar symptoms initially, with decreases in oxygen saturation and bilateral infiltrates on radiography. However, TACO responds well to diuretics, whereas TRALI patients do not. Thus, choice E is incorrect. TACO is a cardiogenic pulmonary edema whereas TRALI is noncardiogenic; thus, choice D is incorrect. TRALI is an antibody-mediated process in some cases, whereas TACO is a nonimmune process altogether; hence, choice A is incorrect. Finally, choice C is incorrect because despite the rarity (likely due to underreporting/diagnosing of TRALI and TACO) the symptoms occur much closer to the transfusion, any time immediately after the transfusion, during the transfusion, and up to 8 hours posttransfusion (12 hours is too far out from transfusion).

251
Q

A 16-year-old female collapses while playing basketball. She undergoes cardiopulmonary resuscitation, endotracheal intubation, and has return of spontaneous circulation within 10 minutes. She is admitted to the ICU and the rhythm shown in the figure is noted. Which of the following arrhythmias is the most likely primary cause of her collapse?

A. Third-degree heart block

B. Supraventricular tachycardia

C. Atrial fibrillation

D. Ventricular tachycardia

E. Junctional tachycardia

A

Answer: D - Ventricular tachycardia

The ECG demonstrates a prolonged QT interval (0.44, corrected 0.55). Patients with prolonged QT intervals are at risk for the development of ventricular tachycardia and sudden death. Specifically, they are at risk for the development of polymorphic ventricular tachycardia (torsade de pointes). This is a relatively rare disorder, which can be familial or caused by various medications/toxins, metabolic abnormalities, or various myocardial diseases. Torsade de pointes is frequently poorly tolerated hemodynamically and can quickly lead to cardiovascular collapse. Although asystole can be an end point of ventricular tachycardia, in the question provided it would not be the primary cause of this patient’s collapse. Third-degree heart block, supraventricular tachycardia, atrial fibrillation, and junctional tachycardia are not typically associated with prolongation of the QT interval.

253
Q

A 1-month-old boy presents with sluggish weight gain and sweating during feeding. Upon questioning, the mother describes the boy as extremely tired after feedings. Which one of the following diagnoses is the most likely of the given option?

A - Primary hypertension

B - Failure to thrive

C - Eisenmenger syndrome

D - Small ventricular septal defect

E - Moderate ventricular septal defect

A

E - Moderate ventricular septal defect

This question describes the most accurate diagnosis of a 1-month-old boy presenting with the symptoms of slow weight gain, sweating during feeding, and fatigue following feeding. Option A is incorrect because these symptoms are not characteristic of primary hypertension in infants. Option B is incorrect because sweating during feeding is not characteristic of failure to thrive. Option C is incorrect because Eisenmenger syndrome is typically asymptomatic at rest. During exercise or physical exertion, sweating during feeding, fatigue following feeding, and sluggish weight gain are not typical symptoms of Eisenmenger syndrome. Option D is incorrect because infants with a small ventricular septal defect are typically asymptomatic, except for heart murmur. Option E is correct because infants with a moderate ventricular septal defect often present with excessive sweating—especially during feedings—due to heightened sympathetic tone. In addition, infants typically display fatigue following feeding due to the increased cardiac output that occurs during feeding.

254
Q

A teenager with known type 1 diabetes mellitus presents with diabetic ketoacidosis. Following 15 mL/kg of normal saline and initiation of normal saline with 20 mEq/L each of potassium acetate and potassium phosphate, the patient’s level of consciousness remains depressed. Head CT findings are normal. Laboratory studies at this time include the following: sodium, 146 mEq/L; potassium, 4.0 mEq/L; chloride, 100 mEq/L; bicarbonate, 5 mEq/L; blood urea nitrogen, 20 mg/dL; creatinine, 0.7 mg/dL; glucose, 320 mg/dL; osmolality, 361 mOsm/L. Arterial blood gas results include pH of 7.26, PaCO2 of 20 mm Hg, PaO2 of 120 mm Hg, bicarbonate of 5 mEq/L, and base deficit of 20 mEq/L. Which of the following studies would provide additional important diagnostic information?

A. Calculated osmolality

B. Corrected serum sodium level

C. Serum ammonia level

D. Serum beta-hydroxybutyrate level

E. Serum lactate level

A

Answer: A - Calculated osmolality

A normal osmolar gap should be less than 10 mOsm. This patient with diabetic ketoacidosis has a significant osmolar gap; that is, serum osmolality (Osm) not accounted for by levels of sodium, glucose, and blood urea nitrogen (BUN).

Calculated Serum Osmolality = 2 × Sodium + Glucose/18 + Blood Urea Nitrogen/2.8 = 2 × (146) + 320/18 + 20/2.8 = 317

mOsm gap = (Measured) – (Calculated)
mOsm gap = 361 mOsm – 317 mOsm = 44 mOsm

Any exogenous ingestion of alcohol, ethanol, methanol, or ethylene glycol, could account for this scenario. In this vignette, the patient’s blood ethanol concentration was 0.2% g/dL.

0.2 g/dL × 10 dL/1 L × 1 mol/46g × 1,000 mmol/mol × 1 mOsm/mmol 44 mOsm, where ethanol molecular weight = 46 g/mol

Serum beta-hydroxybutyrate and serum ammonia levels would not contribute substantially to serum osmolality. Corrected serum sodium level for this patient would not be worrisome in this setting. In fact, corrected serum sodium would be lower. Serum lactate can contribute to the osmolar gap but not to the degree illustrated in this scenario. The acidosis in diabetic ketoacidosis would not be attributed to lactic acidosis.

256
Q

A 3-year-old boy was found unconscious by his mother with a treadmill cord around his neck. His mother called 911 and performed CPR on the child. When paramedics arrived, the child had palpable pulse, but was unconscious with irregular breathing, so he was intubated at the scene and is now admitted to PICU for further care. On examination 4 hours after the accident, the child is found to be comatose with Glasgow coma scale of 6 with withdrawal response to painful stimulus. His pupils are 5 mm in size and nonreactive to light on both sides; he does not have cough or gag reflux. His heart rate is 140/min on ventilator with respiratory rate of 24, blood pressure of 88/45 mmHg. The parents of the child ask what the outcome will be for their son. Which one of the following responses is the best response to parent’s question?

A - Patient most probably is going to be brain dead within 24 to 36 hours

B - Child is most probably going to have severe neurological deficits

C - The best next step is to wait and monitor his neurological status for next 24 hours

D - Organ donation is indicated due to his lack of brainstem reflexes

E - The next best step would be electroencephalogram (EEG) right away to better answer the question

A

C - The best next step is to wait and monitor his neurological status for next 24 hours

This child has hypoxic ischemic encephalopathy after accidental strangulation. He had spontaneous return of circulation at the scene and is hemodynamically stable. It is important to know the limitations of examination or tests in predicting prognosis. No examination findings or tests done within 24 hours after CPR can help to accurately predict outcome. Option A is incorrect because it is difficult to say that he is going to be brain dead within a certain timeframe. Option B is incorrect because neurological examination findings less than 24 hours after CPR are not predictive of outcome. Option C is correct because it is prudent to wait for 24 hours and to monitor neurological recovery of the child. Option D is incorrect, as the child is not brain dead, so organ donation discussion is not appropriate at this time. Option E is incorrect, as EEG done so early after resuscitation is also not predictive of child’s outcome.

257
Q

A 5-year-old boy with myasthenia gravis is admitted to ICU for pneumonia. Which one of the following antibiotics is the safest for treating his condition?

a. Amikacin
b. Ciprofloxacin
c. Ampicillin
d. Clindamycin
e. Vancomycin

A

E - Vancomycin

Clinician must always be cautious when initiating medications in the patient with myasthenia gravis. Many drugs interfere with the neuromuscular junction; the best-known are the aminoglycoside medications. Other antibiotics that have been implicated in the worsening of myasthenic symptoms include ampicillin, ciprofloxacin, clindamycin, erythromycin, sulfonamide, tetracycline, and the peptide antibiotics (polymyxin A and B and colistin).

259
Q

A 4-year-old girl with a long-standing history of cyanosis and a cardiac murmur presents with fever, new-onset tonic-clonic generalized seizure activity on the right side, and altered mental status. Notably, she recently moved with her family to the United States from Africa. She has had a fever, headache, and vomiting for the past week. Physical examination reveals oxygen saturation of 76% on room air, a temperature of 101.3 F (38.5), a loud harsh systolic ejection murmur, and hemiparesis on the left side. Which of the following is the most likely cause of the clinical findings?

A. Brain abscess

B. Cerebral embolism

C. Cerebral hemorrhage

D. Encephalitis

E. Meningitis

A

Answer: A - Brain abscess

Causes of brain abscess include embolization due to congenital heart disease with right-to-left shunts (especially tetralogy of Fallot), meningitis, chronic otitis media and mastoiditis, sinusitis, soft-tissue infection of the face or scalp, orbital cellulitis, dental infections, penetrating head injuries, immunodeficiency states, and infection of ventriculoperitoneal shunts.

This clinical presentation is consistent with uncorrected tetralogy of Fallot (systolic ejection murmur and cyanosis) and a brain abscess. Congenital heart disease is the most common presenting factor for brain abscess. The classic triad is fever, headache, and neurologic deficit. Not all patients present with focal findings; clinical findings depend on the region of the brain that is affected. Causative agents depend on the predisposing factor. In the case of congenital heart disease, the most common causative organisms are Streptococcus viridans, microaerophilic streptococci, and Haemophilus species.

260
Q

Bartter’s syndrome is a rare syndrome due to defective transport in the thick ascending limb of loop of Henle. Which one of the following options represents a typical clinical feature of this condition?

A - Hypertension

B - Hypokalemia

C - Mental retardation

D - Deafness

E - Blindness

A

D - Deafness

Bartter’s syndrome is a rare condition that is due to defective transport in the thick ascending limb. It is characterized by chronic sodium loss in the urine, with resultant hypovolemia, causing stimulation of renin and aldosterones secretion without hypertension, plus hyperkalemia and alkalosis. Due to defective Na+ and K+ transport in the scala media, patients with Bartter’s syndrome are very commonly deaf.

261
Q

A 3-year-old boy is found in the garage with a milk jug in his hand which is used to store radiator fluid. He is rushed to the hospital by his father and transferred to the pediatric intensive care unit after labs are obtained and maintenance IV fluids (5% dextrose and half normal saline) are started. The child has a HR of 114, BP of 98/62, and RR of 20. He is awake but a bit irritable. His labs are notable for the following levels: sodium of 136, potassium of 4.4, chloride of 98, bicarbonate of 12, BUN of 21, creatinine of 0.8, and glucose of 88. Which one of the following therapies would you now prescribe?

A - Ethanol 40 ml/kg over 24 hours

B - Fomepizole 15 mg/kg followed by 10 mg/kg every 12 hours

C - Increase hydration to twice maintenance

D - Titrate hydration to maintain urine output =1 ml/kg/h and administer fomepizole

E - Change IV fluid to normal saline and increase rate to twice maintenance

A

D - Titrate hydration to maintain urine output =1 ml/kg/h and administer fomepizole

This child presents with likely ethylene glycol poisoning evidenced by his anion gap metabolic acidosis. Hydration is crucial to maintain urine output but should be titrated to avoid fluid overload. Fomepizole is more expensive than ethanol, but has substantially less side effects (somnolence, confusion, etc.) and thus is the preferred agent if available. Blockade of the alcohol dehydrogenase enzyme with the use of fomepizole is crucial in preventing formation of the toxic metabolites of ethylene glycol (and methanol).

262
Q

Draw a graph of the relationship of pulmonary vascular resistance with lung volume. Explain the shape of the curve in terms of what happens to extra- and intra-alveolar vessels.

A

The low part of this curve represents the area where lung volume = functional residual capacity. Here pulmonary vascular resistance is lowest.
As expected, as lung volume increases, pulmonary vascular resistance increases b/c intra-alveolar vessels are compressed.
even though extra-alveolar vessels should distend, the net effect is increased resistance
For decreased lung volume to occur, there must be a positive pleural pressure → compression/collapse of extra-alveolar vessels and a small increase in pulmonary vascular resistance.
here effects in extra-alveolar vessels dominate

263
Q

A 10-year-old child with pneumonia is on mechanical ventilator. His current ventilator settings are TV 300, rate 20, FiO2 90%, PEEP 7. PaO2 on his blood gas is 240 mmHg. Assuming his minute ventilation, cardiac output, and VQ mismatching does not change, to achieve a desired PaO2 of 80 mmHg, what should the FiO2 be changed to?

A - 21%

B - 30%

C - 50%

D - 60%

E - 70%

A

B - 30%

The desired FiO2 can be calculated by the following equation: Desired FiO2 = PaO2 (desired) × FiO2 (known) / PaO2 (known) 80 × 90 / 240. Desired FiO2 = 30%.

264
Q

Which one of the following conditions can cause a clinically significant hypophosphatemia?

A - Hypoparathyroidism

B - Metabolic alkalosis

C - Respiratory alkalosis

D - Vitamin D intoxication

E - Hypothermia

A

C - Respiratory alkalosis

Clinically significant causes of hypophosphatemia include decreased intake, malabsorption, Vitamin D deficiency, inadequate amounts in TPN, aluminium and magnesium based P-binding antacids, increased renal losses, DKA, hyperparathyroidism, intracellular shift, respiratory alkalosis, refeeding after starvation, burns, and rewarming after hypothermia. Moderate hypophosphatemia is common in pediatric ICU patients and is usually asymptomatic. Severe hypophosphatemia (

265
Q

A 17-year-old boy, who was in a motor vehicle accident, presents at the ED. He is making incomprehensible sounds. He showed no response to command; however, on giving painful stimuli to the right shoulder, he opened his eyes and tried to reach for his shoulder. Which one of the following values represents his GCS score?

a. 7
b. 8
c. 9
d. 10
e. 11

A

C - 9

Glasgow coma scale:

266
Q

A 5-year-old with asthma is admitted to the pediatric ICU. He is receiving continuous albuterol along with IV methylprednisolone at 2 mg/kg/day. Which one of the following options is a correct order of steroids, with respect to their relative antiinflammatory activity?

A - Dexamethasone> methylprednisolone> prednisone> hydrocortisone

B - Hydrocortisone> prednisone > methylprednisolone> dexamethasone

C - Methylprednisolone> prednisone> dexamethasone> hydrocortisone

D - Dexamethasone> hydrocortisone> prednisone> methylprednisolone

E - Prednisone> dexamethasone> methylprednisolone> hydrocortisone

A

A - Dexamethasone> methylprednisolone> prednisone> hydrocortisone

Steroids are recommended for virtually all patients with acute asthma. The steroid preparation most often used in acute asthma is methylprednisolone (for intravenous therapy) or prednisone (for oral therapy). Various steroid preparations differ in their relative antiinflammatory activity (RAIA) and relative sodium retention (RSR) properties.

267
Q

A 15-year-old patient is admitted due to cough and shortness of breath. His symptoms had developed rapidly over the past 2 days and include significant chest pain and extreme exercise limitation. His past medical history is significant for intermittent asthma and history of several skin abscesses requiring incision and drainage, submandibular lymphadenopathy and nasal infections. Which one of the following tests could be ordered next to definitively define the diagnosis?

A - Serum precipitins

B - Oxidative burst assay

C - Candida skin testing

D - HIV antibodies

E - CH50

A

B - Oxidative burst assay

This case illustrates chronic granulomatous disease (CGD). This is caused by a defect in the NADPH oxidase system which generates superoxide for killing of bacteria and fungus. They can present in young childhood, though diagnosis in adulthood is not uncommon. Typical sites of infection include the lungs, lymph nodes, skin, and liver. Osteomyelitis and gingivitis are also common. Neutrophil oxidative burst assay is absent or markedly reduced in CGD. Serum precipitins are used in hypersensitivity pneumonitis. Since CGD effect is on neutrophil activity, skin testing for immune response would be unaffected. HIV is not associated with CGD. Complement as measured in CH50 is not affected.

268
Q

An infant is brought to the ICU after a Norwood operation that included insertion of an aortopulmonary shunt. The pulse oximetry reading is 50%. The chest radiograph shows that the lungs are clear. Assuming adequate cardiac output and normal oxygen extraction, what is the approximate ratio of pulmonary to systemic blood flow?

A. 3:1

B. 2:1

C. 1:1

D. 1:2

A

Answer: D - 1:2

The ratio of pulmonary to systemic blood flow is easily calculated in single-ventricle physiology patients using the Fick equation. This calculation is based on the following assumptions:
The arteriovenous oxygen difference (oxygen extraction) between the aorta and the mixed venous source is 25%.

There is no significant pulmonary disease, so that the pulmonary venous saturation is 95% to 100%.

The aortic and pulmonary artery saturations are equal (the former measured by pulse oximetry). The equation to calculate Qp:Qs is as follows:
(Aortic – Mixed Venous Saturation)/(Pulmonary Venous – Pulmonary Artery Saturation)

In this case:

(25%)/(100% – 50%])=1:2.

269
Q

A 14-month-old patient is being cared for by his grandparents and they witness him swallow a button battery from a hearing aid. He is rushed to an outside ER which then transfers him to the pediatric intensive care unit within one hour of ingestion. When you enter the room, the child is sitting in his grandparents lap looking apprehensive but not overly distressed. He has HR of 122, BP of 94/60 and RR of 28. His saturations are 98% on room air, and he has no increase in work of breathing. He pushes away food and water when it is offered to him. The chest radiograph from the outside ER shows a 20 mm battery lodged in the midesophagus without any other abnormalities. Which one of the following management steps is best in this case?

A - Continued observation in the pediatric intensive care unit until the next day in case of impending respiratory distress

B - Obtain a barium swallow to better visualize any obstruction and attempt to dislodge the battery

C - Transfer to the floor for continued observation given the child’s stability

D - Continued observation for 12-24 hours with repeat radiography and ENT and/or gastroenterology consult if the battery remains in place

E - Emergent ENT and/or gastroenterology consult to immediately remove the battery

A

E - Emergent ENT and/or gastroenterology consult to immediately remove the battery

Button batteries of this size tend to lodge in the esophagus of infants and will not displace themselves. Furthermore, because of the electrical conduction and alkali contents, these batteries continue to cause tissue damage; therefore, if left in place, these batteries will often result in esophageal or tracheal injury which may be fatal. This represents an emergency, and the battery needs to be immediately removed as a life threatening- injury has not yet occurred.

271
Q

A 5-year-old boy with ARDS is on mechanical ventilator. His current settings are as follows: mode: volume control, TV 200, PEEP 8, FiO2 100 %, rate 24. He is on nitric oxide at 20 ppm. On these settings, his mean airway pressure is 30 mmHg and his last blood gas is 7.31/PaCO2 60/PaO2 90. Which one of the following values represents his oxygenation index?

a. 21
b. 24
c. 27
d. 30
e. 33

A

e. 33

Oxygenation index is calculated by the following equation: oxygenation index = MAP × FiO2 / PaO2. In the current question: 30 × 100/90 = 33.

272
Q

A 16-year-old, previously healthy child was admitted to the ICU with concerns of surgical abdomen due to intestinal perforation. His physical examination is significant for tachycardia, abdominal tenderness in the right lower quadrant, and cold extremities. Hypovolemic shock is supected, and intravenous fluid bolus (20 ml/kg) is started and he is given 4 mg of morphine for pain. He remains tachycardic. The surgical team is ready for the patient, and they want to operate on him immediately. The anesthesiologist is told about his clinical condition and advised to hydrate him before induction of inhaled anesthesia. Which one of the following options explains the cardiovascular effects of inhaled halogenated alkane anesthetic agents?

A - Depressed cardiac output and hypotension

B - Increased cardiac output and hypotension

C - Depressed cardiac output and hypertension

D - Increased cardiac output and hypertension

E - No known cardiovascular effects

A

A - Depressed cardiac output and hypotension

Option A is correct because inhaled halogenated alkane anesthetic agents are well known to depress cardiac output and cause vasodilation. This leads to hypotension, which is accentuated in hypovolemic patients like the one in the vignette. Option B and D are incorrect, as inhaled anesthetic agents do not increase cardiac output. Option C is incorrect because depressed cardiac output causes hypotension and not hypertension. Option E is incorrect, as the cardiovascular effects of inhaled anesthetic agents are well documented.

273
Q

A 14-year-old boy is in the ICU following open laparotomy for a perforated appendix. He is intubated and is on volume control mode on a ventilator, with the settings of TV 240, RR 22, PEEP5, and FiO2 80%. His blood gas is 7.20/PaCO2 90/PaO2 100. His end tidal CO2 is reading 60. Which one of the following courses of action would lead to the largest increase in his alveolar minute ventilation?

A - Increase TV to 290

B - Increase RR to 27

C - Increase PEEP to 8

D - Decrease TV to 200

E - Increase FiO2 to 100%

A

A - Increase TV to 290

Since gaseous exchange in the respiratory system occurs only in the terminal portions of the respiratory tract, the gas that occupies the rest of the respiratory system is not available for extended pulmonary capillary blood. Consequently, the alveolar ventilation (i.e., the amount of air reaching the alveoli per minute) is less than the respiratory minute volume. In addition, because of the dead space, rapid shallow breathing produces much less alveolar ventilation then slow, deep breathing at the same respiratory minute volume. Dead space ventilation can be calculated by the following equation: Vd = (PaCO2 - PeCO2) / PaCO2 * Vt. In the current question: Vd = (90 - 60) / 90 × 240. Vd = 80 ml. With this information, it’s possible to calculate his current alveolar ventilation: (240 - 80) × 22 = 160 × 22 = 3520 ml. Of the options given, only increase in TV (option A) and increase in RR (option B) will lead to an increase in minute ventilation. With option A, the resulting alveolar ventilation will be (290 - 80) × 22 or 210 × 22 = 4620 ml. With option B, the resulting alveolar ventilation will be (240 - 80) × 27 or 160 × 27 = 4320 ml.

274
Q

A 10-year-old child with pneumonia is on mechanical ventilator. His current ventilator settings are TV 300, rate 20, FiO2 90%, PEEP 7. PaO2 on his blood gas is 240 mmHg. Assuming his minute ventilation, cardiac output, and VQ mismatching does not change, to achieve a desired PaO2 of 80 mmHg, what should the FiO2 be changed to?

A - 21%

B - 30%

C - 50%

D - 60%

E - 70%

A

B - 30%

The desired FiO2 can be calculated by the following equation: Desired FiO2 = PaO2 (desired) × FiO2 (known) / PaO2 (known) 80 × 90 / 240. Desired FiO2 = 30%.

275
Q

A 6-day-old infant with uncomplicated newborn nursery course was discharged home within 2 days. He presents to the ER with a history of poor feeding, irritability, and pale extremities. There is no documented fever. The peripheral perfusion is compromised with the capillary refill time of more than 4 seconds. The cardiovascular examination shows normal heart sounds with a gallop, and no heart murmurs are present. Femoral pulses are barely palpable with good brachial pulses. Once the initial ABC’s are taken, which one of the following recommendations is most appropriate for this patient?

A - Obtain an electrocardiogram

B - Start a prostaglandin drip

C - Obtain a chest x-ray

D - Administer furosemide

E - Administer antibiotics

A

B - Start a prostaglandin drip

The infant described above has a diagnosis of coarctation of the aorta, unless proven otherwise. The infant appeared well at discharge from the nursery due to the fact that the patent ductus arteriosus (PDA) was open at the time. As the ductal tissue involutes, the coarctation becomes very evident. Since the coarctation is severe and the PDA is closing, every effort should be made as soon as possible to open the duct before any elaborate diagnostic tests are undertaken. This will help with the antegrade blood flow until the severity is judged and further management can be undertaken. Thus, prostaglandins have to be started immediately once there is a suspicion of coarctation. An electrocardiogram might show nonspecific left ventricular hypertrophy, but will not be helpful in the management of this patient. A chest x-ray has no specific finding in coarctation of the aorta other than nonspecific cardiomegaly occasionally. This patient may need furosemide to help treat the heart failure, but it is not the immediate next step. There is no evidence of fever to justify antibiotics before transfer

276
Q

A child has been admitted in the pediatric ICU for 4 days. He presented in septic shock requiring fluid and vasopressors as part of his resuscitation. During his time in the pediatric ICU, he has received no formal nutritional support and remains persistently hyperglycemic. What is the primary source of the glucose at this time?

A. Adipose tissue

B. Hepatic glycogen

C. Muscle glycogen

D. Muscle protein

E. Serum albumin

A

Answer: D - Muscle protein

During the initial stress response (first 24 hours), children derive the majority of their serum glucose from hepatic glycogen. Glycogen found in muscle would be largely depleted after 4 days of sepsis without formal nutrition. Glucose derived from muscle glycogen cannot be transported out of the muscle cell because of lack of glucose phosphatase. Subsequent to glycogen metabolism, adipose tissue would be the next resource utilized for energy; however, adipose tissue and visceral proteins (albumin, immunoglobulin) are not major sources of glucose.

In a proinflammatory state, cytokines(specifically interleukin-1 and tumor necrosis factor-alpha) induce the stress response that leads to lean muscle catabolism. This leads to the release of amino acids. Amino acids, especially alanine and glutamine, are then utilized for gluconeogenesis to create glucose.

277
Q

During forced expiration, intraluminal pressure at the “equal pressure point” is most nearly equal to which one of the following options?

A - Alveolar pressure

B - Static recoil pressure

C - Transthoracic pressure

D - Atmospheric pressure

E - Pleural pressure

A

C - Transthoracic pressure

Along the airway, intraluminal pressure falls progressively from alveolar pressure in the periphery to atmospheric pressure at the airway opening. Downstream of the equal pressure point where intraluminal equals transthoracic pressure, the airway is dynamically compressed. Once the forced expiration is flow limited, more transthoracic pressure will rather effect more compression than a further increase in flow. It follows that the achieved maximum expiratory flow rates are then exclusively defined by the resistance of the intrathoracic airways and no longer by muscular effort. The end of a forced expiration is determined by the elastic resistance of the thoracic cage and by airway closure.

278
Q

A 2-year-old girl is on venovenous extracorporeal membrane oxygenation (VV ECMO) for acute hypoxic respiratory failure. During morning rounds, patient is on ECMO flows of 100 ml/kg with premembrane O2 saturation of 75%, postmembrane O2 saturation of 100%, and cephalad catheter O2 saturation of 65%. You do not make any changes to extracorporeal membrane oxygenation (ECMO) parameters. In the afternoon, patient’s premembrane O2 saturation increases to 90% and cephalad O2 saturation decrease to 55%. ECMO flows are still 100 ml/kg with post membrane O2 saturations of 100%. Which one of the following options is the most likely cause for these changes in O2 saturations?

a. Thrombus formation in the membrane oxygenator
b. Improved cardiac output of the patient
c. Decreased intravascular volume of the patient
d. Cephalad catheter affecting ECMO flows
e. Partial occlusion of ECMO catheter

A

c. Decreased intravascular volume of the patient

These changes of O2 saturations on ECMO reflect increased recirculation fraction from 28%-77%. This will adversely affect the oxygen supply of the patient. The factors that affect recirculation on ECMO are pump flow, catheter position, cardiac output, and right atrial size (intravascular volume). Option A is incorrect. Thrombus formation in the membrane oxygenator will decrease post membrane O2 saturations and can affect flows. In this case, both have not changed. Option B is in correct because if patient’s native cardiac output has increased it will decrease premembrane O2 saturations. Option C is the correct response because decreased intravascular volume or right atrial size will increase recirculation and hence increase premembrane O2 saturations and decrease mixed venous saturations. Option D is incorrect. Cephalad catheter augments the amount of venous drainage and will not increase recirculation. Option E is incorrect. Partial occlusion of catheter may decrease ECMO flows and will not affect recirculation.

279
Q
  1. Which one of the following fluid compositions most closely resembles the recommendations of the modified Brooke formula for resuscitation of burn injuries during first 24 hours after injury?
    a. 4 ml/kg/%total body surface area (TBSA) burn with one half during first 8 hours and next half in 16 hours b. 2 ml/kg/%TBSA burn with 1 ml/kg of crystalloid and 1 ml/kg of colloid over 24 hours c. 4 ml/kg/%TBSA burn with 2 ml/kg of crystalloid and 2 ml/kg of colloid over 24 hours d. 20 ml/kg fluid bolus followed by 3 ml/kg/%TBSA burn over 24 hours e. 2 ml/kg/%TBSA burn of crystalloids over 24 hours without any colloids
A

e. 2 ml/kg/%TBSA burn of crystalloids over 24 hours without any colloids

Consensus fluid resuscitation by standardized formula has not been reached to treat burn injuries. Original Parkland formula recommends 4 ml/kg/%TBSA burn fluids with 50% given during first 8 hours and remaining given over next 16 hours. Originally, the Brooke formula recommended 1.5 ml/kg/%TBSA burn of crystalloid and 0.5 ml/kg/5TBSA burn of colloid. Modified Brooke formula recommends 2 ml/kg/%TBSA burn over first 24 hours after injury and no colloids. Option A is incorrect. This is Parkland formula. Options B, C, and D are incorrect. None of the recommended formulas resemble these compositions. Option E is correct because this is the modified Brooke formula which has shown to achieve same end points as the Parkland formula.

280
Q

A 60 kg, 16-year-old boy is admitted to PICU for management of splenic injury he sustained during high school football game. His heart rate on admission is 110/minute; blood pressure is 94/48 mmHg; O2 saturation on room air is 96%. His hemoglobin on admission was 5.2 gms/dL and platelet count was 140,000 per mm3. CT scan of abdomen showed grade IV splenic injury. He has received 2500 ml of blood transfusion. His hemoglobin now is 7.6 gm/dL and platelet count is 90,000 per mm3. His last blood transfusion was 8 hours ago. Which one of the following statements regarding the management of this patient is true?

A - Operative intervention is indicated for this patient

B - This patient needs activity restriction for 6 weeks after discharge

C - He needs abdominal CT scan before discharge

D - He needs abdominal CT scan during follow-up visit

E - He is likely to stay in the hospital for 8 more days

A

B - This patient needs activity restriction for 6 weeks after discharge

This is a patient with grade IV splenic laceration who had significant blood loss. He is now stable after 40 ml/kg of blood transfusion with acceptable blood pressure and no oxygen requirement. Indications for surgical intervention in splenic injury patients are persistent hypotension, >50% blood volume replacement, or additional life-threatening abdominal injuries. In isolated splenic injury need for surgical intervention is very rare. Option A is incorrect due to the reasons stated above. Option B is correct. Proposed activity restriction for grade IV splenic injury is 6 weeks. Options C and D are incorrect because repeat abdominal CT scans for splenic injury is not recommended. Option E is incorrect. Average duration of hospital stay for grade IV splenic injury is 5 days.

281
Q

In which one of the following stages is pertussis most contagious?

A - Paroxysmal

B - Catarrhal

C - Convalescent

D - Febrile

E - Chronic

A

B - Catarrhal

This question describes the stage of pertussis when the disease is most contagious. Option A is incorrect as the most infectious stage is not the paroxysmal stage, which is when the characteristic “whooping” cough occurs. Option B is correct, as the first stage (called the catarrhal stage) is when the disease is most contagious. During the catarrhal stage, the patient experiences symptoms similar to common upper respiratory infectious including sneezing, coughing, nasal congestion, and a low-grade fever. Option C is incorrect, as the third stage of pertussis (the convalescent stage) is not the most infectious stage. In the convalescent stage, the patient experiences a chronic cough. Options D and E are incorrect, as they are not stages of pertussis.

282
Q

A 16-year-old, healthy male is admitted to the pediatric ICU with a first episode of atrial fibrillation. The episode lasted for more than 24 hours. His initial heart rate was 160 bpm. He was started on a diltiazem drip to control his heart rate. The next plan was elective cardioversion. Which one of the following diagnostic tests is most appropriate to rule out any clots in the heart prior to cardioversion?

A - Electrocardiogram

B - Transthoracic echocardiogram

C - Transesophageal echocardiogram

D - Cardiac MRI

E - V/Q scan

A

C - Transesophageal echocardiogram

Any patient who has to undergo an elective cardioversion for atrial fibrillation lasting for more than 24 hours needs an imaging study to rule out clots, especially in the left atrial appendage. The diagnosis of choice is a transesophageal echocardiogram, which can effectively rule out any clots due to its excellent acoustic windows. An electrocardiogram is helpful in determining the rhythm, but does not assist in locating clots. While a transthoracic echocardiogram is a good diagnostic modality as a screening test for looking at the cardiac function and significant clots, it can miss small clots due to the limitations and difficult acoustic windows. A cardiac MRI is a good test to detect clots, but difficult to obtain in a patient with atrial fibrillation since the test needs electrocardiogram gating; therefore, this is not the test of choice. Finally, an AV/Q scan is a test obtained in the workup of pulmonary embolism.

283
Q

A 3-year-old child with unknown muscular dystrophy is scheduled to undergo a muscle biopsy for diagnostics purposes. Anesthesia is induced using sevoflurane and you start injecting succinylcholine in preparation for intubation. You immediately notice masseter spasm and his temperature is 41 °C. After immediate discontinuation of inhaled anesthesia and succinylcholine, which one of the following subsequent steps is the most appropriate?

A - Start 100% oxygen with high flow rate to wash out residual sevoflurane

B - Start normal saline bolus to avoid hypotension

C - Intravenous injection of dantrolene

D - Call ICU for arranging a bed for this patient

E - Paralyze and intubate the patient with another agent

A

A - Start 100% oxygen with high flow rate to wash out residual sevoflurane

The patient in this vignette has features of malignant hyperthermia (MH). Patient with muscular dystrophy and myotonia are at increased risk of MH when exposed to inhalational anesthetic agents like halothane, sevoflurane, isoflurane, and/or succinylcholine. Features of MH are muscle rigidity, fever, increased carbon dioxide production, acidosis, and tachycardia. It is important to recognize this potentially fatal condition immediately. Immediate intervention in order of sequence includes discontinuation of offending agents, oxygenation with 100% oxygen to wash out the offending inhalational agent, intravenous injection of dantrolene, use of cold normal saline if temperature above 30 °C. Hence, option A is correct. All other interventions are part of management; however, getting rid of the offending inhalation agent is of prime importance. Option E is incorrect because you will try to manage MH and avoid paralysis and intubation in this patient.

284
Q

A 4-year-old child presents to the emergency department with a cerebrovascular accident. His rhythm strip appeared abnormal, and an ECG shows biatrial enlargement and intermittent atrial flutter. An echocardiogram shows dilated atria, a left ventricle ejection fraction of 62%, normal left ventricle end-diastolic dimension, and a right ventricle pressure estimated at 55% of systemic pressure. Which of the following is the most likely diagnosis?

A. Acute myocarditis

B. Dilated cardiomyopathy

C. Restrictive cardiomyopathy

D. Hypertrophic cardiomyopathy

E. Pulmonary hypertension and right ventricle failure

A

Answer: C - Restrictive cardiomyopathy

Restrictive cardiomyopathy is characterized by diastolic ventricular dysfunction with systolic function often preserved. The clinical presentation may be subtle, with growth failure or exercise intolerance. Arrhythmias are a common presenting sign. Marked atrial dilation can lead to atrial flutter, atrial fibrillation, or supraventricular tachycardia. Atrial thrombi may form resulting in stroke. The left atrial pressure is elevated, resulting in pulmonary hypertension caused by impaired pulmonary venous drainage.

Acute myocarditis and dilated cardiomyopathy are characterized by decreased left ventricle ejection fraction and increased left ventricle end-diastolic dimension. Children with hypertrophic cardiomyopathy do not have dilated atria but have hypertrophied ventricles. Patients with pulmonary hypertension sometimes have dilated right atrium from tricuspid regurgitation, but the left atrium usually is not dilated.

285
Q

Which of the following inborn errors of metabolism presents with metabolic acidosis and an increased serum lactate level?

A. Biotinidase deficiency

B. Isovaleric acidemia

C. Maple syrup urine disease

D. Myoclonic epilepsy with ragged red fibers (MERRF)

E. Propionic acidemia

A

Answer: D - Myoclonic epilepsy with ragged red fibers (MERRF)

Acidosis is a common presentation of children with inborn errors of metabolism. The etiology of the acidosis is key in pointing the clinician to the correct diagnosis.

Typically, the organic acidemias and aminoacidopathies present with an anion gap acidosis due to accumulation of the abnormal organic acids or ketoacids. This increased anion gap is not due to lactate accumulation. The common symptoms associated with this group of inborn errors include vomiting and encephalopathy.

Biotinidase deficiency is an inherited disorder in which the body is unable to reuse and recycle the vitamin biotin. This disorder is classified as a multiple carboxylase deficiency, a group of disorders characterized by impaired activity of certain enzymes that depend on biotin.

Mitochondrial disorders like myoclonic epilepsy with ragged red fibers (MERRF) and mitochondrial encephalomyopathy, lactic acidosis, and stroke-like episodes (MELAS) present with disorder of oxidative phosphorylation and lactic acidosis.

286
Q

A child with tricuspid atresia and prior aortopulmonary shunt returns from surgery after undergoing bidirectional cavopulmonary anastomosis. The oxygen saturation is 50%. The superior vena cava (SVC) pressure and pulmonary artery pressure are 28 mm Hg and 30 mm Hg, respectively. The left atrial pressure is 8 mm Hg. What is the most likely reason for hypoxemia with these hemodynamic findings?

A. Poor ventricular compliance

B. Mitral valve regurgitation

C. Elevated pulmonary vascular resistance

D. Thrombosis at the site of the anastomosis

A

Answer: C - Elevated PVR

Following a bidirectional cavopulmonary anastomosis, there are many possible causes of hypoxemia. If the superior vena cava–pulmonary artery pressure (SVC-PAP) gradient is elevated (>2 mm Hg), then obstruction of SVC-PA anastomosis must be considered. If SVC pressure and the transpulmonary gradient are elevated (transpulmonary gradient >7 to 10 mm Hg), then increased pulmonary vascular resistance should be considered. If both SVC and left atrial pressures are elevated, problems to consider include ventricular dysfunction, atrioventricular valve regurgitation, and anomalous venous connection.

287
Q

Which of the following is the most common clinical occurrence in the adolescent following repair of coarctation of the aorta?

A. Left ventricular dysfunction

B. Ischemia of the artery of Adamkiewicz during aortic cross-clamp time

C. Rebound hypertension

D. Mesenteric arteritis

E. Gradient of 25 mm Hg between upper extremity and lower extremity

A

Answer: C (Rebound hypertension)

Coarctation of the aorta in the older child causes high obstruction of systemic blood flow and leads to left ventricle overload and pulmonary edema. This obstruction causes aortic collaterals to be present. The relief of this obstruction can cause a rebound hypertension postoperatively. In the immediate postoperative phase this is believed to be due to both catecholamine release from manipulation of the aortic arch and the stimulation of the renin-angiotensin system. Hypertension is controlled through beta-blockade, vasodilators, analgesia, and sedation. Spinal cord injury due to lack of collaterals and ischemia to the spinal cord by not perfusing the anterior spinal artery can cause ischemic shock to the cord and paralysis. Mesenteric arteritis can occur as the intestinal vasculature responds to the introduction of pulsatile flow by vasoconstriction. This vasoconstriction can cause intestinal ischemia, which may cause bowel infarction.

Left ventricular dysfunction may be seen in the neonatal period, but rarely in the adolescent population.